Вы находитесь на странице: 1из 312

. .

1
,

, 2004

514.112
22.151.0
56

56

. .
: 2 . . 1: , . .: , 2004. 312 .: .
ISBN 5-94057-170-0
ISBN 5-94057-171-9 ( 1)

. , . ,
, , , .
,
, .

22.151.0

ISBN 5-94057-170-0
ISBN 5-94057-171-9 ( 1)

c . ., 2004.

c , 2004.

. . . . . . . . . . . . . . . . . . . . . . . . . . . . . . . . .

I.
1. , . . . . . . . . 13
1.1. (13). 1.2.
(13). 1.3.
(14).
2. . . . . . . . . . . . . . . . . . . . . . . 16
2.1. (16). 2.2.
(17). 2.3.
(18). 2.4.
(18). 2.5. (19).
2.6. (19). 2.7. (21).
3. . . . . . . . . 25
3.1. (25). 3.2. (26). 3.3. (28).
3.4. (29). 3.5.
(30).
4. . . . . . . . . . . . . . 34
4.1. (34). 4.2. (37). 4.3. (38). 4.4.
(40).
5. . . . . . . . . . . . . . . 45
5.1. (45). 5.2.
(46). 5.3. , (47).
6. . . . . . . . . . . . . . . . 49
6.1. (49). 6.2.
(50).
7. . . . . . . . . . . . . . 53

7.1. (53). 7.2.


(54). 7.3. , (56).
8. . . . . . . . . . . . . . . . . 61
8.1. (61). 8.2. (62).
9. . . . . . . . . . . . . . . . . . . . . . . . . . . . . . . 68
9.1. (68). 9.2. ()
(69). 9.3. (70).
10. . . . . . . . 75
10.1. (75). 10.2. (76). 10.3. (77). 10.4. (78).
10.5. (79).
11. . . . . . . . . . . 82
11.1. (82). 11.2. (83). 11.3.
(85). 11.4. (86). 11.5. (87). 11.6.
(88).
12. . . . . . . . . . . . . . . . . . . . . . 91
12.1. (91).
12.2. (92).
13. . . . . . . . . . . . . . . . . . . . . . 97
13.1. (97). 13.2.
(99). 13.3. (101). 13.4. (102). 13.5. (103). 13.6.
(105). 13.7. ,
(106).
14. . . . . . . . . . . . . . . . . . . . . . 110
14.1. (111). 14.2. (112). 14.3. (113).
14.4. (113). 14.5. (114).
15. . . . . . 118
15.1. (118). 15.2. (119). 15.3.
4

(119). 15.4.
(121). 15.5. ,
(123). 15.6. ,
(124).
16. . . . . . . . 126
16.1. (126). 16.2. (126). 16.3.
(128). 16.4. (129).
17. . . . . . . . . . . . . . . . . . . . . . . . . . . 130
17.1. . (130).
17.2. . (132). 17.3. (133). 17.4.
(134).
18. . . . . . . . . . . . . . . . . . . . . . . . . 136
18.1. (136). 18.2.
(138). 18.3. . (138).
18.4. .
(139).
. . . . . . . . . . . . . . . . . . . . . . . . . 143

II.
. . . . . . . . . . . 157
I.
1. . . . . . . . . . . . . . . . . . . . . . . . 160
1.1. (160). 1.2. (160). 1.3. (162). 1.4. (163).
2. . . . . . . . . . . . . . . . . . . . . . . . . 164
2.1. (164). 2.2. (165).
3. . . . . . . . . . . . . . . . . . . . . . . . . . . . . 169
3.1. (169).
3.2. (171).
4. . . . . . . . . . . . . . . . . . . . . . . . . . . . . . . . . . . 175
4.1. (175). 4.2.
(176).
5

5. . . . . . . . . . . . . . . . . . . . . . . . . . . . . . . . . . . 179
5.1. (179). 5.2. (180). 5.3.
(181).
6. . . . . . . . . . . . . . . . . . 181
7. . . . . . . . . . . . . . . . . . . . . . . . . . 187
7.1. (187).
7.2. (188). 7.3. (189). 7.4. (189). 7.5.
(190). 7.6. (190).
7.7. (191). 7.8. (191). 7.9. (192).
7.10. (192).
8. . . . . . . . . . 193
9. . . . . . . . . . . . . 197
9.1. (197). 9.2. (198). 9.3. (198). 9.4. I II (199). 9.5. (200).
10.
. . . . . . . . . . . . . . . . . . . . . . . . . . . . . 202
10.1. (202). 10.2. (203).
11. . . . . 206
11.1. (206). 11.2. (207).

12.

13.

14.
15.

II.
. . . . . . . . . . . . . . . . . . . . . . . . . . . . . . . . 211
12.1. (211). 12.2. (212). 12.3. ,
(213). 12.4. .
(213).
. . . . . . . . . . . . . . . . . . . . . 214
13.1. (214). 13.2. (215).
. . . . . . . . . . . . . . . . 216
. . . . . . . . . . . . . . . . . . . . . . . . 223
6

16.
17.

18.

19.

20.

21.
22.
23.

24.

15.1. (223). 15.2. (224).


15.3. (225). 15.4. (226).
. . . . . . . . . 227
. . . . . . . . . . . . . . . . . . . . . . . 230
17.1. (230). 17.2. .
(231).
. . . . . . . . . . . . . . . . . . . . . . 232
18.1. (232). 18.2.
. (232).
. . . . . . . . . . . . . . . . . 233
19.1. (233). 19.2. (235).
, . . . . . . . . . . . . . . 237
20.1. (237). 20.2. (237). 20.3. (238). 20.4. (238).
. . . . . . . . . . . . . . . . . . . 239
. . . . . . 249
. . . . . . . . . . . . . . . . . . . . . . . 251
23.1. (251). 23.2. (252). 23.3. (253).
. . . . . . 254

III.
25. . . . . . . . . . . 259
25.1. . (259). 25.2. (260). 25.3. (260).
26. . . . . . . . . . . . . . . . . 262
26.1. (262). 26.2. (262).
27. . . . . . . . . . . . . . . . . . . . . . 264
27.1. (264). 27.2.
(264).
28. . . . . . . . . . . . . . . . . . . . . . . . . . 265
29. . . . . . . . . . . . . . . . . . . . . . . . . . . . . . . . . 266
, , . . . . . . . . . . . . . . . . . . . . . . . . . 271
. . . . . . . . . . . . . . . . . . . . . . . . . . . . . . . . . . 312
. . . . . . . . . . . . . . . . . . . . . . . . . . . 315
7


. ,
.
, .
, ,
.
, ,
. ,
, . , , . ,
,
.
, , .
. .

.
, . . -
.
. , , .

, ,
, .
, , ,
. . -
8

. , ,
, .
,

.

. , .
,
.

. . , .
. .
. , . ,


.
. , .
,
,

. . .
19661993 [29]. , .
. ,
.
. ,
. , , . ,
, .
9

, ,
. , .
.
, .
. .

10

1. ,

, () . , , . .
1.1. . S
ASB ,
A B (. 1). , SA
SB, C D.
ASB AB
CD. ASB SBC.
ASB = ACB + DBC.
AB CD.
1
ASB = (`AB + `CD).
2
,
,
, .
, BE, AC (. 2). ASB = DBE =
1

= `DE = (`DC + `CE). AB CE ,


2
2
,
AC BE.
A

C
C

B
E
. 1

. 2

1.2. . ,
, ,
.
13

, ASB C D (. 3). CBD


SBC CBD = ASB + ACB, ,
CD AB, CBD ACB,
1
: ASB = (`CD `AB).
2
BE,
AC (. 4):
1
2

1
2

1
2

ASB = DBE = `DE = (`DC `CE) = (`DC `AB).


C

. 3

. 4

1.3.
(. 5) (. 6). ,
ASB , BSD 1
1
1
ASD. , ASB = 90 `DA = `SD `DA =
2

= `SA. ASB , 2
. , ,
, .
D

A
A

. 5

. 6


1
SAB = `AB ,
2
(. 6) :


1
2

1
2

1
2

ASB = ABC SAB = `AC `AB = (`AC `AB).


14

,
,
, , .
1. ABCD .
AB CD a, AD BC
b. .
. AB, BC, CD DA x, y, z t . g AC
BD (. 7). x + y + z + t = 360 . 1

: a = (t y), b = (x z), g = (y + t).


2
2
2
: t = a + g, y = g a, x = 180 + b g,
z = 180 b g. :
1
2
1
ABC = (z + t) =
2
1
BCD = (x + t) =
2
1
ADC = (x + y) =
2

1
2
1
(180 + a b),
2
1
(180 + a + b),
2
1
(180 a + b).
2

BAD = (y + z) = (180 a b),

2. ABC A1 B1 , AA1 BB1 , AB a.


C .
. A1 B1 AB , AB (. 8).
1
1
a
C = (`AB `A1 B1 ) = (180 a) = 90 .
2

a
B

B1

A1

g
A

D
t
. 7

. 8

15


1.1. , ,
, .
1.2. A. M N . ,
M AN .
1.3. S. AB P .
, SP ASB .
1.4. ABC ,
AB AC D E. , ,
ADE, .
1.5. ABCD. A1 , B1 ,
C1 , D1 AB, BC, CD, DA. , A1 C1 B1 D1 .
1.6. ABC .
A, B, C A1 ,
B1 , C1 . , AA1 , BB1 , CC1 A1 B1 C1 .
1.7. AA1 , BB1 , CC1 ,
ABC,
A1 , B1 , C1 . ,
A1 B1 C1 .
1.8. B C ABC
I. C ABC D. , AD = DI.

2.
2.1. .
a1
a2
an
=
= ... =
= k,
b1

b2

bn

ai = kbi (i = 1, 2, . . . , n). t1 , t2 , . . . , tn , t1 b1 + t2 b2 + . . . + tn bn 6= 0. t1 a1 = t1 kb1 ,


t2 a2 = t2 kb2 , . . . , tn an = tn kbn . , : t1 a1 +
+ t2 a2 + . . . + tn an = k(t1 b1 + t2 b2 + . . . + tn bn ),
ai
t1 a1 + t2 a2 + . . . + tn an
=k= ,
t1 b 1 + t2 b 2 + . . . + tn b n
bi

16

i = 1, 2, . . . , n.

(2.1)

. , t1 = t2 = . . . = tn = 1
ai
a1 + a2 + . . . + an
= ,
b1 + b2 + . . . + bn
bi

i = 1, 2, . . . , n.

, ai bi
, , .
2.2. . , , ,
C
,
(. 9):
B
C2

OA
AB
BC
=
=
= ...
OA1
A1 B1
B1 C1

B2

A1
B1
C1
AB2 , BC2 , . . . , O
OA1 O. . 9
OAA1 , ABB2 , BCC2 , . . . OA1 , AB2 , BC2 , . . .
AA1 , BB1 , CC1 , . . . :
OA
AB
BC
=
=
= ...
OA1
AB2
BC2

AB2 = A1 B1 , BC2 = B1 C1 , . . . , .
, OA = AB = BC = . . . , OA1 = A1 B1 = B1 C1 = . . .
,
, ,
( ).
. O
OA, AB, BC, . . . O
OA1 , A1 B1 , B1 C1 , . . . (. 9):
AB
BC
OA
=
=
= ...,
OA1
A1 B1
B1 C1

AA1 , BB1 , CC1 , . . . .


17

OB
OA + AB
OA
OB
OA
=
=
, . .
=
. ,
OB1
OA1 + A1 B1
OA1
OB1
OA1

OAA1 OBB1 AA1 k BB1 . AA1 k CC1 .


, OA = AB = BC = . . . OA1 = A1 B1 = B1 C1 = . . . ,
AA1 , BB1 , CC1 , . . . . (
.)
2.3. .
,
, O
(. 10):

AB
BC
CD
=
=
= ...
A1 B1
B1 C1
C1 D1

,
O

A1 A. B1 B, C1 C,
A1
B1 C1
D1
D1 D, . . . . 10

.
, OAB OA1 B1 , OBC OB1 C1 , OCD OC1 D1 .
2.4. . , E
.
. CD ABC (. 11).
C
BE k CD. ACD = AEB BCD =
= CBE, ACD = BCD, AEB =
= CBE BC = CE. . 2.2
AC
AD
AC
AD
=
, . .
=
, DB
CE
DB
BC

A
D
.

. 11
.
() .
,

18

. , :
ABC
, , C


E
(. 12):
DA
AC
=
.
DB
BC

. 12
.
2.5. . ,
, .
. ,
, ,
, , .
M AB CD (. 13 14). M AD M CB
, B = D A = C .
, M , M AD M BC
.
MA

MC

=
, M A M B = M C M D, MD
MB
.
B
C

A
M

D
M
D
. 13

C
. 14

2.6. .
a : x = x : b,
x ( ) a b.

x2 = ab x = ab.
19

,
.
ABC CD AB (. 15). : BC = a,
CA = b, AB = c, CD = h, AD = b1 , BD = a1 . ABC ACD ( A).
ABC BCD. C
, ACD
BCD.
:
b

h
b1

a1
D

c
. 15

c
b
= ,
b1
b

a
c
= ,
a1
a

b2 = cb1 ,

a2 = ca1 ,

b1
h
= ,
h
a1

h2 = a1 b1 .

,
, ,
h
,
A
B
D O
.
C

AB.
. 16

(. 16):
, ,
, , ,
,
.
. a2 = ca1 b2 = cb1 , :
2
a + b2 = (a1 + b1 )c = c2 . :
C

a2
a1
= .
b2
b1

, :
1)
, 2) .
20

M , , AB M C (. 17). M C 2 = M A M B. , M AC M BC C
: M M
ACM ABC ( A
AC).
.
B
,
. 17
, , ,
,
.
2.7. . : a ,

( ).
. x a,
x2 = a(a x), x2 + ax a2 = 0,
r 

a 2
51
a
a.
+ a2 =
x = +
2

A
a/2
P
a/2
x
x
a

. 18


x a
.
ABC a
a (. 18).
2
AB
r

a 2
+ a2 . 2

AP = , BP = x, S,
2
BC = a

BS
x
a
2
5+1
=
= =
.
=
SC
ax
x
2
51

5+1
t =
1,61803398 . . .
2

, , , p ( ), ,
21

. , = t 1 0,61803398 . . .
t
t ,
.
t . . . .
.
1. ,
,

C
.
. C b
l
a
ABC AB D, m
n
E (. 19). AD = m, A
D
B
DB = n, CD = l,
(. 2.5) mn = l DE = l(CE l) = l CE l2 .
ACE CDB E
: l CE = ab. mn = ab l2 ,
. 19
l2 = ab mn.
2. l l1 . ,
O, l A, B, C, D, l1 A1 , B1 , C1 , D1 . ,
A1 C 1 A1 D1
AC AD
:
=
:
.
CB DB
C1 B 1 D1 B 1

O
Q
A

Q1

P1

l
l1

B1

C1

D1

.
B B1 , OA (. 20). OC OD P
Q, P1 Q1 . -

AC
AO
AD
AO
=

=
. CB
PB
DB
QB
AC AD
QB
:
:
=
.
CB DB
PB
A1 C 1 A1 D1
Q1 B 1

:
=
.
C1 B 1 D1 B 1
P1 B 1
QB
Q1 B 1

=
.
PB
P1 B 1
AC AD
.
:
(CB DB
A1

. 20

) A, B, C, D .
22

: .
2.2.

2.1. , . 2.3.
2.2. AB. A M ,
N B. , AM AN M .
2.3. . , , , .
2.4. , ,
,
.
2.5. , ,
, ,

.
2.6. ABC C B.
AB C, BC = a AC = b.
2.7. ABC , A, BC D.
AD CD, a, b, c ABC.
2.8. ABCD. , A BC, BD M . , B AD, AC N .
, M N CD .
2.9. . , .
, AB

.)
. ( A, D, C
CD
2.10. w P K . P
w, A B.
23

, P K ,
K, A, B, AB.
2.11. , M N , P Q. ,
M Q .
2.12. ,

.
2.13. ,
.
2.14. .
2.15. O AB CD. E OA.
EB EK, CE. ,
KC ,
.
2.16. . AP , BP , CP BC, CA, AB, ABC, A1 , B1 , C1 ,
:
AC 1
AB 1
AP
+
=
.
C1 B
B1 C
P A1

2.17. ,
.
2.18. , , .
2.19. , , .
.
2.20. ABC CD
AB. , ACD
BCD, r1 r2 . , ABC.
2.21. ABC BK BM ,
AM = BM . KBM , AB = 1, BC = 2.
2.22. S, O, , A B,
, M N . AB SO
24

K. , M , N , K, O
.
2.23. ABCD , A C BD.
, B D AC, A C BD, B D AC. ,

.

3.

3.1. .
.
OD , ABC, BC (. 21).
BAC BOD , BC. E BC OD BC.
a
BOE :
= R sin A, a =
2
A
= 2R sin A, R .
b = 2R sin B c = 2R sin C.
c

b
c
a
=
=
= 2R.
sin A
sin B
sin C

(3.1)
B

a/2

, , A
, .
D
. 21
:
, , .
:
a sin B = b sin A,

b sin C = c sin B,

c sin A = a sin C.

(3.2)

. , ,
, , .
25

. CD ABC, DP DQ
AC BC (. 22). P Q CD .
CD. P Q =
2S

= CD sin ACB. CD =
,
AB
S ABC,
P Q = 2S

sin ACB
S
= ,
AB
R

R , ABC. A
B P Q
D
.
3.2. .
. 22
D
A ABC BC.
B , C : 1) C , 2) C , 3) C (. 23). 1) a = BD + DC, 2) a = BD DC,
3) a = BD. BD = c cos B, 1)
DC = b cos C, 2) DC = b cos C, 3) DC = 0 = b cos C.
:
Q

a = b cos C + c cos B.

(3.3)

b = c cos A + a cos C,

c = a cos B + b cos A

1)

2)

C D

B
3)

. 23

.
. BC = BA + AC
26

BC: BC 2 = BA BC + AC BC. a2 = ca cos B + ba cos C,


a = b cos C + c cos B.
(3.3) .
, (3.3) :
a < b + c,

b < c + a,

c < a + b,

(3.4)

.
, (3.3) a, b, c , : c2 a2 b2 = 2ab cos C
c2 = a2 + b2 2ab cos C,

(3.5)

. . .
ab cos C = CA CB,
:
2CA CB = a2 + b2 c2 .

(3.6)

. (3.3)
a, b, c:

a + b cos C + c cos B = 0,
a cos C b + c cos A = 0,

a cos B + b cos A c = 0.
(a, b, c),
, :


1 cos C cos B


(3.7)
cos C 1 cos A = 0.
cos B cos A 1
. ,
cos2 A + cos2 B + cos2 C + 2 cos A cos B cos C = 1.

(3.8)

,
, (3.8)
27

a, b, c (3.3). (3.3)
c = a cos B + b cos A :
a(1 cos2 B) = b(cos C + cos A cos B),
b(1 cos2 A) = a(cos C + cos A cos B).
:
(1 cos2 A)(1 cos2 B) = (cos C + cos A cos B)2 ,
(3.8).
3.3. . :
1
1
abc
S = ah, S = ab sin C, S = pr, S =
,
2

4R

r R 1
, p = (a + b + c) .
2
. .
1
1) S = ab sin C,
2

a2 + b2 c2

. cos C =
. 2ab 

2
2
2 2
(a
+
b

c
)
4S 2 = a2 b2 sin2 C = a2 b2 (1 cos2 C) = a2 b2 1
,
2 2
4a b

16S 2 = 4a2 b2 (a2 + b2 c2 )2 = (2ab + a2 + b2 c2 )(2ab a2 b2 + c2 ) =


= ((a + b)2 c2 )(c2 (a b)2 ) = (a + b + c)(a + b c)(a b + c)(a + b + c).
, , a + b c = a + b + c 2c = 2(p c),
S 2 = p(p a)(p b)(p c).

(3.9)

,
.
2) O ABC . ABC , O . , BOC, COA, AOB
ABC.
1
2

1
2

1
2

SABC = SBOC + SCOA + SAOB = R2 sin 2A + R2 sin 2B + R2 sin 2C.


28

,
1
2

S = R2 (sin 2A + sin 2B + sin 2C).

(3.10)

, , A ,
BOC , (3.10),
sin 2A < 0.
3) BOC,
1
2

BC, R cos A (. 21). , S = aR cos A


1
2

S = R(a cos A + b cos B + c cos C).

(3.11)

.
4) A, B
C x,
y z . a = y + z, b = x + z, c = x + y (. 24)
2p = a + b + c = 2(x + y + z). :
A

x = p a,

B1

C1

A1

z = p c. (3.12)

I . AIB1 -

y = p b,

: r = x tg = (p a) tg .
2
2
S = pr :

. 24

S = p(p a) tg

A
.
2

(3.13)

A
, (3.9)
2
A
: S 2 = p(p a)(p b)(p c) = pr(p b)(p c) ctg . pr = S,
2

5) , , p a = r ctg

S :
S = (p b)(p c) ctg

A
.
2

(3.14)

3.4. . (3.3)
, :
2p = a + b + c = (b + c) cos A + (c + a) cos B + (a + b) cos C =
= 2p(cos A + cos B + cos C) (a cos A + b cos B + c cos C).
29

(3.11), a cos A + b cos B + c cos C =


2p = 2p(cos A + cos B + cos C)

2pr
,
R

2S
2pr
=
. ,
R
R

cos A + cos B + cos C = 1 +

r
.
R

(3.15)

. d1 + d2 + d3
R + r
( ).
, (3.15) :
R cos A + R cos B + R cos C = R + r.
R cos A = d1 , R cos B = d2 , R cos C = d3 ,
d1 + d2 + d3 = R + r.
. ABC M , AB, M
AB , M C
AB, , AB.
3.5. . .
C ABC
AB D. ABC ACD BCD, :
1
C
1
C
1
bl sin + alc sin = ab sin C,
2 c
2
2
2
2

lc =

ab sin C
(a + b) sin

2ab cos

=
C

a+b

C
2 .

(3.16)

lc a, b c ABC
. , O ,
AM
OA + lOB
= l, OM =
, l 6= 1 (
1+l
MB
AB l).
, AM = OM OA M B = OB OM .
30

CD ABC. . 2.4
b
AD
=
a
DB

CD =

CA + (b/a)CB
aCA + bCB
=
.
1 + b/a
a+b

CD2 =

1
(a2 CA2 + 2ab(CA CB) + b2 CB 2 ) =
(a + b)2
1
=
(2a2 b2 + 2ab(CA CB)).
(a + b)2

2CA CB = a2 + b2 c2 . ,
lc2 =

ab
(2ab + a2 + b2 c2 ) =
(a + b)2
ab(a + b + c)(a + b c)
ab
((a + b)2 c2 ) =
.
=
(a + b)2
(a + b)2

,
lc2 =

ab(a + b + c)(a + b c)
(a + b)2

(3.17)

4abp(p c)
.
(a + b)2

(3.18)

lc2 =

. , ,
( ).
1840 . . .
.
20 .
XX .
(3.17) .
la = lb ,
ac(a + b + c)(c + a b)
bc(a + b + c)(b + c a)
=
,
(b + c)2
(a + c)2


b(a + c)2 (b + c a) = a(b + c)2 (c + a b).
31

, :
(a3 b ab3 ) + 3abc(a b) + c2 (a2 b2 ) + c3 (a b) = 0
(a b)(a2 b + ab2 + ac2 + bc2 + c3 + 3abc) = 0.
a, b c ,
. , a b = 0, a = b.

3.1. , a
b :
1
2

r = (a + b c).
3.2. , , , ,
, .
3.3. ,
,
.
3.4. a. ,
, , .
.
3.5. p q , A ABC BC B C,
pq = R2 . .
3.6. , :
a(sin B sin C) + b(sin C sin A) + c(sin A sin B) = 0.
3.7. ,
. .
3.8. , , , 1 : 2.
3.9. A ABC
, AB AC BC M N . ,
AB 2
BM BN

=
CM CN
AC 2

( ).
32

3.10. , , -
, .
(3.113.20).
1

sin A sin B

.
3.11. S = c2
2
sin C
2
3.12. S = 2R sin A sin B sin C.
1
3.13. S = (a2 sin 2B + b2 sin 2A).
4

3.14. S = p2 tg tg tg .
2
2
2
3.15. S = Rr(sin A + sin B + sin C).
A
B
C
cos cos .
2
2
2
B
C
A
3.17. S 2 = abcp sin sin sin .
2
2
2
p2
3.18. S =
.
ctg(A/2) + ctg(B/2) + ctg(C/2)

3.16. S = 4Rr cos

3.19. 4S 2 = CA2 CB 2 (CA CB)2 .


3.20. 16S 2 = a2 (b2 + c2 a2 ) + b2 (c2 + a2 b2 ) + c2 (a2 + b2 c2 ).
3.21. , ABC
r = 4R sin

B
C
A
sin sin .
2
2
2

3.22. AB ABC
ABC1 . ,

1
CC12 = (a2 + b2 + c2 ) 2S 3,
2

a, b c ABC, S .

ABC (3.233.25).
3.23. a2 = b2 + c2 4S ctg A.
A
.
2
A
3.25. a2 = (b c)2 + 4S tg .
2

3.24. a2 = (b + c)2 4S ctg

3.26. ABC :
cos2 A + cos2 B + cos2 C = 1.

.
3.27. R A B ABC AC A.
ABC, BAC = a ABC = b.
3.28. , ABC
CD CB
33

2p(p c)
.
a+b

3.29. ABC r.
,
ABC a, b g. , pr2

.

2R
3.30. , :
a2 + b2 + c2 = 2p2 2r2 8Rr.
3.31.

.

4.
: , , .
.
4.1. .
.
. G, 2 : 1,
.
C

.
1. A1 , B1
B1
A1
C1 BC, CA AB ABC (. 25).
G
G P
Q
AA1 BB1 . ,
A
B
C1
AG : GA1 = BG : GB 1 = 2.
P Q . 25
AG BG.
B1 A1 P Q AB . A1 B1 P Q .
G P A1 QB1
. , P G AA1
, Q G BB1
. , G 2 : 1, . ,
34

CC1 G. ,
AA1 CC1 2 : 1.
AA1 G,
AA1 CC1 . .
2 (). O . AA1 G, 2 : 1, A (. 26).

O
(. 3.5) :
C
1
2

OA1 = (OB + OC)

A1
G
B

OG =

OA + 2OA1
.
1+2

. 26

1
3

OG = (OA + OB + OC).

(4.1)

OA, OB OC ABC
(). ,
2 : 1, (4.1). , .
. O G ABC
,
OA + OB + OC =
0,
,
GA + GB + GC =
0

(4.2)

G ABC.

A
.
. , M M
, AA1
E
ABC, , AM B AM C - B A1
C
D
.
. 27
. A1 BC M (AA1 ). BE CD
AM B AM C (. 27).
BEA1 CDA1 ( ) BE = CD,
AM B AM C. .
35

. AM B AM C . BE = CD. AM BC A1 , A1 BC.
BEA1 CDA1 ( ). AM BC, AM B AM C ,
, .
3 ( ). G AA1 BB1 . AGB
AGC , AGB BGC .
AGC BGC,
, G (CC1 ).
ABC
.
AG : GA1 = SAGC : SA1 GC = 2.
BG : GB1 = 2 CG : GC1 = 2.
, . ABG, BCG CAG G
ABC
, , , .
C
,
P
,
M
A1
B1 , A1 B1 C1 .
Q
G
4 ( ). CC1
B
C1
A
M G : CM =
. 28
= M G = GC1 , AG M P C1 Q,
BC A1 , P Q (. 28).
(. 2.2) BC Q, A1 P
. , A1 BC.
BG AC B1 . P M A1 GC QC1 ABA1 ,
AA1
2QC1
CC1
=
=
= 3,
GA1
2P M
CM

AG : GA1 = 2, BG : GB1 = 2.
36

4.2. . I,
. .
I
? .
ABC AB,
c
BC CA C1 , A1 B1 . BA1 : A1 C =
b
ac
ab
(. 2.4) BA1 + A1 C = a, BA1 =
A1 C =
. CI
b+c
b+c

AA1 C,
AI
AC
ab
b+c
=
=b:
=
.
IA1
A1 C
b+c
a

,
b+c
AI
=
,
IA1
a

BI
c+a
=
,
IB1
b

CI
a+b
=
.
IC1
c

(4.3)

, O
(. 29) OI
OA, OB, OC
C
:

A1

B1
I

C1
. 29

BA1
c
bOB + cOC
= OA1 =
,
b
b+c
A1 C
b+c
OA +
OC
AI
b+c
a
=
OI =
.
b
+
c
a
IA1
1+
a

,
OI =

a OA + b OB + c OC
.
a+b+c

(4.4)

. I
,
a IA + b IB + c IC =
0.

(4.5)

4.3. .
.
. , , ( ).
.
37

1. ABC , .
A, B C A2 B2 C2 (. 30). , ABA2 C
ABCB2 CA2 = AB = CB2 .
A B B2 C2
B2
A2
C
C2 A2 . AA1 , BB1 , CC1 A1
B1
ABC A2 B2 C2 . ,
H
, A
B
C1
,
A2 B2 C2 .
.
C2
. 30

.
2 (). A,
B, C D :
AB CD + BC AD + CA BD = 0,

(4.6)

, : CD = AD AC, BC =
= AC AB, BD = AD AB, CA = AC. , , BC AD = 0
CA BD = 0, (4.6) , AB CD = 0. , BC AD
CA BD ,
AB CD. , D
AA1 BB1 ABC,
CC1 .
C
ABC H, .
B1
A1
H
ABC: A, B, C, H
H
, A
B
C1
.
3. . 31
A1 B1 C1
ABC (. 31). ABC. :
38

CA cos C
CA1
CA
=
=
.
CB1
CB cos C
CB

4A1 B1 C 4ABC, CA1 B1 = BAC , ,


AA1 B1 = 90 A. , AA1 C1 = 90 A. , AA1 B1 A1 C1 . BB1
CC1
C
A1 B1 C1 . , A1
a.
. C1
B . BAC
A
BB1 CC1 ABC B1
H (. 32).
H
BCH (
) CB1 BC1 , A.
. 32
A, . . HA BC. .
. , .
. O , ABC, H ,
OH = OA + OB + OC.

(4.7)

1. ABC
A1 B1 C1 G 1
k = . A A1 H O (
2

1
2

O A1 B1 C1 ). OA1 = HA, 1

(OB + OC) = (OA OH), (4.7).


2
2
2. H O AH BC = 0 OB 2 = OC 2 . :
(OH OA)(OC OB) = 0,

(OC + OB)(OC OB) = 0.

, : (OH OA OB OC)BC = 0. (OH OA OB OC)CA = 0.


BC CA .
,
OH OA OB OC = 0.
4.4. .
O ABC ,
39

(4.1) (4.7) , :
OH = 3OG.
, O, G, H ,
G OH 1 : 2. ,
, (. 33).
O I R r :
OI 2 = R2 2Rr,
(4.8)
. .
P

K
O
M

GH

N
I

B
D

B
. 33

. 34

1. C
ABC D (. 34).
DP IK I AC. IK = r, DP = 2R.
OI M N .
(. 2.5)
CI ID = IM IN = (R + d)(R d) = R2 d2 ,
d = OI. CI ID . P AD CKI (
CI

P C)
=
. AD = ID ( 1.8).
2R
AD
CI ID = 2Rr. , R2 d2 = 2Rr,
(4.8).
2. (4.4),
, O .
40

:
OI 2 =

1
1
(a OA + b OB + c OC)2 = 2 (a2 OA2 + b2 OB 2 + c2 OC 2 +
4p2
4p

+ 2ab(OA OB) + 2ac(OA OC) + 2bc(OB OC)),


OA = OB = OC = R. (3.6) 2OA OB =
= 2R2 c2 , 2OA OC = 2R2 b2 , 2OB OC = 2R2 a2 . ,
, :
OI 2 =

R2 (a + b + c)2 abc(a + b + c)
abc
= R2
.
4p2
2p

, SABC =

abc
abc
= pr,
= 2Rr, 4R
2p

(4.8).
. . , .
. AA1 CC1 ABC, H (. 35). CHA1 AHC1
. :
C

CH
CA1
CA cos C
cos C
=
=
=
.
AH
AC1
CA cos A
cos A

, AH =
=

CH
CH
HC1
,
=
cos B. ,
cos B
AH
HC1

A1

HC1
=
sin HAC1
A

C1

. 35

cos C
CH
cos B =
,
HC1
cos A

CH
cos C
=
.
HC1
cos A cos B

(4.9)

,
.

4.1. , ,
, , - ( 2.16).
41

4.2. , ,
,
.
4.3. H ABC. , , ABH, BCH, CAH, .
4.4. , ,
, . .
4.5. M O
ABC BC. , AOM H (H ).
4.6. ,
O, G, I, H.
4.7. , -
O, G, I, H, .
4.8. A1 B1 AA1 BB1
ABC. , A1 B1 = AB |cos C|.
4.9. , C ABC
H 2R |cos C|.
4.10. , C ABC
H AB |ctg C|.
4.11. ABC:
a2 + AH 2 = b2 + BH 2 = c2 + CH 2 = 4R2 .
4.12. , :
CH
a2 + b2 c2
=
.
R

ab

4.13. , OH 2 = 9R2 (a2 + b2 + c2 ).


1

4.14. , OG2 = R2 (a2 + b2 + c2 ).


9
4.15. G ABC, P
. ,
3P G2 = P A2 + P B 2 + P C 2 (GA2 + GB 2 + GC 2 )
( ).
4.16. M
ABC AB. ,
CM 2 = R2 + a2 + b2 c2 .
4.17. , ABC
AH + BH + CH = 2(R + r).
42

?
?
4.18. , AA1 , BB1 , CC1
ABC, AH HA1 = BH HB1 = CH HC1 .
4.19. ABC AA1 ,
BB1 , CC1 . ,
1
2

AH AA1 + BH BB1 + CH CC1 = (a2 + b2 + c2 ).


?
4.20. 60 . ,
.
4.21. ABC , AB. , ,
(a + b)c

,
.
a+b+c
4.22. ABCD
M , M AB, M BC, M CD, M DA , . .
4.23. ,
. ,
,
,
.
4.24. , ABC
, A, BC sin 2C : sin 2B, B.
4.25. , A1 AA1 ABC
BC tg C : tg B, B.
4.26. , P P O O
ABC :
PO =

P A sin 2A + P B sin 2B + P C sin 2C


.
sin 2A + sin 2B + sin 2C

4.27. , P P H ABC :
PH =

P A tg A + P B tg B + P C tg C
.
tg A + tg B + tg C

43

ABC (4.28
4.36).
4.28. CI 2 = ab 4Rr.
4.29. OH 2 = R2 (1 8 cos A cos B cos C).
4.30. IA IB IC = 4Rr2 .
4.31. GI 2 = (1/9)(p2 + 5r2 16Rr).
4.32. AA1 , BB1 , CC1 ABC,
AA1 + BB 1 + CC 1 = 0.
4.33. AA1 , BB1 , CC1 ABC,
a(b + c)AA1 + b(c + a)BB 1 + c(a + b)CC 1 = 0.
4.34. AA1 , BB1 , CC1 ABC,
a2 AA1 + b2 BB 1 + c2 CC 1 = 0.
4.35. O ,
sin 2A OA + sin 2B OB + sin 2C OC = 0.
4.36. H ,
tg A HA + tg B HB + tg C HC = 0.

5.
5.1. :
, , , , ( ).
. ABC
( ) .
, 360 ,
360 , , 180 .
(
, ),
, 180 (. 36). I1
, AB
BC. A.
I1 r1 , I1 ABC, BC
44

X1 AC AB Y1
Z1 . .
, (. 37).

Y1
r1

r1

I1
r1

X1
A

Z1

. 36

Y1
X2
C
I2

I1

X1

Y2
I

Z2

Z3

Z1
X3

Y3
I3

. 37

5.2. . , . 37. ,
45

, :
AY1 = AZ1 ,
BX1 = BZ1 ,
CX1 = CY1 ,

AY2 = AZ2 ,
BX2 = BZ2 ,
CX2 = CY2 ,

AY3 = AZ3 ,
BX3 = BZ3 ,
CX3 = CY3 .

:
2p = AC + CB + BA = (AC + CX1 ) + (X1 B + BA) =
= (AC + CY1 ) + (BZ1 + BA) = AY1 + AZ1 = 2AY1 .

AY1 = AZ1 = BX2 = BZ2 = CX3 = CY3 = p.

(5.1)

, CX2 = BX2 BC = p a.
CX2 = CY2 = BX3 = BZ3 = p a,
CX1 = CY1 = AY3 = AZ3 = p b,
AZ2 = AY2 = BZ1 = BX1 = p c.

(5.2)

5.3. , .
:
S = r1 (p a) = r2 (p b) = r3 (p c),

(5.3)

r1 , r2 , r3 I1 , I2 ,
I3 .
. 36, ,
1
2

1
2

1
2

1
2

SABC =SABI1 +SACI1 SBCI1 = cr1 + br1 ar1 = r1 (b+ca)=r1 (pa).


S = r2 (p b) = r3 (p c).
(5.3) S = pr :
3p (a + b + c)
1
1
1
pa
pb
pc
p
1
+
+
=
+
+
=
= = .
r1
r2
r3
S
S
S
S
S
r

,
1
1
1
1
=
+
+ .
r
r1
r2
r3

(5.4)

:
4R = r1 + r2 + r3 r.
46

(5.5)


p(p b)(p c) + p(p c)(p a) + p(p a)(p b) (p a)(p b)(p c) = abc.
abc = 4SR,
p(p b)(p c) + p(p c)(p a) + p(p a)(p b) (p a)(p b)(p c) = 4SR.
(5.3) (3.9), :
4SR =

S2
S2
S2
S2
+
+

= S(r1 + r2 + r3 r),
pa
pb
pc
p

4R = r1 + r2 + r3 r.

(5.15.5).
5.1. ABC
I1 I2 I3 (. . 4.3).
5.2. ABC
,
.
5.3. ,
, ,
.
5.4. ,
.
.
5.5.
, .
5.6. , P
P I1 =

bPB + cPC aPA


.
b+ca

(5.75.19).
5.7. OIi2 = R2 + 2Rri , i = 1, 2, 3 ( ).
5.8. r1 r2 = p(p c).
5.9. rr1 = (p b)(p c).
5.10. r1 r2 r3 = pS.
5.11. rr1 r2 r3 = S 2 .
5.12. I1 I2 = 4R cos

C
.
2

5.13. AI AI1 = AI2 AI3 .


47

5.14.
5.15.
5.16.
5.17.
5.18.

AI II1 = 4Rr.
r1 + r2 = 4R cos2 C.
r + r1 + r2 r3 = 4R cos C.
II1 II2 II3 = 4rR2 .
II12 + I2 I32 = II22 + I3 I12 = II32 + I1 I22 .
A

5.19. II1 : II2 : II3 = sin : sin : sin .


2
2
2
5.20.
. , , ,
, .

6.
6.1. .
. ,
, ,
E OH 1
R.
2
1. A1 , B1 , C1 BC,
CA, AB ABC, H1 , H2 , H3
A2 , B2 , C2 AH, BH, CH (. 38). B1 C1 B2 C2 C1 A1 C2 A2
. , B1 C1 B2 C2
ABC HBC BC . B1 C1 B2 C2 .
, B1 C2 k AH, AH BC, B1 C2 B2 C2 . C1 A1 C2 A2 .
C1 C2 . , E. A1 , B1 , C1 , A2 , B2 , C2
E. H1 , H2 , H3 ,
. A1 B1 C1 ABC 1

k = ,
2
R ABC.
A1 B1 C1 A2 B2 C2 E. ,
O H E (O ABC).
48

2. , , H
, (1

4.2). H .
2
, , (. 39).


 ABC (O; R)
1

E; R , .
2
O E, E OH.
C
C
H2
C2

B1

H1

B1

A1
A1

H2

A2

C1

H1
H
B2
H3

B2

A2
A

C2

H3

C1

. 38

. 39



1
E; R 2
. , , .
,
.
ABC A1 B1 C1 H1 H2 H3 .
6.2. . 1804 .
. . , 1765 .
,
.
(18001834), , , 1822
.
49

.
.


1
E; R 2
(I; r). ,

1
EI = R r.
(6.1)
2

(4.7) (4.4) OH OI:


OH = OA + OB + OC,

OI =

1
(aOA + bOB + cOC).
2p

1
2

, OE = OH, :
EI = OI OE =

1
((a p)OA + (b p)OB + (c p)OC).
2p

2OA OB = 2R2 c2 , 2OB OC = 2R2 a2 , 2OA OC = 2R2 b2 ,

EI 2 =

1
((p a)2 R2 + (p b)2 R2 + (p c)2 R2 +
4p2

+(pa)(pb)(2R2 c2 )+(pa)(pc)(2R2 b2 )+(pb)(pc)(2R2 a2 )).


, R2 , p2 R2 .
1
4

EI 2 = R2

1
(c2 (p a)(p b) + b2 (p a)(p c) + a2 (p b)(p c)).
4p2

,
a2 (p b)(p c) + b2 (p a)(p c) + c2 (p a)(p b) = 4p2 r(R r). (6.2)
,
(p a)(p b) =

S2
pr r3 (p c)
=

= rr3
p(p c)
p
pc


(p c)(p a) = rr2 ,

(p b)(p c) = rr1 .

(6.3)

4R = r1 + r2 + r3 r (6.2) :
a2 r1 + b2 r2 + c2 r3 = p2 (r1 + r2 + r3 5r).
50

(6.4)

ar1 = pr1 S,
a2 r1 + b2 r2 + c2 r3 = a(pr1 S) + b(pr2 S) + c(pr3 S) =
= p(ar1 + br2 + cr3 ) (a + b + c)S = p(ar1 + br2 + cr3 2S).

(6.4):
p(pr1 + pr2 + pr3 5pr) =
= p(ar1 + S + br2 + S + cr3 + S 5S) = p(ar1 + br2 + cr3 2S).
, (6.2) .

2
R2
1
EI 2 =
r(R r) =
Rr .
4

, (6.1)
.
. 1
OI 1 =
(bOB + cOC aOA)
2(p a)
1

EI1 = R + r1 .
2
.
[10],
[13], [33], . . .

6.1. , ABC
I1 I2 I3
ABC.
6.2. , .
6.3. ,
.
6.4. ,

.
51

7.
,
,
. B

C ,
, .
,
,
D
A
. 40, ABCD . 40
AB CD . .
.
7.1. . , a
. ,
C
,
- D
(. 41).
O
, , , , A
B
.
. 41
.
. ,
, ,
180 (. . ).
: A C
ABCD (. 41)
BCD BAD, , 180 .
52

. A + C = 180 .
.
, A > 90 . ABD ,
B
C .
E

C : 1) C
A
, 2) F
. A > 90 BC DC D
. 42
E F (. 42). ABED A + BED = 180 .
BED > C A + C < 180 ,
.
A + C > 180 . .
, . 40,
A C BD,
A, B, C, D
:
, A, B, C, D ,
, BAD BCD (
ABC ADC) 180
D
.
7.2. .
, , , C
. J
,
. , B
- A
, . 43
, . .
(. 43).
, ,
, ,
.
.
53

. , , ,
.
, : AB + CD =
= (x + y) + (z + t) = (y + z) + (x + t) = BC + AD (. 44).
, ABCD AB + CD =
= BC + AD. , . , ABC BAD ,
360 , ,
180 . I z C
D t
, z
t
AB, BC AD (. 45). , x
y
CD . x
y
: 1) CD A
B
, 2) CD .
. 44
C1 D1 , - B
CD. C1
C
I
ABC1 D1 : AB + C1 D1 = BC1 + AD1 .
BC1 = BC CC1 , AD1 = AD DD1 ,
AB + C1 D1 = BC CC1 + AD DD1 ,
A
D1 D
C1 D1 + CC1 + DD1 = BC + AD AB.
. 45
AB + CD = BC + AD
BC + AD AB = CD.
,
C1 D1 + CC1 + DD1 = CD. , CC1 D1 D
, .
. .
, B
,
C
, .

K
( ). ,
A
D
. 46
, , ABCD K, C BD (. 46). AB + CD = BC + AD BC = BK, CD = DK.
54

AB + KD = BK + AD ABKD .
7.3. , .
D
D1
.
N
C
P
. BD Q
ABCD
A
B
M B1
B1 D1 (. 47). ,
. 47
AB1 CD1 ,
, :
1) ABCD
,
2) BB1 + DB1 = DD1 + BD1 .
. AB1 CD1 ,
P , M , Q, N (. 47). : AB + CD = AM + M B +
+ DQ CQ AD + BC = AP + P D + BN CN . AM = AP ,
BM = BN , DP = DQ CQ = CN , AB + CD = AD + BC. : BB1 + DB1 = BM M B1 + B1 Q + QD = BM + QD
DD1 + BD1 = DP D1 P + D1 N + N B = DP + N B. BM = BN
DP = DQ, BB1 + DB1 = DD1 + BD1 .
.
1) AB + CD = BC + AD AB > AD. BC > CD AB
AD = BC CD. CB CF = CD AB AE = AD (. 48). BE = BF ( AB AD = BC CD).
ADE, BEF , CDF ,
A, B, C DE, EF , F D. I
DEF AB1 CD1 , , . AB = AD B1 C = CD1 ,
AB1 = AD1 AB1 + CD1 = AD1 + B1 C. , AB1 CD1 .
2) : BB1 + DB1 = DD1 + BD1 , DB1 DD1 = BD1 BB1 ( , ).
55

DE = DD1 BF = BD1 (. 49). EB1 = F B1 , B1 EF ,


DD1 E, BD1 F .
B1 , D, B ED1 F . I
AB1 CD1
.
B

D
C

D1

D1

I
B1

C
I

. 48

B1

. 49

1. A B M AN P BQ ( M P
, N Q ). N
A
, M P N Q
M
(. 50).
. M ABQ ,
ABP
P
Q
B
180 . N = ABP .
. 50
ABP + ABQ = 180 , M + N = 180 ,
M P k N Q.
2. , . , , , , .
.
. 51. P
BDF CDE. BAC + BP C = BAC + BP D +
+ DP C.
AEF , . . 180 , BP D = BF D DP C = AEF
( DEC 180 ).
BAC + BP C = 180 , P
56

ABC. , P AEF .
3. ,
( ).
. ABCD O, M N AC BD.
M N M BD (. 52). . 4.1
F
P
C

B
D
N

. 51

. 52

, O M N , , OBM ODM .
. :
1
2

1
2

SABO + SCDO = (AB + CD)r,

SBCO + SADO = (BC + AD)r,

r . AB + CD = BC + AD. ,
1
2

SABO + SCDO = SBCO + SADO = SABCD .

(7.1)

SABM = SBCM SADM = SCDM ,


1
2

SABM + SCDM = SBCM + SADM = SABCD .

(7.2)

(7.1) (7.2):
SABO SABM = SBCM SCDO ,
SM AO + SM BO = SM CO + SM DO . SM AO =
= SM CO , SM BO = SM DO , .
57


7.1. 4.2, .
7.2. ABCD BAC ACD
BCA CAD 90 . , .
7.3. . ,
.
7.4. ,
, .
7.5. ABCD . , , ABC ACD, .
7.6. .
, .
7.7. , ABC, AB AC D E. ,
DE B C
B C.
7.8. , , . ,
.
7.9. ABC
, AB. , ,
a, b, c .
7.10. , ,
, .
7.11. , , ,
.
7.12. , , , , .
7.13. ABC M . M A, M B,
M C BC, CA, AB
A1 , B1 , C1 . ,
AB1 M C1 , BC1 M A1 , CA1 M B1 , .
58

7.14. ABCD, . A C ,
CD AB BC AD B1 D1 . ABCD , AB1 CD1 . ABCD , AB1 CD1 . .
7.15.
. , , . ,
.
7.16. . ,
.
7.17. .
,
. .
7.18. ABCD . ,
, , .
7.19.
. ,
.
7.20. . , , , .
7.21. , ABCD BC AD
,
:
T B + BP = DP,
T C + AP = AD + CP,
P , T D BC.
7.22. ABCD ,
A

BC AD tg : tg .
2
2
7.23. . ,
a m n ,
59

( m
).
7.24. ,
.

8.

( 7) .
8.1. 1 . , , , ,
, .
, , , .
ABCD .
ABCD
A1 , B1 , C1 D
A1
D
BC, CA, AB (. 53).
C
A1 B1 CD
, A1 CB1 D
B1
CD. AC1 B1 D
A
C1
B AD. :
CDA1 = A1 B1 C ADC1 = AB1 C1 .
, DAC1 = DCA1 ,
. 53

BCD 180 . ADC1 CDA1 ( ADC1
CDA1 ). A1 B1 C AB1 C1 ,
A1 , B1 , C1 , A, B1 , C .
1 (16871768) , . . 1797 .

60

. A1 , B1 , C1 . AD CD.
B1 C1 , A1 B1 .
AB1 C1 A1 B1 C ADC1 CDA1 DAC1 DCA1 . DAB + DCB = 180 , ,
ABCD .
:
, , ,
.
8.2. 1 . ,
, ,
.
. ABCD .
CD,
: A1 B1 = CD sin A1 CB1 = CD sin BCA
(. 53). ABC : AB =
= 2R sin BCA (R ABC).
A1 B1 =

B1 C1 =

BC AD
2R

AB CD
.
2R

C1 A1 =

CA BD
.
2R

(8.1)

A1 , B1 , C1
A1 B1 + B1 C1 = A1 C1 ,

(8.2)

BC AD
CA BD
AB CD
+
=
2R
2R
2R

AB CD + BC AD = CA BD.

(8.3)

, B1
A1 C1 . A,
B, C, D .
, (8.3) , D
, ABC. ,
1 (II . . .) -,
, .

61

D A1 , B1 , C1
BC, CA, AB (8.1), (8.3) :
A1 B1 2R + B1 C1 2R = C1 A1 2R,
A1 B1 + B1 C1 = C1 A1 . , A1 , B1 , C1 . D
ABC.
D
c
C
. AC
e
d
ABCD K
b
f
K
, ADK = BDC (. 54). a
A
B ADK BDC , AD : BD =
= AK : BC. DKC
ABD (DKC = DAB DCK = DBA) :
DC : BD = KC : AB. . 54
AD BC = BD AK AB DC = BD KC,
:
AD BC + AB DC = BD(AK + KC) = BD AC.
,
,
. AB = a, BC = b, CD = c,
DA = d, AC = e, BD = f . ABC ACD
:
cos ABC =

a2 + b2 e2
2ab

cos ADC =

c2 + d2 e2
.
2cd

ABC + ADC = 180 , cos ABC + cos ADC = 0

a2 + b2 e2
c2 + d2 e2
+
= 0,
2ab
2cd

e2 =

cd(a2 + b2 ) + ab(c2 + d2 )
(ac + bd)(ad + bc)
=
.
ab + cd
ab + cd

f2 =

(ab + cd)(ac + bd)


.
ad + bc

(8.4)

,
(ef )2 = (ac + bd)2 ,

ef = ac + bd.

(8.4)
,
e
ad + bc
=
,
f
ab + cd

62

(8.5)

. . , .
1. , , , , , ,
.
. D ABC
A1 B1 C1 (. 55). D k = 2 A1 , B1 ,
C1 A2 , B2 , C2 , D BC, CA, AB. , A2 , B2 C2 .
, H ABC . : P CH , (DP ) (AB) = E, (HE) (DC1 ) = K.
H P AB ( 4.2). DP KH, D K
AB. , K C2 . , HC2 A2 B2 C2 . B2 C2 B1 C1 ,
. , HC2 k B1 C1 .
EHP EDC2 DP H DC2 H . , DP H = DAC = DC1 B1 ( A, C1 , B1 D
). DC2 H = DC1 B1 . , HC2 k B1 C1
B1 C1 k B2 C2 , , H (B2 C2 ).
D
A
A1
B1

C1

C2
K

A2

B2
P

B
. 55

2. , , ,
,
.
63

. 2 7 , ABC, AEF , DCE, BDF , P (. 51).


,
P . ,
P (. 56). .
3. AB. ABCD, .
F
B
H1

D
H2

D
x

H3

x
m

E C
H4

. 56

O a

. 57

. ,
. , AD = BC = x. AB = a, CD = b, AC = BD = m.
: a + b = 2x ( ), x2 + ab = m2 ( ), x2 + m2 = a2 (
). b = 2x a m2 = a2 x2
x2 + ab = m2 , x2 + ax a2 = 0
x. . 2.7, , x
AB = a. (. 18). (A, x)
D (. 57). .

8.1. , .
64

8.2. M , ABC, M A, M B, M C
( ). .
8.3. P CD ,
ABCD, P A(P A + P C) = P B(P B + P D). .
8.4. A ABCD , AB, AC, AD B1 , C1 , D1 . ,
AC AC1 = AB AB1 + AD AD1 .
8.5. A, B, C, D
. ,
1
1
1
=
+
.
AB
AC
AD

8.6.
, ,
. .
8.7. ,
, a,
a+b

b, c. ,
c
.
8.8. ABCD. ,
BD , BD2 = AB BC + AD DC. .
8.9. . ,
,
. .
8.10. P , , OP . .
8.11. P
ABC AQ, Q , P
BC. .
8.12. M N ,
,
M N . .
65

9.
1 .
9.1. . AB, BC, CA,
ABC, C1 , A1 , B1 . ,
AA1 , BB1 , CC1 ,
,
AB 1 CA1 BC 1

= 1.
B1 C A1 B C1 A

(9.1)

. AA1 , BB1 , CC1


P . A ABC , BC (. 58 59). AB1 N
AB 1
AN
=
. AC1 M
B1 C
BC
BC 1
BC
=
. BCC1 ,
C1 A
MA
CA1
MA
( 2)
=
.
A1 B
AN

BB1 C ,

C1
M

C1
P
B

B1

B1

A1

. 58

A1

. 59

. ,
, , (
!), (9.1).
AA1 , BB1 , CC1 P .
1 (16481734) - .
.
1678 .

66

AA1 , BB1 , CC1 (. 60).


( 2)
AB 1
A1 B
=
B1 C
BC

BC 1
BC
=
.
C1 A
CA1

(9.1) .
. (9.1). ,
AA1 , BB1 , CC1 . AA1 BB1 : )
P , ) . , , CC1 P .
(. 61). CP , AB
C2 .
AB 1 CA1 BC 2

= 1.
B1 C A1 B C2 A

(9.2) (9.1)

(9.2)

BC 1
BC 2
=
. , C1 C2
C1 A
C2 A

. , P CC1 .
C1

B1
C1
C2

A1

C
. 60

B1
P
A1

. 61

AA1 BB1 , CC1


, CC1 BB1 , AA1 ,
AA1 k BB1 . .
9.2. () . a1 = BAA1 , a2 =
= A1 AC, g1 = ACC1 , g2 = C1 CB, b1 = CBB1 , b2 = B1 BA (. 62),
(9.1) . ,
AB
AB1
=
sin b2
sin AB1 B

B1 C
BC
=
.
sin b1
sin CB1 B

67

( ) ,
AB sin b2
AB 1
( ):
=
.
BC
sin b1
B1 C

AC sin a2
CA1
=
AB sin a1
A1 B

a1 a2

BC 1
BC sin g2
.
=
AC sin g1
C1 A

C1

B1

b2
B

g1
g2

b1
A1
. 62

AB 1 CA1 BC 1
sin b2 sin a2 sin g2

.
sin b1 sin a1 sin g1
B1 C A1 B C1 A

, , AA1 , BB1 ,
CC1 , ,
sin a1 sin b1 sin g1

= 1.
sin a2 sin b2 sin g2

(9.3)

, AA1 , BB1 , CC1 ABC,


(9.1) , .
, a1 = a2 , b1 = b2 ,
g1 = g2 (9.3). ,
.
9.3. .
AA1 , BB1 , CC1 , ABC, P BC, CA, AB
A1 , B1 , C1 . A2 , B2 , C2 ,
C
A1 , B1 , C1
BC, CA, AB. A1
B2
AA2 , BB2 , CC2
B1 P
Q A2 (. 63). . C
1
C2 E A C1

C2 B
. 63

AB, ,

AC 1
BC 2
=
.
C1 B
C2 A

BA1
CA2
CB 1
AB 2
=

=
. A1 C
A2 B
B1 A
B2 C

(9.1) A1 , B1 , C1 ,
A2 , B2 , C2 , .
P Q

ABC. , ,
ABC.
68

, , AA1 AA2 A ABC, BB1 BB2


B, CC1 CC2 C.
P , M .
(9.3) .
, AA1 AB AC a1 a2 , AA2 AB AC
a2 a1 . (9.3)
, .
P M
ABC. , , ABC.
, O H ABC .
1. ABC BCA1 , CAB1 ABC1 . ,
AA1 , BB1 , CC1 (. 64).
. , . 9.2,
.
(9.3). ,
ABC 120 . ABA1
C1
CAA1 :
AA1
a
=
sin(60 + B)
sin a1

AA1
a
=
,
sin(60 + C)
sin a2

a = BC, B C ABC.

sin(60 + B)
sin a1
=
,
sin a2
sin(60 + C)

sin(60 + C)
sin b1
=
,
sin b2
sin(60 + A)

A
1

sin(60 + A)
sin g1
=
.
sin g2
sin(60 + B)

B1

C
a

A1

, (9.3) . ,
. 64
AA1 , BB1 , CC1
.
ACB = 120 , B, C, B1 , A, C, A1
. C. ACB > 120 ,
69

. AA1 BB1 ABC, 60 + C 360 (60 + C),


.
2. M
M A M B CD EF (. 65). ,
CF DE
AB,
.
.

AB, CF , DE,
D
AEF .
A


C
,
.
F
M
E
:
BE DF CA

= 1.
BF DA CE

M AC M DA, M CE
M F D, M EB M BF :
. 65

AC
MA
=
,
AD
MD

DF
MD
=
,
CE
ME

BE
ME
=
.
BF
MB

M A = M B,
, .
.
3. ABC M N .
AM , BM , CM BC, CA, AB A1 , B1 , C1 . AN , BN , CN B1 C1 , C1 A1 ,
A1 B1 A2 , B2 , C2 (. 66). ,
A1 A2 , B1 B2 , C1 C2 .
. 2 ( 2) ,

. :
BA1 BN 1
C1 M 1 C1 A2
:
=
:
,
A1 C N1 C
M1 B 1 A2 B 1

70


BA1 C1 A2
BN 1 C1 M 1

.
A1 C A2 B 1
N1 C M1 B 1

:
CB 1 A1 B 2
CN 2 A1 M 2

,
B1 A B2 C 1
N2 A M2 C 1
AC 1 B1 C 2
AN 3 B1 M 3
=
.

C1 B C2 A1
N3 B M3 A1

:


BA1 CB 1 AC 1

A1 C B1 A C1 B

 

C1 A2 B1 C 2 A1 B 2

=
A2 B 1 C2 A1 B2 C 1

 

BN 1 CN 2 AN 3
C1 M 1 B1 M 3 A1 M 2
=

.
N1 C

N2 A

N3 B

M1 B 1

M3 A1

M2 C 1

,
(
). ,
, .
A

B1

A2
N3
C1

M1
M

M2

N2

C2

M3

B2
B

A1

N1

. 66

9.1. , ,
, .
71

9.2. ABC BC, CA, AB A1 , B1 , C1 . , AA1 , BB1 ,


CC1 .
9.3. , ,
.
9.4. ABC ,
BC. AB AC C1 B1 . , BB1 CC1
AA1 ABC.
9.5. AA1 ABC M . BM CM CA BA B1 C1 . , BCB1 C1 .
9.6. , , ,
.
9.7. ,

.
9.8. . , , ,
, .
9.9. AP , BP , CP BC, CA, AB ABC A1 , B1 , C1 . A1 B1 C1 , BC,
CA, AB A2 , B2 , C2 . , AA2 , BB2 , CC2
.
9.10. 2 9 , M ,
( ).
9.11. 2 9 , AB, CE,
DF (. 65).
9.12. , AA1 , BB1 , CC1 AB1 CA1 BC1 , ,
AB1 BC1 CA1 = A1 B B1 C C1 A.
.
72

10.

10.1. 1 . BC, CA, AB, ABC, A1 ,
B1 , C1 . , ,
C
, B1
A1
:
K

AB 1 CA1 BC 1

= 1.
B1 C A1 B C1 A
A

C1

. 67
C

g1
B1

a2
a1

g2

b1
b2

. A1 ,
B1 , C1 (. 67). B BK k A1 B1 ,
K (AC). ( 2) :

CB 1 BC 1
KB 1
CA1
=
,
=
.
A1 B
B1 K C1 A
B1 A


(10.1) :

A1

AB 1 CB 1 KB 1

= 1.
B1 C B1 K B1 A

B
C1

(10.1)

. 68

.
(10.1) . , A1 ,
B1 , C1 . AB A1 B1 AB 1 CA1 BC 2

= 1.
B1 C A1 B C2 A
BC 1
BC 2
(10.1) ,
=
. ,
C1 A
C2 A

C2 ,

C1 C2 .

( ), (10.1) :
sin a1 sin b1 sin g1

= 1,
sin a2 sin b2 sin g2

(10.2)

a1 , a2 , b1 , b2 , g1 , g2
. 68. . 9.2 .
1 (III . . .) .
.

73

10.2. . ,
ABC, BC, CA, AB A1 , B1 , C1 , AA1 , BB1 , CC1
.
, M , N , P
C
AA1 , BB1 , CC1 B1
B2 C2 , C2 A2 , A2 B2 ,
A2 B2 C2 A1


P
B2
ABC
(.
2.3) (. 69). A2

N

M
A2 P
BC 1
B2 M
CA1
=
,
=
,
P B2
C1 A
M C2
A1 B
C2 N
AB 1
=
.
N A2
B1 C

C1
A

C2

B
. 69

:
C2 N A2 P B2 M
AB 1 CA1 BC 1

=
B1 C A1 B C1 A
N A2 P B 2 M C 2

AA1 , BB1 , CC1 ,


1.
M , N , P .
.
, AB, BC, CA, A1 B1 , , . ,
, . M N P ,
, .
10.3. 1 . AA1 , BB1 , CC1 ,
ABC A1 B1 C1 , S
, AB A1 B1 , BC
B1 C1 , CA C1 A1 ( ) .
. ,
AA1 , BB1 , CC1 S (. 70). (AB) (A1 B1 ) =
= P , (BC) (B1 C1 ) = Q, (CA) (C1 A1 ) = R.
1 (15911661) ,
. .

74

SAB A1 B1
SA1 AP BB 1

= 1.
A1 A P B B1 S

SBC
SB 1 BQ CC 1

= 1.
B1 B QC C1 S

B1 C1
S


SCA C1 A1 :
SC 1 CR AA1

= 1.
C1 C RA A1 S

A
R1

:

 

AP BQ CR
SA1 AA1

PB

QC

RA

SB 1 BB 1

B1 B B1 S

A1 A

B
Q

B1

A1
C1

A1 S

 

SC 1 CC 1

= 1.
C1 C

. 70

C1 S

1,
AP BQ CR

= 1.
P B QC RA

, P , Q, R .
AA1 , BB1 , CC1 (. 71).
(AB)(A1 B1 )=P , (BC)(B1 C1 )=Q, (P Q)(AC)=R, (P Q)(A1 C1 )=R1 .
, R R1
C
. A
BP Q AC
B

B1

R
A1

BA P R QC

= 1.
AP RQ CB


B1 P Q A1 C1

C1

B1 A1 P R1 QC 1

= 1.
A1 P R1 Q C1 B 1

. 71

BA
B1 A1
QC
QC 1
=

=
(. 2.2), AP
A1 P
CB
C1 B 1
PR
P R1

=
, . . R R1 .
RQ
R1 Q

75

. AA1 R
BB1 Q.
. 70, , 10
,
.
. S , P QR ABC
A1 B1 C1 . : 10 ,
;
10 , . .
10.4. 1 .

,
, , ( ).
(10.2). A, B, C
BC, CA, AB
A1 , B1 , C1 (. 72).
A1
C1
|a1 | = BAA1 = C, |a2 | = A1 AC = A +
a1 B
+ C, A, B, C B1
A
C
a2
ABC. a1 a2 ,
sin a1
sin C
sin C
=
=
.
sin a2
sin(A + C)
sin B

. 72

sin b1
sin A
=
,
sin b2
sin C

sin g1
sin B
=
.
sin g2
sin C

( b1 , b2 , g1 , g2 . 68). ,
(10.2) A1 , B1 , C1 .
10.5. . ( 1 (16231662) , . 16- .
.

76

) (
).
Z

M
X
P
C B1

A1
B

N
C1

. 73

. AB1 CA1 BC1 (. 73). , M = (AB1 ) (A1 B),


P = (BC1 ) (B1 C), N = (CA1 ) (C1 A) . X = (AB1 ) (CA1 ), Y = (BC1 ) (CA1 ), Z = (AB1 ) (BC1 ),
.
XA XB 1 = XC XA1 ,
Y B Y C 1 = Y C Y A1 ,

(10.3)

ZB ZC 1 = ZA ZB 1 .

XY Z AC1 , CB1 , BA1 :
XA ZC 1 Y N
= 1,

AZ C1 Y N X

XB 1 ZP Y C
= 1,

B1 Z P Y CX

XM ZB Y A1
= 1.

M Z BY A1 X

. (10.3) .
:
XM ZP Y N

= 1,
MZ P Y NX

M , P , N .
77

1. M N M P N Q , . ,
, P
Q , M N S
M
P
L, (. 74)
MP
ML
=
.
LN
NQ
Q
L

(10.4)

. (M P ) (N Q) = S.

SM N P QL :

SP M L N Q

= 1.
P M LN QS

. 74

SP = SQ, (10.4)
2. ,
(
) (. 75).
. ABCD ,
A C M ,
B D N (AB) (M N ) = P .
MP
MA
=
. (CD) (M N ) = Q,
PN
NB
MC
MQ
=
. M C = M A N D = N B,

NQ
ND

A
B

K
Q P

. 75

MP
MQ
=
, , P Q . , ,
PN
QN

P AB CD
78

M N , .
BC AD.

10.1. , , ,
.
.
10.2. ,
, , .
10.3. BC, CA, AB, ABC, A1 , B1 , C1 . A2 ,
B2 , C2 A1 , B1 , C1
BC, CA, AB. , A2 , B2 , C2
( A1 B1 C1 A2 B2 C2
ABC).
10.4. ( 8), .
10.5. , , ,
,
.
10.6. , , ,
. .
10.7. (4.3) (4.9), .
10.8. , AB1 CA1 BC1 AC A1 B1 , BB1 C1 C1 , AB A1 C1
.
10.9. A, B, C A1 ,
B1 , C1 , M = (AB1 ) (A1 B),
N = (AC1 ) (A1 C), P = (BC1 ) (B1 C), M , N , P
( ). .
10.10. , , P , ()
.
10.11. . , ,
. .
79

10.12. ABCD AA1 CC1 ,


A C A1 C1
BC AB, BD.
10.13. . P
Q. , P Q ,
, P
Q .
10.14. . ( ),
, .

11.

11.1. . ( ), , .
. , ,
.
1. M N P Q ABCD, E F AC BD
(. 76). M P
N Q AC
C

. ,
P

M P N Q B
.

G
E
N
M EN F P EQF M
. F

A
D . Q
.
. 76
2.
1
O . OM = (OA + OB)
2

1
2

ON = (OC + OD). G M N .
1
2

1
4

OG = (OM + ON ) = (OA + OB + OC + OD).


80

. G P Q EF .
. .
G :
1
4

OG = (OA + OB + OC + OD).

(11.1)

O G :
GA + GB + GC + GD = 0.
. ,
, , 3 : 1, .
, G1 BCD,
O (4.1)
1
3

OG1 = (OB + OC + OD).


AG1 3 : 1.
OA + 3OG1
1
= (OA + OB + OC + OD) = OG,
1+3
4

.
11.2. .
AB
CD. (3.6), :
2AB CD = 2AB(AD AC) =
= 2AB AD 2AB AC = AB 2 + AD2 BD2 AB 2 AC 2 + BC 2 .
,
2AB CD = AD2 + BC 2 BD2 AC 2 .

(11.2)

:
AB CD AD2 + BC 2 = AC 2 + BD2 ,
AC BD AD2 + BC 2 = AB 2 + CD2 .

(11.3)

,
,
81

.
,
.
: AB = a, BC = b, CD = c, DA = d,
AC = e, BD = f . M N : M N = M A +
+ AD + DN M N = M B + BC + CN . ,
M A + M B = 0 DN + CN = 0 (. 77), D
:
c N
2M N = AD + BC.
(11.4)
C
d
e

2P Q = BA + CD 2EF = AD +
+ CB.
, (11.2):

M
A

. 77

4M N 2 = AD2 + BC 2 + 2AD BC =
= AD2 + BC 2 + AC 2 + BD2 AB 2 DC 2 .

1
4

, M N 2 = (b2 + d2 + e2 + f 2 a2 c2 ).
1
4
1
2
EF = (a2 + b2 + c2 + d2 e2 f 2 ).
4

P Q2 = (a2 + c2 + e2 + f 2 b2 d2 ),

(11.5)

.
.
1) ABCD , E
F . ,
a2 + b2 + c2 + d2 = e2 + f 2 ,

(11.6)

. . .
2) ABCD AB
CD, (11.2) : 2ac = d2 + b2 e2 f 2 , e2 + f 2 =
= b2 + d2 + 2ac
1
4
1
EF 2 = (a2 + c2 2ac),
4

P Q2 = (a2 + c2 + 2ac),

1
2
1
EF = |a c|.
2

P Q = (a + c),

3) . , C D
82

, N CD (. 78). M N ABC, P Q EF
ABC. e = d, b = f , c = 0
(11.5)
D C

Q

1
4

M N 2 = (2b2 + 2d2 a2 ),

P
E

:
A

1
2

P Q = EF = a.

(11.7)

. 78

(11.5) :
4)
1
4

M N 2 + P Q2 + EF 2 = (a2 + b2 + c2 + d2 + e2 + f 2 ).

(11.8)

5) ,
.
6) ,
, .
11.3. . , , :
a2c2(b2 + d2 + e2 + f 2 a2 c2) + b2d2(a2 + c2 + e2 + f 2 b2 d2) +
+ e2f 2(a2 + c2 + b2 + d2 e2 f 2) = (abe)2 + (bcf )2 + (cde)2 + (daf )2. (11.9)
.
(11.5) (11.9) :
(2AB CD M N )2 + (2BC AD P Q)2 + (2CA BD EF )2 =
= (AB BC CA)2 + (BC CD DB)2 + (CD DA AC)2 + (DA AB BD)2 .
(11.9) :


0 a e d 1
a 0 b f 1


e b 0 c 1 = 0.
d f c 0 1
1 1 1 1 0
83

(11.10)

11.4. . AD = AB + BC + CD. :
AD2 = AB 2 + BC 2 + CD2 + 2AB BC + 2AB CD + 2BC CD.
, :
d2 = a2 + b2 + c2 2ab cos B 2ac cos w 2bc cos C,

(11.11)

w AB CD (. 79), B C ABCD B C.
,

B w

C
B
, A
D
, .
B
, C
,
C
w
,
w
A
D ,
D A
. 79
(. 79). w
p (A + D).
(11.11) :
d2 = a2 + b2 + c2 2ab cos B 2bc cos C + 2ac cos(A + D).

(11.12)

(11.5). , M E M F
EM F (. 76) DC AB, EM F = w. e2 = a2 + b2 2ab cos B f 2 = b2 + c2 2bc cos C,

1
4

EF 2 = (a2 + b2 + c2 + d2 a2 b2 + 2ab cos B b2 c2 + 2bc cos C) =


1
4

= (d2 b2 + 2ab cos B + 2bc cos C).


 2
c
+
, EF 2 = M E 2 + M F 2 2M E M F cos w =
2
 2
a
c a
+
2 cos w.
2
2 2
EF 2 , (11.11).
84

11.5. . ABCD:
(ef )2 = (ac)2 + (bd)2 2abcd cos(A + C).

(11.13)

1. .
ABCD ABF
ADE, CAD CAB (. 80).
:
C

AF
c
= ,
a
e

BF
d
AE
b
DE
a
= ,
= ,
= ,
a
e
d
e
d
e
bd
ad
ac
AF = , AE = , BF = DE = .
e
e
e

d
E

A
. 80

B
a

D
e


(11.13).
2. AD
ABCD ADC1 , ABC: DAC1 = BAC AC1 D = BCA
(. 81).

B D BDEF ABD, . . 180 . , BF k DE. , , BF = DE,


BDEF
F E = BD = f . AEF EAF
A + C . :
 2   2
ac
bd
2abcd
f2 =
+
2 cos(A + C),
e

C1
. 81

d
, a

AC1 = e, DC1 = b. CC1 AD


a
a
AD, D, AD. CDC1 B + D, 180 ,
360 (B + D).
ACC1 DCC1 (CAC1 = BAD):
 2
 2
de2
bd
bcd
de
2
cos A, CC12 = c2 +
2
cos(B + D).
CC12 = e2 +
a

,
a2 + d2 2ad cos A = f 2 .
85

(11.13), cos(B + D) = cos(A + C).


bd

C, D, C1 , cos(B + D) = 1, CC1 = c +

a
.
11.6. . A, B,
C, D , , cos(B + D) = 1,
, ABCD . (11.13) :
(ef )2 = (ac)2 + (bd)2 + 2abcd,
ef = ac + bd. , ( 8).
(11.13) cos(A + C) =
= cos(B + D) = 1. ,
(ef )2 6 (ac)2 + (bd)2 + 2abcd,

ef 6 ac + bd.

(11.14)

A, B, C, D ,
. , A, B, C, D . ,
A = 0 C = 180 ,
cos(A + C) = 1. A
D
C
B
a)
, A C
B D (. 82).
)
D A
B
C
, .

. 82
:
B
a

AC BD 6 AB CD + BC AD,


,
C
D
, (A, C)
. 83
(B, D).
ABCD, D
AC (. 83). D = 180 , c + d = e
A

(ef )2 6 (ac)2 + (bd)2 2abcd cos(180 + B).


86

a2 + b2 e2

, (ef )2 = (ac)2 + (bd)2 +


cos(180 + B) = cos B =
2ab
2
2
2
2
2
2
+ cd(a + b e ) = ca (c + d) + db (c + d) cde = ca2 e + db2 e cde2 . ,
ef 2 = ca2 + db2 cde.

(11.15)

.
, BD ABC,
1
4

c
d
1
= = ,
e
e
2

cd = e2 (11.15) :
1
4

f 2 = (2a2 + 2b2 e2 ).
BD ABC,

d
a
= .
c
b

1
1
d
(ca2 + db2 ) = ((e d)a2 + db2 ) = a2 + (b2 a2 ).
e
e
e
ac

e = c + d = c + =
b
:

(a + b)c
1
, (ca2 + db2 ) = ab (11.15) b
e

f 2 = ab cd
(. 2, 1).

11.1. ABCD. , BCD, CDA, DAB, ABC


ABCD.
.
11.2. , , .
11.3. , (3.3) .
11.4. (11.11),
.
11.5. . , .
11.6. BC ABC BCD. ,

1
AD2 = (a2 + b2 + c2 ) + 2 3S,
2

S ABC.
87

11.7. ABC A1 , B1 , C1 . , AA1 = B1 C1 , BB1 = C1 A1 ,


CC1 = A1 B1 .
11.8.
. , , , .
11.9. ,
.
11.10. ,
, .
11.11. 90 270 ,
. .
11.12. , 45
.
11.13. (11.15) , .
11.14. , C ABC D
:
CD2 =

1 2 2
(a m + b2 n2 ),
c2

a b , m n .
11.15. , A, B, C, D
(A, C) (B, D),
AC BD = AB CD + BC DA.

12.
12.1. . e f AC BD ABCD f
. A C
BD, B D AC (. 84).
M N P Q, , . ,

88

, . ,
1
2

SABCD = ef sin f.

(12.1)
Q

. 84

, , .
. (11.2)
2AC BD = AD2 + BC 2 AB 2 CD2 ,

|cos f| =

|b2 + d2 a2 c2 |
.
2ef

(12.2)

4S 2 = (ef )2 sin2 f = (ef )2 (1 cos2 f) =




(b2 + d2 a2 c2 )2
1
= (4e2 f 2 (b2 + d2 a2 c2 )2 ).
= (ef )2 1
2
4(ef )

,
16S 2 = 4e2 f 2 (b2 + d2 a2 c2 )2

(12.3)

16S 2 = (2ef + b2 + d2 a2 c2 )(2ef + a2 + c2 b2 d2 ).

(12.4)

S ABCD:
S 2 = (p a)(p b)(p c)(p d) abcd cos2
89

A+C
,
2

(12.5)

p = (a + b + c + d) .
2
(12.1):
4S 2 = (ef )2 (ef )2 cos2 f.
(ef )2 (11.13),
(12.2). :
1
4

4S 2 = (ac)2 + (bd)2 2abcd cos(A + C) (a2 + c2 b2 d2 )2 .


(ac)2 + (bd)2 = (ac + bd)2 2abcd,
16S 2 = 4(ac + bd)2 (a2 + c2 b2 d2 )2 8abcd(1 + cos(A + C)) =
A+C
=
2
A+C
.
=(a+b+cd)(a+bc+d)(ab+c+d)(a+b+c+d)16abcd cos2
2

=(2ac+2bd+a2 +c2 b2 d2 )(2ac+2bda2 c2 +b2 +d2 )16abcd cos2

a + b + c d : a + b + c d = (a + b + c + d) 2d = 2p 2d =
= 2(p d). a + b c + d = 2(p c), a b + c + d = 2(p b),
a + b + c + d = 2(p a).
16S 2 16 (12.5).
12.2. . .
1. ABCD ,
sin f = 1 (12.1)
1
2

S = ef.

(12.6)

2. ABCD , A +
A+C

+ C = B + D = 180 cos
= 0. (12.5) 2
:
S 2 = (p a)(p b)(p c)(p d).
(12.7)

VII
. (12.4),
ef = ac + bd.
3. ABCD A D
. d = 0 (12.7)
ABC:
S 2 = (p a)(p b)(p c)p.
90

(12.8)

4. (12.5) , S 2 ,
cos

A+C
= 0, . . A + C = 180 .
2

, , . ( 7.24).
5. ABCD ,
a + c = b + d b2 + d2 a2 c2 = 2(bd ac). (12.4)

4S 2 = (ef + bd ac)(ef bd + ac).

(12.9)

6. , (12.7) (12.9) :
S 2 = abcd.

(12.10)

1. ABCD AB CD K. E F
AC BD. , EF K
ABCD.
. AB CD BC
(. 85), P BC EF K.
, P K
K
EF .
, EP k AB,
P B EK ( BEP K ).
C
F P k CD, C P
P
F K.
B
E
P
EF K
F
A

SEF K = SEF P + SEP K + SF P K .

D
. 85

EP K EP B
F P K F P C , EF K EF CB.

1
ABCD. (12.1) SEF CB = SABCD , ,
4

1
4

SEF K = SABCD .
91

. SEF L = SABCD (L 4
AD BC). EF K EF L , 4 EF T LK
(. 86). : ,
, .
( 10).
2. , .
. , .
, .
. A1 , B1 , C1 , D1 , AB, BC, CD, DA
ABCD (. 87).
. 3.1
A1 B1 =

SABC
,
R

C1 D1 =

SACD
,
R

R . A1 B1 + C1 D1 =
S

S
,
R

S ABCD. B1 C1 + A1 D1 = . R
A1 B1 + C1 D1 = B1 C1 + A1 D1 , A1 B1 C1 D1
r. S1
1
1
(A1 B1 + B1 C1 + C1 D1 + D1 A1 )r. (A1 B1 + B1 C1 + C1 D1 +
2

2
2S
S : S1 = R : r.
R

+ D1 A1 )r = S1 A1 B1 + B1 C1 + C1 D1 + D1 A1 =
K

B
A1

B1

A
E

F
C1
D1

. 86

D
. 87

92


12.1. , .
12.2. . ,
.
12.3. S
, . .
12.4.
. , .
12.5.
, . .
12.6. () ABCD
4S 2 = AC 2 BD2 (AC BD)2 .
12.7. O ABCD,
. , ABCO AOCD .
12.8. , E F AC
BD ABCD, AB CD K
L. , ABL CDK .
12.9. .
, ,
, .
12.10. M N AB CD ABCD M C, M D, AN , BN .
, , , ,
AD BC.
12.11. , BD ABCD AC, AD
T . , CT ABCD
.
12.12. . .
93

,
.
12.13. ,
:
1
4

4S 2 = (ab)2 + (cd)2 (a2 + b2 c2 d2 )2 2abcd cos(B + D).


12.14. ABCD R. ,
16S 2 R2 = (ab + cd)(bc + ad)(ac + bd).
12.15. , ,
, , .
?
12.16. , , , , .
12.17. E F AC BD ABCD E
C F D, CDEF , ADEF CBEF , ABED CBED . .
12.18.
2S 6 ab + cd,

2S 6 ac + bd.

?
?

13.

, - .
.
13.1. . ,
A, B C :
AB 6 AC + CB,
94

(13.1)

,
, C A B.
AB < AC + CB ,
A, B, C .
,
.
(13.1) ()
.
1. C
D
ABCD (. 88):
O

AC + BD > AB + CD,
AC + BD > BC + AD.

(13.2)
B

, (AC) (BD) = O, AB <


< AO + OB CD < CO + OD. , :

A
. 88

AB + CD < (AO + OC) + (BO + OD) = AC + BD.


2. O, ABC, :
1
2

OA + OB + OC > (AB + BC + CA).

(13.3)

, AB < AO + OB, BC < BO + OC, CA < CO +


+ OA, AB + BC + CA < 2(OA + OB + OC), (13.3).
3. ABC
C
mc CM :
b
a
1
1
(a + b c) < mc < (a + b).
2
2

M
a
D
. 89

(13.4)


CM M (. 89). : CD < AC + AD, . . 2mc < b + a,
1
mc < (a + b). , BC < CM + BM
2

CA < CM + AM , a < mc +
1

c
c
b < mc + .
2
2

a + b < 2mc + c, , (a + b c) < mc .


2
4. ABC
:
sin A + sin B > sin C.
95

(13.5)

, a = 2R sin A, b = 2R sin B,
c = 2R sin C. a + b > c
(13.5).
sin C = sin(A + B), (13.5) :
sin A + sin B > sin(A + B).

(13.6)

, sin 1 + sin 2 > sin 177 = sin 3 .


A+B

B+C

C +A

+
+
= 180 ,

2
2
2
, (13.5) :
sin

A+B
B+C
C +A
+ sin
> sin
.
2
2
2

(13.7)

:
1) , ;
2)
, ;
3)
, , ,
.
5. , ,
.
C
, M AB ABC AC > CB (. 90).
, CM < AC. (1)
AC > CB B > A. (2)
A
B AM C > B. AM C > A, M
AC > CM .
. 90
6. AC ABCD BD AB > BC, AD > DC.
AO = OC (. 88). OA OB AOB
OC OB BOC, (AB > BC),
: AOB > BOC, COD > AOD. AOD COD
, CD > AD.
13.2. . , 96

( ). .
1. (4.8) OI 2 = R2 2Rr.
2
OI > 0, R2 2Rr > 0,
R > 2r,

(13.8)

O I, . .
. , .
2. (3.15)
ABC:
cos A + cos B + cos C = 1 +

r
.
R

0 < r/R 6 1/2 (13.8),


:
3
1 < cos A + cos B + cos C 6 .
(13.9)
2

3. ABC
(. 4.13):
OH 2 = 9R2 (a2 + b2 + c2 ).
OH 2 > 0, 9R2 (a2 + b2 + c2 ) > 0,
1
9

R2 > (a2 + b2 + c2 ),

(13.10)

.
4. (. 3.22)

1
CC12 = (a2 + b2 + c2 ) 2S 3,
2

a2 + b2 + c2 > 4 3S

(13.11)

. C C1 , . .
ABC.
5. (11.5)
e2 + f 2 6 a2 + b2 + c2 + d2

(13.12)

,
.
97

6.
:
d2 < 3(a2 + b2 + c2 ).

(13.13)

d = a
+ b + c
(. 91) :
d2 = a2 + b2 + c2 + 2
ab + 2
ac + 2b
c.

B
b
a

2
ab = a2 + b2 (
a b)2 , 2
ac =
2
2
2
2
2

= a + c (
a c) , 2b
c = b + c (b c)2 , :

C
c

d2 = 3(a2 + b2 + c2 ) (
a b)2 (
a c)2 (b c)2 .
D

(
a b)2 > 0, (
a c)2 > 0, (b c)2 > 0,

(13.13).
13.3. .
1 6 sin x 6 1 1 6 cos x 6 1 .
1
1. (12.1) S = ef sin f 2
sin f
. 91

2S 6 ef.

(13.14)

, sin f = 1, . .
.
2. cos C =
=

a2 + b2 c2
. ,
2ab

1 <

a2 + b2 c2
< 1.
2ab

c2 < (a + b)2 ,
c < a + b. (a b)2 < c2 ,
|a b| < c, a < b + c, b < a + c. :
.
3.
1
6

S < (ab + bc + ca).


98

(13.15)

1
2

1
2

1
2

, S = ab sin C, S 6 ab S 6 bc,
1

S 6 ca,
2
. (13.15).
4. S ABC
(. 3.13)
1
4

S = (a2 sin 2B + b2 sin 2A),

1
4

S 6 (a2 + b2 ).

(13.16)

,
A B 45 .
13.4. . a
b = ab cos f
:
a
2 = a2 > 0 a
b 6 ab.
(13.17)
.
a
= 0,
. .
1. ,
:
C
3
2

cos A + cos B + cos C 6 .

(13.18)

e3
e2

e1 , e2 , e3 , e1
A
B
AB, BC,
CA (. 92). (
e1 + e2 + e3 )2 > 0 . 92
(13.18). ,
,
e21 = e22 = e23 = 1 e1 e2 = cos B, e1 e3 = cos A, e2 e3 = cos C,
3 2(cos A + cos B + cos C) > 0,
(13.18). ,
e1 + e2 + e3 = 0, . . ABC .
2. ABC
3
2

cos 2A + cos 2B + cos 2C > .


99

(13.19)

e1 , e2 , e3 O ABC (. 93), OA, OB, OC. e2


e3 A, (
e1 + e2 + e3 )2 > 0 3 + 2(cos 2A + cos 2B + cos 2C) > 0, (13.19). .
3. (11.2)

C
A, B, C, D:
e3
O
e1

2AC BD = AD2 + BC 2 AB 2 CD2 6 2AC BD,


2A
e2

AC BD. ,
( 11, . 77)
2ef > b2 + d2 a2 c2 .

. 93

(13.20)

ABDC
e f , A, B, C,
D AC  BD.
,
.
2AC BD = |AD2 + BC 2 AB 2 CD2 | AC BD.
13.5.
. a b
a b :

a+b
2

ab

a2 + b2
2

2
2ab
=
1/a + 1/b
a+b

:
r

2ab
a+b
a2 + b2
6 ab 6
6
,
a+b

(13.21)

a = b.
100

. ,

a+b
.
( a b)2 > 0 ab 6
2

,
a+b
6
2

(a + b)2
a2 + b2
a2 + b2

6
(a b)2 6 0.
2
4
2

. ,

ab(a b)2
2ab
4a2 b2
6 ab
6 ab
6 0,
2
a+b
(a + b)
(a + b)2
.
(13.21) n > 2
. n = 3 :
r

3
a+b+c
a2 + b2 + c2
3
6 abc 6
6
.
(13.22)
1/a + 1/b + 1/c


.
1. ABC


1
1 a+b 2
1 a2 + b2
1
6
.
S = ab sin C 6 ab 6
2

,
1
8

1
4

S 6 (a + b)2 6 (a2 + b2 ).

(13.23)

a = b sin C = 1, . . .
2. (11.14) ef 6 ac + bd
, .
,
, .
1
2

ac 6 (a2 + c2 )

1
2

1
2

bd 6 (b2 + d2 ),

ef 6 (a2 + b2 + c2 + d2 ),

(13.24)

. . , .
101

3. , :
1
8

cos A cos B cos C 6 .

(13.25)

,
(13.22)

1
3
cos A cos B cos C 6 (cos A + cos B + cos C).
3

(13.18) :

1
3
cos A cos B cos C 6 ,
2

(13.25). ( ), (13.25) .
13.6. .
,
. ,
.
1. cos 2a = 2 cos2 a 1,
(13.19) :
3
4

(13.26)

9
4

(13.27)

cos2 A + cos2 B + cos2 C > ,


:
sin2 A + sin2 B + sin2 C 6 .
2. (13.10) (13.11) :

4 3S 6 a2 + b2 + c2 6 9R2

S6
S =

3 3 2
R .
4

(13.28)

abc
, :
4R

abc 6 3 3R3 .

(13.29)

S = 2R sin A sin B sin C ,


(3.28) :
sin A sin B sin C 6
102

3 3
.
8

(13.30)

3. a, b, c ABC. (13.22)

1
2
8
3
abc 6 (a + b + c) = p, abc 6 p3 .
3

27

S
abc = 4SR p = ,
r

4R 6

8S 2
.
27r3

R > 2r, . (13.8), :

S > 27r2 .

(13.31)

2
27 2
R .
27r 6 S 6
8

.
13.7. ,
. , ,
.
( ) ,
.
1.
A1
B
AB1 CA1 BC1 . , - C
N
1
C
P
ABC .
M
. A, B, C ,
A
B1
. 94
(. 94). , ABC S .
, M N P ( . 94),
S = (SAB1 CP + SCA1 BN + SBC1 AM ) + SM N P .
,
1
2

SABC = (SAB1 CP + SCA1 BN + SBC1 AM ) + SM N P ,


103


SAB1 CP + SCA1 BN + SBC1 AM = 2SABC 2SM N P .
1
2

1
2

, S = 2SABC SM N P , SABC = (S + SM N P ) > S. 1

, : SABC > S.
2
2. M N BC CD
ABCD. , SABCD < 4SAM N .
. AM N , .
A1 M1 N1 (. 95), AM N . SABCD SA1 M1 N1 .
M A1 C M N1 B
M , N A1 C N M1 D
N , SM A1 C = SM N1 B SN A1 C = SN M1 D . M1 N1 k BD
ABCD , M1 N1 . : 1) C
M A1 N (
M A1 N A1 ), 2) C M1 AN (. 96).
A1 M1 N1
BCDM1 N1 , ABCD ( SABN1 + SADM1 ).
B AM N1 . SBM N1 = SCA1 M
SABN1 = SADM1 (AN1 = AM1 M1 N k BD), SA1 M1 N1 > SABCD (
SCA1 N + SADM1 ).
A1
A1
B

M
C

N1

N
N1
A

D
A
M1

D
M1

. 95

. 96

ABCD, .
, C
104

A1 M N B N1 AM .
. E AC. ABCD O
(. 97).
M N BCD,
C
T AC N
T f
OC ( ). , D
1

AC > OC AC > OC EC > T C.


2
2
AT + T C = AE + EC, AT > AE. S ABCD
AM N :
1
2

S = AC BD sin f = AE BD sin f,

f = BOC;

1
2

4SAM N = 4 M N AT sin f = BD AT sin f;

O f
B
. 97

AT > AE 4SAM N > S.

(13.1
13.9).
1
2

13.1. h 6 c.

13.2. c < a + b 6 c 2.
13.3. c + h > a + b.
1
c
1
.
13.4. r < a, r < c, r 6
2
4
2(1 + 2)

r
13.5. h 6 (1 + 2)r, 0,4 < < 0,5.
h

13.6. R > S, R + r > 2S.


2
13.7. ab + bc + ca < 2c
.
13.8.

m2a
n

3 8
r2
6
.
+ m2b
5

13.9. a + bn < cn n > 2.


(13.10
13.12).
13.10. a + b + c > 4R.
13.11. ma + mb + mc > 4R.
ma
ma
R
ma
+
+
61+ .
13.12.
ha

ha

ha

(13.13
13.35).
13.13. a2 + b2 + c2 < 2(ab + bc + ca).
105

13.14. a(b c)2 + b(c a)2 + c(a b)2 + 4abc > a3 + b3 + c3 .


13.15. (a + b c)(a b + c)(a + b + c) 6 abc.
13.16. a2 b(a b) + b2 c(b c) + c2 a(c a) > 0.
13.17.
13.18.
13.19.
13.20.

a2 + b2 > c2 .
2
ha + hb + hc < 2p.
ha + hb + hc> 9r.
a + b + c 6 27R.

9
2
27
13.22. m2a + m2b + m2c 6 R.
4
9
13.23. m2a + m2b > c2 .
8

13.21. ma + mb + mc 6 R.

13.24. la + lb + lc 6 3p.
13.25. la2 + lb2 + lc2 6 p2 .
13.26. sin

A
a
6 .
2
2 bc

C
.
2
A
B
C
1
13.28. sin sin sin 6 .
2
2
2
8

13.27. c > (a + b) sin

3r
.
R

3 3
sin A + sin B + sin C 6
.
2
A
B
C
3
sin + sin + sin 6 .
2
2
2
2
A
B
C
3 3
cos + cos + cos 6
.
2
2
2
2
1
1
1
1
+ 2 + 2 6 2.
a2
b c
R

13.29. cos A + cos B + cos C >


13.30.
13.31.
13.32.
13.33.

13.34. p2 > 3 3S.

13.35. a2 + b2 + c2 (a b)2 (b c)2 (c a)2 > 4 3S.


13.36. D BC ABC. , , ABC, , ABD
ACD.
13.37. ABC ABD
C D. , |AC CB| < |AD DB|, )
ABC ABD, )
ABC ABD.
13.38. ,
4Rr.
106

13.39. , ,
.
13.40. b d , e f
, |e f | > |b d| e + f > b + d.
13.41. ABCD . ,



1
1 1
1
1
1
1
+
6
+
+
+
.
2
2
2
2
2
2
e

4 a

13.42. ABC A1 , B1 , C1 . ,
SABC 6 SA1 B1 C1 .
13.43. ABC. AB, BC, CA
B, C, A BB1 , CC1 , AA1 , BB1 = AC, CC1 = AB, AA1 = BC. ,
SABA1 + SBCB1 + SCAC1 > 3SABC .
13.44. ABC A1 , B1 , C1 . , A1 B1 C1 ABC.

14.
() () ,

, ().
x > a (x 6 a),
, , . (13.18)
3
2

cos A + cos B + cos C 6 . 3

. .
2
. r
.


S > 27r2 (. 13.6). 27r2 S.
14.1. . .
107

(13.21) (13.22)
.
, .
: 1) , , 2) ,
. , .
1 . ,

.
2 . ,
.
.
1. , .
.
S 2 = p(p a)(p b)(p c),
p . (p a) + (p b) + (p c) =
= 3p 2p = p, 2 S 2 p a =
= p b = p c, . . a = b = c .
2. , ,
, ,
.
. , K
M , Q
O P Q (. 98).
y
P Q , B
. OA = a, OB = b, AP = x, BQ = y.
M
b
OP + OQ = (a + b) + (x + y). a
x
a + b , O
A P
,
. 98
x + y. , . , xy = ab = const. 1
x + y x = y. x2 = ab OP + OQ = a + b + 2 ab = ( a + b)2

x = ab AP = BQ , P Q. .
108

14.2. .
12 (. 12.2), (12.5) , ,
.
.
1. , a a
.
. R,
2p = a + b + c = a + 2R(sin B + sin C) =
= a + 4R sin
a 4R sin

pa

BC
pa
BC
B+C
cos
= a + 4R sin
cos
.
2
2
2
2

, 2p -

2
BC
BC
, . . cos
= 1, B = C,
cos
2
2

, .
2.
D
C
a,
f
.
M
. ,
1 2
, S = d sin f, d 2
, f .
a
d = 2R sin a (R - A
B
). S = 2R2 sin2 a sin f.
R a , S . 99
sin f = 1, . . (. 99).
M AB = M BA = 45 . , a 6 45 .
. a. , , . 45
.
.
14.3. . ,
.
109

. AB = a, BC = b, CD = c ABCD. , .
?
. : S = pr, p = a + c = b + d. p ,
S r .
: ,
(p a)(p b)(p c)(p d). , a + c = b + d = p,
S 2 = abcd d = a b + c. , r
r=

S
=
p

p
abc(a b + c)
.
a+c

14.4. . ( , , )
,
N
.
C
1. C1

A. ABC
A
, B C
.
O
B1 B
. M N ,
A M
. M N . 100
B
C (. 100). , B1
A
M
a
C1 - , AB1 + B1 C1 + C1 A = M B1 +
r
A1
b
+ B1 C1 + C1 N > M N = M B + BC + CN =
N
= AB + BC + CA. , ABC B
.
, . 101
.
2. A B.
, A B ?
. a b (. 101) Tr ,
a b. A A1 M
N = b (BA1 ), A B AM N B 110

, M N ,
AM + N B = A1 N + N B .
3. ABC P , ( ).
. B 60 .
A A1 , P P1 (. 102). BP P1 , BP = P P1 P A + P B + P C = P1 A1 + P1 P + P C.
, A1 , P1 , P , C
. BP C = 120 AP B = A1 P1 B = 120 .
AP C = 120 . , P , ABC 120 . . AP BA1
. ABC CA1
, ABA1 .
A
A1
P1
P

B
. 102

ABC .
, ,
(. 64).
ABC : , 120 .
A = 120 , P A.
14.5. . y = ax2 + bx + c,

a > 0, a < 0, x = b/2.
b2 4ac > 0, x
.
111

.
1. AB, BC, CD, DA ABCD
A1 , B1 , C1 , D1 , AA1 = BB1 = CC1 = DD1
(. 103). AA1 : AB
A1 B1 C1 D1 ?
. AB = 1, AA1 = x. A1 B1 C1 D1 ( 90 ).
A1 B12 = x2 + (1 x)2 = 2x2 2x + 1.
B1

x = , . .
2

C1 .
O
2. ,
A1
, ,
A
D
D1
.
. R ,
. 103
x , y .
4R2 x2 .
B

4R2 x2 = x2 + 2R y, y =
1

2R2 x2
R

2x + y + 2R = (x2 + 2Rx). x2 + 2Rx


R
x = R. y = R. ,
, .
3. AB BC ABCD
N P , BN = n, BP = m. M P N M E M F , (. 104).
S M F DE, BC = a, AB = b.
T

K n
N

y
x
a

D
. 104

112

. x, y DE EM M F DE. M K = EA = a x, KN = AK AN = y + n b.
m
ax
= ,
M KN P BN :
y+nb

y=

1
(nx + an + bm mn).
m
1
S = xy = (nx2 + (an + bm mn)x),
m

x > a m.

a > m b > n, an + bm mn = (a m)n + bm > 0.


an + bm mn
.
2n

nx2 + (an + bm mn)x x0 =


, x0 6 a.

an + bm mn
6a
2n
b
a
mn > bm an, 6 1.
n
m

S M F DE
(an + bm mn)2
.
4mn
b

> 1 x0 > a. [a m, a]
n
m
S . x = a m
b(a m) x = a
a(b n). , S M F DE ,
M P .
mn > bm an M P N , mn < bm an
N .
S M
P N M DE = x0 , :


1
bm
x0 =
(a m) +
.
2

bm

DE CP = a m BT =
n
AT k P N :
1
2

DE = (CP + BT ),

AT k P N.

, DE
, CP BT . DE > CP (x > a m) BT > CP . , BT = CP , . . mn = an bm
M = P .
113


14.1. .
14.2. AB.
C, ACB .
14.3. M . ,
) , ) .
14.4. ,
R

. .

r
14.5. AC ABCD (AB k CD) M
(. 105), AM P CM Q
(P Q k AB).
D
C
14.6. ABC. ,
Q
P

M
.
14.7. ABCD.
A
B
,
. 105
.
14.8. ,
.
14.9. ABC M , M A, M B, M C,
A1 , B1 , C1 . ,
+

AM
BM
+
+
M A1
M B1

CM
AM
BM
CM

M C1
M A1 M B1 M C1

.
14.10. M , ABC, M A, M B, M C,
BC, CA, AB A1 , B1 , C1 .
M A1 B1 C1 ?
14.11. A ABC ,
B C .
,
.
14.12. AB ABCD M .
CD N , ,
AN , BN , CM , DM , .
114

14.13. ,

r

1 2
(a + b2 + c2 + 4 3S).
2

15.

15.1. .
(. 14.1) , . 12
,
.
(. 7.24). ,
2p . , S

2
p2
p
S = x(p x) =

x ,
4

, , p/2 x =
= 0, . . x = p x = p/2. , .
.
: , (
), , .
XVIIIXIX . , . . .
.
. , .
, : (
), . , .
()
. , ,
.
15.2. .
115

1 . ,
, . ,
ABCDE (. 106) , , A , 180 ,
BE F ,
BCDEF .
2 . , , .
. m
ABCDE (. 107), . . P A + AB + BQ = P E + ED +
+ DC + CQ, : SP EDCQ > SP ABQ . P EDCQ m,
P EDCQC 0 D0 E 0 , , SP EDCQ , . .
, ABCDE,
. , 1
.

D1
C1
E1
B

E
C
D

E
. 106

. 107


.
. .
15.3. , .
1. , c
2p, .
. S 2 = p(p a)(p b)(p c), p
p c . S ,
116

(p a)(p b). (p a) +
+ (p b) = c , (p a)(p b)
: p a = p b, . . a = b (. 14.1).
, ,
, ,
.
2. . () ,
, ,
() .
. P Q M (. 108). M , P QC .
.
A
AB P
E
P QC
K
P1
M
A1
ABC. , SP QC > SABC
O
E
SBQK SAP K . E1
, SP QC
, M
C
B1 Q1
B
Q
E. . 108
N ,
, P1 Q1 C ,
E .
, ,

( )
( ). .
15.4. .
.
. A1 A2 . . . An 2p (
, ). 1 (. 15.2) . , .
, Ai .
117

Ai1 Ai Ai+1 Ai1 Ai+1


( ).
, Ai ,
Ai Ai1 = Ai Ai+1 ( 1 . 15.3). Ai , .
(
) , A1 A2 . . . An , Ai Ai+1 . ,
ai .
,
.
: 1) ,
ai , O,
ai , , 2) O Ai+1
A0i+1
ai , 3) .
Ai
O
A0i
109. OAi Ai+1 . c . 109
S A1 A2 . . . An ,
,
C
Ai Ai+1 , . S , c. A0i+1

2 . 15.3 OAi Ai+1


Ai+1
, , Ai Ai+1 .
B
Ai A0i
O
A0i A0i+1 A1 A2 . . . A0i A0i+1 . . . An
. 110
(
. 7.3).
OAi Ai+1 (. 110). c S OAi Ai+1
( ). c S .
2 ,
OAi Ai+1 ,
118

Ai Ai+1 . A0i A0i+1 A1 A2 . . . A0i A0i+1 . . . An ,


, A0i Ai + Ai Ai+1 = Ai+1 A0i+1 + A0i+1 A0i ( Ai BCAi+1 , A0i BCA0i+1 ).
, Ai1 Ai k Ai+1 Ai+2 (. 111), Ai1 Ai+2 . h, Ai1 Ai+2 . ,
l = a + b + x, x =
1
Ai+2
b
= Ai Ai+1 . (a + b)h Ai+1
2

x
h
a + b = l x, . .
x, Ai Ai+1 Ai Ai1 . , a
Ai
Ai1
Ai
. 111
Ai+1 . Ai Ai+1 ,
.
, ,
. . .
. , n-180

2p p2 ctg
.
n
n
n-
:
S6

1 2
180
p ctg
,
n
n

(15.1)

n-.
(15.1) , n- S ( n) n-.
15.5. , . n- ,
,
.
. w n M . , a n- M0 . He ,
, M , a. 119

, M
w, 1/n- .
M ,
a. , , SM < SM0 .
, M , .
,
B1
AB BC M
B
ABC (. 112)
A
C . ,
.
. 112
AB BC

( ). ABC w
B1 , AB1 a M0 (AB < AB1 < BC). AB1 C
ABC,
AC, . , M
M1 , , M ,
AB1 , a. M1
, , .
n M0 ,
SM 0 > SM i > SM .
,
, . 14.2 ( 1),
, ,
, , .
15.6. , . n- ,
, n- .
120

. M M0
n-, w. M0
K, . M0 SK
(c1 + c2 + . . . + cn ) SK K ci (i = 1, 2, . . . , n) ,
M0 (. 113). ,
M K , ,
M0 ,
w
.
c,
, c1 + c2 + . . . + cn
( ).
K M = F ( . 113).
:
. 113
SM > SF = SK c > SK (c1 + c2 + . . . + cn ) = SM0 .
S 2p M ,
w r, S = pr,
r S p , . .
n-.
. R1 , R2 , R3 M
ABC. ,
p
R1 + R2 + R3 > 2
3S > 6r,
(15.2)
r ABC .
. , M
ABC.
, 2S 2(R1 + R2 + R3 ).
(15.1) n = 6:

1
2S 6 (R1 + R2 + R3 )2 3,
6

S >

R1 + R2 + R3 > 2

3S.

27r (13.31),
R1 + R2 + R3 > 6r.


.
121

16.

16.1. . ( 2).
w(O, R) M , A B (. 114), M T .
M A M B = M T 2 = OM 2 R2 .
, M .
M w,
AB CD,
OM (. 115). M A M B = M C M D = M C 2 = (OC 2 OM 2 ) =
= OM 2 R2 . , , M w, , OM 2 R2 = 0.
T

A
O

. 114

. 115

, OM 2 R2 w(O, R) M . M
w. , w, . , w,
. w .
16.2. , .
. (O1 , R1 ) (O2 , R2 ) ,
.
. M : O1 M 2 R12 = O2 M 2 R22 , O1 M 2
O2 M 2 = R12 R22 . R1 > R2 . R1 = R2 ,
O1 M = O2 M M
O1 O2 . R1 > R2 a,
R12 R22 = a2 , ABC BC = a, , AB + AC > O1 O2
(. 116, ). (O1 , AB) (O2 , AC)
122

M N . ,
O1 M 2 O2 M 2 = AB 2 AC 2 = BC 2 = a2 = R12 R22 .

M
R1

R2 C

O1 O2

)
. 116

, . ,
, O1 O2
. M , X
. O1 M 2 O2 M 2 = O1 X 2
O2 X 2 O1 M 2 + O2 X 2 O1 X 2 O2 M 2 = 0. ,
(11.2) O1 M 2 + O2 X 2 O1 X 2 O2 M 2 = 2O1 O2 XM .
, O1 O2 XM = 0 O1 O2 XM
X . M , , M O1
O2

(. 116, ). M .

. 117
(. 117), () .
, , .
O1

O2
, (. 118).
. 118
, , M (. 119).
123


, . R2 = 0 M
O2 M 2 . (O1 , R1 )
O2 , O2
(. 120).

T
M

O1

O2

O1

. 119

O2

. 120

16.3. . , , , T
.

O1
O2
. ,
(. 121).
. 121

.
A
1 . r
r
,

O1
O2
a2 . , l
a1
,
.
. 122
, A l a1 a2
, ,
A , r (. 122).
(A, r) .
124

, (A, r) a1 a2 , A
(A, r).
,
A
r
r
, ,
A
.
2 . O1
O2

a1
a2
l
, . 123

. ,
, .
, A l
a1 a2 (. 123). A
, A
O1 A O2 A (. 16.1). (A, r), r
, a1
a2 . .
16.4. .
.
. ,
, ,
.
, a a g b b g P (. 124).
P P
a,
a
b
b, g.
a b.
A
B
. , C
b
, a
g
.

. 124
. 16.2
.
125

, , . 124 a b.
, a b
.

16.1. , .
16.2. ,

A.
16.3. , ,
A, B, C,
a, b, c.
16.4. , A
B a.
16.5. ,
.
16.6. A B , a .
16.7. A B ,
.
16.8. ,
. ?
16.9. BC, CA, AB ABC
A1 , B1 , C1 . ,
AA1 , BB1 , CC1
ABC.

17.
17.1. . . .
l a b
b g,
a g .
, P l
a b
126

b g , , a g. P a g. P l, l
a g. ,
, l. ,
, . . a, b, g .
, ,
, A B. AB

(. 125). , .
. ,
A
l,
. l .

, Bl
.
, . 125
.
1) - A
B.
() .
, .
, ,
. ,

(. 125).
2) - A ( A
). l
A. l
. 126

l A.
, ,
.
(. 126).
3) - .
,
127


. .
.
.
17.2. . .
,
.
1. , , , .
, a1 a2
M , m w A
(. 127). 1 (. 16.3) A l a1 a2 . l
a1 a2 M ,
A m. , M
. , M
l.
, x
l, A
w
a1 a2 .
a1 a2 , . . m, m

x w ( 1
O1
O2
). , a2
a1
M l
.
. 127
.
,
.
2. ,
, .
, .
1 .

128

:
1) ,
2) ,
3)
,
4) , .
17.3. . 2 ,
, ,
. . .
, . 128
, (. 128). , I1
I2
, (. 129).
. a
b. l
. 129
A
(. 130). r l
a
A a b ,
b
T
A O1 O2
O1
O2
. I1 r
I2
w(A, r) O1 O2
A
I1 , I2 . ,
w , w
,
. 130
O1 O2 . , . . .

129

I1 I2
.
17.4. . , , .
. , ,
.
. (. 125 126).
,
.
.
a1 a2 (. 131)
M , l . w, a1 a2 . , . b
, M ( I1 I2 M ). x,
M b. b M
O1 O2 X .

M
x

a2

B
m
A
O1 I1

X
I2 O2 X

a1

a2

g
a1

l
. 131

,
M (O1 O2 ). , x .
g
O l
130

. 132

, a1 a2 (. 132).
m g M .
X ( 1).
, M ,
. M
x1 x2 , a1 a2 .
M , , M .
. ,
A B a.
. x
, A B. w , a
M N (. 133). AB, M N t a, w, x, , P ,
, AB M N .
T T1 a t t1 , P , X X1 .
. 2, 1, 0 .
P
t1
t
T1

A
M
O
X

x1

T
N

B
X1
x

. 133

131

17.1. , , .
17.2. , A
B .
17.3. , w
A.
17.4. , ,
, .

18.
18.1. . A B w(O, R),
OA OB = R2 .

(18.1)

(18.1) :
1) AOB , ;
2) M ,
w,
OM OM = R2 M w;
3) O w , 0 OB = R2 ;
4) A 6= O w,
B , OA OB cos AOB = R2
OA < R OB > R.
(18.1) ,
A, O,
B,
OB cos f =

w
f
O

. 134

B0

R2
= OB0 ,
OA

OB f = AOB ,
B0 OA (. 134).
OB0 R,
R, OA: OA : R = R : OB0 . B0 OA, A w.
132

OB cos f = OB0 (f !) , B0
B OA.
, ,
A w, BB0 , OA. A w. A .
O w . , O,
.
A
w. .
, a A OA.
CD w, OA (. 135).
(AC) w = P , (P D) (OA) = B. B a. ,
C
w

P
AOC BOD OA/OC = OD/OB, . .
O
B
A
2
OA OB = R . a A
B OA.
A w,
a
w
D
,
. 135
.
. A
P
w,
w
, A
B
O
w A.
, AP AQ w (P Q) (AO) = B (. 136),
Q
AOP a
OA OB = OP 2 = R2 , . . B . 136
A w.
. A
p
w
w, a
w
AO.
P

2 9:

. 137

133

.

P . P w (. 137). p P
.
, P . .
P , P w.
18.2. .
: B a A, A b B.
, , B a A, , A B w.
A b B (. 138).
c C
a b A B .
: .
w
B
b
: c
C
, ,
O
.
a
18.3. ..
A
ABC (. 138) :
, . 138
.
, ,
w.
O w , OA a, OB b,
OC c.
ABC ,
w,
.
, , O w
. O ABC,
.
134

ABC , , .
A B w, a b
w C. ,
C ,
(. 18.1, 4). , w. C
ABC, w,
OAB , OAB , ACB .
18.4. .
. O, ( O)
( , O) :
.
, .

.
, .
,
,
P
,
D
, C

M
.
B
A
N
, , A, B, C, D, . 139
,
,
(. 139), ,
F
a, b, c, d,
C
c
, , D
b
(. 140).
d
B a
A
( E
), . 140
.
135

ABCD AB, BC, CD, DA, AC, BD


, M = (AC) (BD), N = (AB) (CD),
P = (BC) (AD) ,
M N , M P , P N .
abcd a, b, c, d
, A = a d, B = a b, C = b c, D = c d, E = a c,
F = b d , AC, BD, EF .
w ,
w ( w).
w , .

.
()
:



()

,
,
,
,
,
(),
,

. . , . .
.
1. ( 10):
. B
C


P
. ,
A
D (). ,
F
E
. 141
. ,
: , 136

(. . ),
(. 141).
(17851864) 1806 ., . .
2. ,
, , ,
(. 142).
:
(. 143). ,
. .
. .

. 142

. 143

3. A B , a b
O, AOB = f.
a b, O, 180 f. : 1)
180 , 2) O , ABCD,
AOB + COD = BOC + AOD = 180 .

18.1. ,
,
.
137

18.2. M AB ABC, H
. ,
1
4

M C M H = AB 2 .
18.3. A B
,
. .
18.4. . , .
18.5. A A1 , B B1 w.
, C = (AB) (A1 B1 ) C1 = (AB1 ) (A1 B)
w. , AA1 , BB1 , CC1 .
18.6. . ,
, . .
18.7. ,
. , .
18.8. A.
A.
18.9. O w. , w .
18.10. , (. 10.9). , .
18.11. ,
(. 10.11).


1. a b,
a. .
2. S, m.
.
3. , , 3 4.
.
138

4. ABCD AD = 3 39 BC = 39,
BAD = 30 , ADC = 60 . D ,
. ,
.
5. , 3 4.
.
6. ABC AK
BM , ABC 120 .
ABC .
7. a, b, g ABC AM = m. .

8. ABCD AD 7.
K, AK = 1, KD = 2, BAC = DAC.
ABC.
9. a.
,
2 : 1 b
?
10. AD BM ABC
5 : 1, B.
ABC?
11. AB, BC, CD, DA
ABCD K, L, M , N , 2 : 1, 1 : 3, 1 : 1, 1 : 5.
ABCD 1. AKLCM N .
12. a b (a > b), ,
a.
13. ABCD AD a, CD, , 2a.
E CD, CBE a.
.
14. , e f m , .
15. C ABC
l, CM ABC. A1
B1 A B l. ,

1) AA1 B1 B
ABC,
139

2) 4AA1 BB1 = A1 B12 .


16.
6 5. .
17. D AC ABC.
, ABD ABC, AD : AC = n.
18. . , ,
A D, B C.
, AB : BC : CD = 2 : 4 : 3.
19. , ABC, BM
. .
20. a b (a < b).
, .
21. ABC CD
AB. ,
BDC ADC, BC = 4, AC = 3.
22. ABC
BC D, BC E. DE, 3 4,
BCA 120 .
23. ABCD BC = 2 AD = 10 ,
. . .
24. E AC ABC,
K AE. , E
AB, , C AB, , C
BC, D. BKD.
25. ABC (AB = BC) A1 B1 C1
(A1 B1 = B1 C1 ) . A1 , B1 , C1
BC, BA, AC ABC. B1 C1 BC
. ABC.
26. AB ABC N L,
AC M BC. AB, M N : AN = 1/6
AM = 10/27.

140

27. ABCD O.
, ,
OAB, OBC, OCD, ODA, . , 45 ,
.
28. P Q BC CD ABCD. , AP AQ , .
29. A B.
A , C D,
B , CD.
E F . , C, D, E,
F .
30. M AC ABCD , AB BC N . ,
DAM DBN , AB k DC. .
31. ABC B , AC = 6. , ACH (H
ABC), ABC. ABC.
32. ABC
A CD, BC M .
,
CM
= cos2 A.
MB

33. I ABC . , , IAB,


ACB.
34. ABC BD. , ABD, AB = 21, BC = 15
5
cos BAC = .
7
35. ABCD S,
M . , S1
S2 M AB M CD

S1 + S2 = S,
.
36. , AB ABC, AC A1 BC B1 . C
, AB D.
141

, CA1 DB1 CAB CA1 B1 .


.
37. M ,
A B. A ,
BD N . ,
M D AN .
38. BD ABCD m, AD n. CD, CD = CA = CB.
39. ABC AB = 20, AC = 24. , C,
A
BC , AC. ,
ABC.
40. , ABC , , AB,
, cos C = cos A cos B. .
41. R ABC. ,

IC 2 = ab 4Rr

ID2 = 4R2 ab,

I , D C.
42. ABC D, I
O .
, CD2 = 4R2 BC AC.
43. w O
OA OB. A OAB M
AA1 w. AM : M A1 .
44. R ABC.
C , AB M , N .
CM CN .
45. C ABC, ,
A B .

46. ABCD AC = AB 2. ,
.
142

47. ABC
sin 2A + sin 2B
sin A + sin B
=
.
sin C
sin 2C

, cos A + cos B = 1.
48. , , m h.
.
49. ABC:
IA2
IB 2
IC 2
+
+
= 1.
bc
ca
ab

50. ABC:
2S = IA2 sin A + IB 2 sin B + IC 2 sin C.
51. , ri :
r 1 r2 + r 2 r3 + r 3 r1 = p2 .
52. .
53. ABC:
r + r1 + r2 r3 = 4R cos C.
54. ,
.
.
55. AB ABC
BB1 K.
BC M .
R, BK = a. M B.
56. O ,
ABC. , B OA,
AC K. , C OA,
AB M . BC, BK = a, CM = b.
57. BC ABCD P . , ABP , BD
Q. , CP Q,
BD L. , A, P , L
.
143

58. R r
A. , ,
B, C. , ABC.
59.
ABC C 60 ,

2 3. D AB 2 : 1,
A. ABC, CD = 2 2.
60. ABC (C = 90 ). D
DEF A B, A D E.
ABC DEF LK, AB. DKLB 5/8
ABC. DEF .
61. ABCD AD = DB, CAD = 30 ,
ABC = 150 . , AC BCD .
62. , , h. ,
.
, .
, , .
63. ,
, , .
64. a b, ,
A B, A a,
B b. A b,
B a. ,
a ,
b . ,
.
65. a,
b. .
66. ABC AM
CN , O . AC, OM BN S, ABC = b.
67. AB CD.
AB M , AM = AC, CD
N , DN = DB. , M N
, M N AD.

144

68. , ,
, , , . .
69.
. , .
70. ABCD. , ABC ACD, AC P Q.
, ABD BCD, BD M N . , P Q = M N .
71. A ABCD , 45 . BC M
BD N , CD P BD Q. , C, M , N , P , Q .
72. , , ,
, .
73. ABCD ,
O, AOB, BOC, COD, DOA . .
74.
,
. .
75. ABCD . , ,
, .
76. ABCD, BD AC. ,
2BD2 = AB 2 + BC 2 + CD2 + DA2 .
77. ABCD, M . , M BD , BA : AD = DC : CB.
78. , , ,
, , 180 .
79. A1 A2 A3 A4 A5 A6 A7 . ,
A1 A24 = A1 A22 + A1 A3 A1 A4 .
145

80. , S ABCD
1
4

S 2 = (p a)(p b)(p c)(p d) + (e2 f 2 (ac + bd)2 ).


81. ,
.
82. ,
ABCD
abcd sin2

B+D
.
2

83. M , BD ABC, AM CM , BC AB
P Q. , P DB = QDB.
84. a.
, , .
, ,
.
85. ABC, 90 . ABC.
86. ,
. .
87.
. .
88. , ABC 4S = BC 2 + CA2 ,
.
89. ABC
A1 , B1 , C1 ,
A2 , B2 , C2 . , A1 B1 C1 , A2 B2 C2
.
90. , , .
91.
H. ,
, H ,
.
146

92. . , ,
, .
93. , , ,
, .
94. .
,
.
95. AB ABC P . , , AP C BP C, .
96. ABC
CD AB CE CF ACD BCD.
,

SABC : SCEF > 1 + 2.


97. , ABC

n
n
n
sin A + sin B > sin C, n N.
98. ABC
3
2

cos 2A + cos 2B cos 2C 6 .


99. ,
R

R 5.
100. a b. R1 R2 ,
.
5
, R1 + R2 > (a + b).
8
101. AA1 , BB1 , CC1 ABC A2 , B2 , C2 . ,
AA1
BB1
CC1
9
+
+
6 .
AA2
BB2
CC2
4

102. r ABCD.
, AB + CD > 4r.
103.
ABCD,

AD k BC. , AB + CD > 2 S (S ).
147

104. I
, S. ,
S > 2r2 (r ).
105.
(m2a + m2b + m2c )(h2a + h2b + h2c ) > 27S 2 .
106. , ABC

3a2 + 3b2 c2 > 4 3S.


?
107. , 1
.
8
108. , ABC M :
AB M C 2 + BC M A2 + CA M B 2 > AB BC CA.
M ?
109113
.

B
C
3 3
A
cos cos 6
.
2
2
2
8
9
110. 9r 6 ma + mb + mc 6 R.
2

109. cos

> 3 3S.
111. la2 + lb2 + lc2
112. 5R r > p 3.
113. r2 + r12 + r22 + r32 > 7R2 .
114. A a, B b.
, A b
B a ,
AB.
115. AB M N ,
AN
AM
=
. , M N (
NB
MB

) , A B.
116. M , ABC,
R1 , R2 , R3 ,
x1 , x2 , x3 . ,
R 1 R2 R3 >

4R
x x x .
r 1 2 3

M ?
148

117. ABC
AC a, a. M
BC
BC AC AB.
M C.
118. ABC AC
a, a. M , , , ABC
. .
119. R AD.
A B, AD
C. BC ?
120. ABC
ABEF , BCP Q, CAM N . EF M N P Q, BC = a, CA = b?
121. . , (
) ,
.
122. , ,
,

a2

4S 2
( ). .
+ b 2 + c2

123. AC C ABCD
P M N AB
CD P M P N , BC AD E F . , EF
.
124. , , AB
ABC , , C
AB.
125. O A
, OA X Y . , XY
.
126. , (. 10.2) ( 8, 2) .
149

127. , ,
.
128.
.
129. ,
.
130. , .
131. A B ,
.
132. ,
.
133. A, B, C D ,
ABCD .
134. O ABC, . , , ,
O BC, CA, AB,
A, B, C, .
135. AOC. B AO ,
CAO = BCO. w O OC.
w1 B O w
M K. , A, M , K .
136. , p S
p2 > 4S.
137.

.
138. AP , BP , CP BC,
CA, AB ABC A1 , B1 , C1 .
A1 B1 C1 , BC, CA,
AB A2 , B2 , C2 . , AA2 ,
BB2 , CC2 .

150

II

.

.
() . , , (), .

.
. , , .
1 . . f X Y , x X
f
f
y Y . : f : X Y, y = f (x), a X Y , x y.
y x, x
y f X Y . x x0 : f (x) = x0 .
X 0 x0 x X
X f . : f (X) = X 0 . ,
X 0 Y . X Y .
2 . f (X) = Y , , X
Y . .
, , ,
: Y
X.
3 . f X Y ( ),
. f 1 Y X. f (x) = y f (X) = Y ,
f 1 (y) = x f 1 (Y ) = X. , f f 1 , . . (f 1 )1 = f .
4 . f (X) X, , X
. f (X) = X , X .
5 . . ,
, 152

. :
1) , 2)
. : 2 )
, 2 ) .
6 . f g X f (x) = y,
g(y) = z x X. , y X z X. f f(x) = g(f (x)), f(x) = z.
f
X. f
() f g.
: f = g f , f = gf . , , , ,
(g f )(x) = g(f (x)).
7 . f1 f2 X (), x X
f1 (x) f2 (x) : f1 (x) = f2 (x).
8 . , . . f , g, h :
h (g f ) = (h g) f.
. x X f (x) = y, g(y) = z,
h(z) = t. (g f )(x) = z, (h (g f ))(x) = t. C ,
(h g)(y) = t, ((h g) f )(x) = t. , (h (g f ))(x) =
= ((h g) f )(x) 7 h (g f ) = (h g) f .
. , , .
, , ,
.
,
.
9 . E X , x X E(x) = x.
f E f = f E = f f 1 f = f f 1 = E.
10 . f :
f (A B) = f (A) f (B).
153

. x A x B. f (x) = x0 , x0 f (A)
x0 f (B), x0 f (A) f (B). , x

f . , y f (A) y f (B).
f 1 (y) A f 1 (y) B, , f 1 (y) A B. ,
y .
11 . f :
f (A B) = f (A) f (B).
.
12 . f ,
, , , . . f = f 1 f 6= E.
x x0 : f (x) = x0 , f (x0 ) = x.
. .
. f , 9 f f = f 2 = E.
:
, , , , , . .
,
.
.
. . :
. .
,
, , . .
.

154


1.
1.1. . f , :
f (A) = A1 f (B) = B1 , A1 B1 = AB ( A, B). (. 3 ), AB > 0 A1 B1 > 0, . .
. , f
(5 ).
. , , . ,
f ,
A B A1 B1 ,
A1 B1 = AB.
1. , , .
2. .
. F1 () F (F1 = F ), , F
F1 .
, .
1.2. . , , ,
. .
, , .
. .
,
.
. f a. A a, B b, A 6= B, f (A) = A1 , f (B) = B1 , A1 6= B1 .
A1 B1 , f (a) = (A1 B1 ).
M1 M a. M
A B, AM + M B = AB. A1 M1 = AM ,
M1 B1 = M B, A1 B1 = AB. A1 M1 + M1 B1 = A1 B1 . -

, M1 A1 B1 ( A1 B1 ). ,
A M B B A M , , M1 M a
A1 B1 . , A1 B1 f a, . .
a A1 B1 . , A1 B1
f 1 AB,
.
1.
.
, , M A B, M1 A1 B1 .
2. .
3. .
,
, . a
.
4. .
, a l,
A
/ l, M ,
AM l. 2
1 f a a0
l0 = f (l) A0 = f (A).
5. (
) .
, , ,
, , ,
. 4
11 12 (. ) .
.
.
a b a0 b0 .
. a0 b0 M 0 , M ,
a, b, a k b.
, , , , ,
156

, ,
, .
1.3. . ,
. , ,
.
( ). A, B, C A1 , B1 , C1 , A1 B1 = AB,
B1 C1 = BC, C1 A1 = CA.
, A A1 , B
B1 C C1 .
A, B, C , A1 , B1 , C1 .
a AB, C, a1 N
A1 B1 ,
M
C
C1 . a
a
1
a
AB A1 B1 (. 1).
f a
A
B
: M M1 , A1 M1 = AM , B1 M1 = BM M1
N1
C1
a1 M a, M1 a
1 M a
. , , , f (A) = A1 , M
1
B1
f (B) = B1 , f (C) = C1
AB f a1
a1
A1 B1 . ,
A
1
f . f (N ) = N1 . , M1 N1 = M N . A, M , N
. 1
,
. , AM B A1 M1 B1 AN B A1 N1 B1 M AN M1 A1 N1
M AN M1 A1 N1 , M1 N1 = M N .
f . f g, g(A) = A1 , g(B) =
= B1 , g(C) = C1 , P , f (P ) = P1 , g(P ) = P2 ,
P1 6= P2 . P1 P2 A1 B1 . AP = A1 P1 = A1 P2 BP = B1 P1 =
157

= B1 P2 . A1 B1 P1 P2 , P1 P2 . , P1 P2 A1 B1 .
f 6= g.

M
(. 2).
A
1.4. . ,
B
ABC C
,
A B C
. C1
A1
,
, ABC .
B1
:
ABC ,
BAC .
M1
: 1) . 2
ABC A0 B 0 C 0 , 2)
. , ABC A0 B 0 C 0 ,
.
ABC A0 B 0 C 0 ,
.
. ,
, . , ,
.

. , A0 B 0
A B (A0 B 0 = AB) , C 0
C : ABC
.
, A B A0 B 0 ,
0 0
A B = AB, ,
A A0 B B 0 .
158


.
, .

2.
2.1. .
M M1
O, O M M1 . O
. ,
M
O, O
ZO : ZO (M ) = M1 .
, ZO (M ) = M1 , ZO (M1 ) = M , . . , , :
ZO1 = ZO , ZO ZO = E.
, (12 ). M M1 O.
F F1 ZO
(. 3).
. .
. ZO (A) = A1 ZO (B) = B1 ,
A B A1 B1 = AB,
OAB OA1 B1 (. 4). , O, A B , .
B

F1
O

M1
O

A1

B1
. 3

. 4

.
. 1.2
.
159

. , , . ,
, .
AB A1 B1 AA1 (. 4).
F O,
: ZO (F ) = F . ,
O
.

. . 5

(. 5). .
, .
2.2. .
.
1. ABCD
,
M N P Q. ,
C
N
P
.
. (AC) (BD) = O (. 6).
O
D

B
ZO (A) = C.
Q
M
A
A (M Q), C (N P ) (M Q) k (N P ),
Z
. 6
O (M Q) = (N P ). ZO (M N ) = (P Q).
10 M
M Q M N P (N P )
(P Q): ZO (M ) = P . , O M P .
2. N
, , - M B
C
O
. , A
P
D
.
?
Q
. ABCD . 7
(AM ) (BC), (AQ) (CD), (CN ) (AB),
(CP ) (AD) (. 7). (AC) (BD) = O,
AM CP AN CQ , O M P = AC = N Q. ZO (M ) = P ZO (N ) = Q, M N P Q , 160

, . A, N , M , C, Q, P
O. BAD = 1/2N OP .
, M N P Q ,
, N OP = 90 , . . BAD = 45 ( ,
A
C ).
3. A ,

.
. m a , CD , C m, D a (. 8). ZA (C) = D. ZA (m) = m1 ,
D m1 , , D a m1 .
m1
: a
F
m1 m ZA , D E
m
D
m1 a
l
A
A DA
C
EA. .
4.
E
, . 8
.
. ab P
(. 9). P M N , M a, N b.
, M P < P N . a1 ,
a P . b a1 = B, a1 (M N ) = K,
(P B) a = A. OAB OM N . , AM P BKP ,
N
P . SOAB = SOM P B + SAM P = SOM P B +
+ SBKP = SOBKM < SOM N . , N
a1
B
K
O B (A O M ),
. b
P
M N , a
O
M
A
AB
P , OAB
. 9

, ,
P . AB:
a1 = ZP (a), B = a1 b
(AB) = (BP ). .
161


1.01. A B AC BD. ,
CD .
1.02. E F AD BC ABCD. , BE F D
AC .
1.03. ABCD A C. , ZA (D) ZC (D) H ABC.
1.04. , 1 : 2,
, ,
. ,
.
1.05.
. ,
.
1.06. ABCD B D, AC . ,
.
1.07. O ABCD. , , AOB, BOC, COD, DOA,
. ?
1.08.
. , .
1.09. ABCD M .
A, B, C, D , M C, M D, M A, M B. ,
.
1.10. O
. , O
, M N
, .
1.11. , ,
, .


1.12. O M . OM , O.
162

1.13. ,
.
1.14. , , ,
.
1.15.
, .
1.16. ,
, .
1.17. O. O,
.


1.18. ,
. ,
. , ,
, .
1.19. A B, , . , ,
?
1.20.
. , .
1.21. , , ,
.
1.22. .

, . , .
1.23. AB CD.
CD P . M ,
AM BM CD ,
P .
1.24. ,
,
a.
1.25. ABC AM CK.
BAM BCK 30 . , ABC .
163

1.26. ABC P . P1 , P2 , P3 BC, CA, AB .


Q1 , Q2 , Q3 P1 , P2 , P3 BC, CA, AB. , BC, CA
AB Q1 , Q2 Q3 .

3.
3.1. . M
M1 l, M M1
(. 10). l . , l,
l Sl : Sl (M ) = M1 .
M M1 l,
Sl (M1 ) = M . , , , : Sl1 = Sl , Sl Sl = E. ,
(12 ).
,
. l , A B
(. 11). M
/ l Sl (M ) = M1 ,
: AM = AM1 BM = BM1 . , M1
: A AM
B BM (M ).
F F1 l (. 12).
M

l
A

B
M1

M1
. 10

. 11

M1

F1

. 12

. .
164

A B Sl (A) = A1 , Sl (B) = B1
, A1 B1 = AB. , . 1317.
,
.
. 1.2
. , , , .
B1

B1

A1
l

A1

. 13

A1

. 14
B

B1
. 15

B1
l

A1
A1

B1

. 17

. 16

. , ,

.
. , l
. m l = P Sl (m) = m1 , m1 6= m Sl (P ) = P ,
P m1 (. 18). m k l, m1 k l,
m m1 l, m k l (. 19).
m
m
l

P
m1

m1
. 18

. 19

165

, , l, l (. 18).
l F , l F : Sl (F ) = F . ,
F l.
, , ,
. , M w O, O l, Sl (M ) = M1 .
Sl (O) = O OM1 = OM , . . M1 w. , . , Sl (w) = w.
,
. ,
.
3.2. . .
1. M N C
ABC , C
A B. , P
T
Q
AB
J
ABC M N .
B
N P M
. Q T A
. 20
BC CA (. 20).
AT = AP , BQ = BP , P Q T .
CQ = CT , M P = N P .
S
2.
D
C
AD BC ABCD S. , , ACS
M
BDS, ,
A
B
.
. M ACS BDS (. 21).
. 21
SM
, , . , ASM =
= BSM `AM = `M C AM = M C. , , AM =
= M B DM = M C, M .
166

3. A. ABC , B C .
. M N , A . M N B C (. 22). , B1 C1 -
,
A

A1 B1 + B1 C1 + C1 A1 = M B1 + B1 C1 + C1 N >
> M N = M B + BC + CN = AB + BC + CA.

N
C

M
B
B1

C1
, ABC . ,
.
4. m a b m.
O
m ,
. 22

m .
p
. M
A
B
a
b
m, a t p a b,
M m
m (. 23). m
M
t p. b1 b t
b1
m, t b1 (. 10 ). B1
,
M
. 23
a b1 . a b a
C
b1 . , .
w Ob
a
. 23 .
B
5. a, b, c A
c
P
n
m
w.
, w,
. 24
.
. ABC A a, B b, C c
(. 24). m = Sa (c) n = Sb (c), P AB
w m n ( 1).
P , AB. w

167

P , a b, A B.
c C.
.


1.27. .
, , . ,
.
1.28. , , , ,
A B, C D, E F . , AB = CD = EF .
1.29.
, . .
1.30. ,
. .
1.31. A, B, C . M . , M A, M B,
M C .
1.32. , ,
, ,
.
1.33. H ABC, , HAB, HBC, HCA, . .
1.34. , ,
, ,
, ,
.
1.35. C ABC M 6= C. , M A + M B > CA + CB.


1.36.
, .
1.37. ,
.
1.38. a b.
, m
a b , m.
1.39. ,
, .
168

1.40. a M N a.
a P , M P + N P
.
1.41. ABCD, AC
A, .
1.42. .
1.43. , ,
.


1.44. M AB . CD M AB 45 . ,
CM 2 + DM 2 M .
1.45. , , . ,
.
1.46. . ,
.
1.47. , , , ,
.
1.48. m, M a
m.
P , M P
P .
1.49. m a. m X, m ,
a X,
.
1.50. AB BC. P ABC. ,
AB. ,
?
1.51.
, ,
.
169

1.52. ,
, .
1.53. ABCD,
, AB AD B D.
1.54. ABCD M
N . M , , AB
BC, N ?
1.55. O P Q.
ABC , AC = BC, AB , C , AC BC
P Q.

4.
4.1. . Tr
r , M M 0 , M M 0 = r.
, , ,
, . . (5 ).
Tr r. ,
:
T0 = E. , Tr, : (Tr)1 = Tr .
. .
. Tr(M ) = M 0 Tr(N ) = N 0 , M M 0 =
= N N 0 = r. , M M 0 + M 0 N = M 0 N + N N 0 , M N = M 0 N 0
, , M N = M 0 N 0 .

a
a0
r
0

, M
M
.
. N
N0
,
.
. 25
. . M N
a Tr(M ) = M 0 , Tr(N ) = N 0 , M N = M 0 N 0 (. 25),
Tr(a) k a. .
, , . ,
170

f M M 0 , N N 0 . M N M 0 N 0
M N = M 0 N 0 , M N = M 0 N 0 , M M 0 = N N 0 , . . f .
, , . , ,
, . . ()
.
4.2. . , .
1. AB ABCD ABE. C D CM DN AE BE. ,
P = (CM ) (DN ) ,
E
B1
A1
ABE (. 26).
H
. H M
N
ABE. BC A
B
P
AH, BH, EH
DN , CM , EH. ,

D
C
P , H
.
. 26
2. M N .
a1
a
P Q M1
M
P
Q
M N (. 27). ,
O1
O
2
2
M N + P Q
N
.
. a a1
. 27
, P a, Q a1 .
P Q a a1 .
M1 = TP Q (M ), M1 a1 , M a. N M M1
M N 2 + M M12 = N M12 . M M 1 = P Q. , M N 2 + P Q2 = d2 , d
.
3. ABCD
A B, .
171

. w , ABCD (. 28). C
D. , TAB (D) = C. D
A
w1 w
AB, C w1 , D w. ,
C
O
B C w w1 , w1 .
w
w
O1
w1 .
w1
4. AB
CD.
M ,
. 28
AM BM CD ,
.
A
. M (M A)
B1

(CD)
= P , (M B) (CD) = Q,
B
a
a a
P Q. C
D r
= QP , a
P
Q
a
, CD, . Tr(B) = B1 (. 29).
M
a AM B AB
a
. (M B) k (P B1 ),
. 29
AP B1 = a. , P AB1 a. : B1 = Tr(B), AB1 ,
, a. P CD.
M . .


1.56. , . .
1.57. , , .
1.58. ABCD M . ,
, AB BC, , AM , BM , CM , DM .
1.59. AB
ABC M . ,
M AC BC .
172

1.60. ,
, .
1.61. K.
KA KB . AB,
r.
1.62. P Q. ,
,
A B, C D (AB = CD). , AP C
.
1.63. ABCD u
v. A1 = Tu (A),
C1 = Tu (C), B1 = Tv (B), D1 = Tv (D), A1 B1 C1 D1 . .
1.64. H ABC. ,
AB 2 + CH 2 = 4R2 , R .


1.65. O M . , OM , O.
1.66. .
1.67. ,
.
1.68. ,
, .
1.69. .
1.70.
.
1.71. , .
, .
1.72. , .
, .
1.73. ,
.
1.74. ABCD,
, ,
A B .
1.75. .
1.76. ABCD,
AB CD, .
173

1.77. A B.
, A B
?


1.78. ABCD BC AD A B M , C D N . , |AB + CD BC AD| = 2M N .
1.79. ABC, AB
. AC BC AA1 C1 C BB2 C2 C. A1 C1 B2 C2
D. AB ABB3 A3 , AA3 = DC. ,

( ).
1.80. A B .
, A B ?
1.81.
, , , a.
1.82. A. A
, .
1.83.
, .

5.
5.1. .
O a , O
M0
, M a 360
M 0 , OM 0 = OM a
M OM 0 a (. 30).
O
M
O , a
.
. 30
a ,
M OM 0 , . O a a
RO
.
174

(360 a)
a360
a+360
a
a
, RO
= RO
= RO
RO
= RO
.

a
a+360 k
RO
= RO
,

k = 0, 1, 2, 3, . . .

0 6 a < 360 180 < a 6 180 .


a
, , RO
a
a 1
.
) = RO
, : (RO

a
180
a
180
RO
6= RO , RO
= ZO (. 31), . .
= RO
B1
A
B
O A1
360
0
= E.
= RO
. , RO
. a
l
.
a
a
(B) = B1 . ,
(A) = A1 RO
RO
. 31
A B A1 B1 = AB. , O, A, B .
, O, A1 , B1 . ]AOA1 = ]BOB1 AOA1 + A1 OB = A1 OB + BOB1
AOB = A1 OB1 . , , OA = OA1 OB = OB1 ,
OAB OA1 B1 A1 B1 = AB.
, .
5.2. . ,
, , ,
.
.
p
1
h1
a
RO

a .
. A

h O ,
1
a
h
O
. p O,
p
a

h O,
A
p. . 32

, h1 p1 h p (. 32).

. p p1 h h1 , . . a
.
175

, ( ),
.
5.3. . , ,
. 180 , . A A1 , B B1
A1 B1 = AB. , A B
.
A1 B 1 = AB . O , OA1 = OA OB1 = OB. , O AA1 BB1 . ,
(. 33). , ,
, A A1 , B B1 l, (AA1 ) 6= (BB1 ) (. 34),
(AA1 ) = (BB1 ) (. 31). O = (AB) (A1 B1 ),
O = l (AB).
. AB A1 B1 K (. 35). O
AA1 K BB1 K.
, , A1 = K, AA1 K A1 B1
A1 .

B1

O
A1

B1

a B1

B
A1

A1

A
B
. 33

O
. 34

. 35

6.

. , 176

, ,
.
1.
ABC A1 B1 C1 . , A2 = (BC) (B1 C1 ), B2 = (CA)
(C1 A1 ), C2 = (AB) (A1 B1 ) .
. O 120 A, B, C B, C, A (. 36).
, (AB) (BC), (BC) (CA),
(CA) (AB) (A1 B1 ) (B1 C1 ),
B
B1
(B1 C1 ) (C1 A1 ), (C1 A1 ) (A1 B1 ).

C2
, C2 A2 ,
A2
O
A2 B2 , B2 C2 . , - A1
A2 B2 C2 .
A
C
B2
2. B A C.
AB
C1
ABM BCP . . 36
K E AP M C.
, BKE
M
(. 37).
60
A P
. RB
EP
M C.
K
AP
M C. A
B
C
60
, RB
(K) = E. BK = BE
. 37
KBE = 60 , ,
BKE .
B
3. A1 B1 C A1
M
ABC M
1
C.
, CM ABC
B2
C
A B1
A1 B1 (. 38).
90
. RC
.
. 38
90
90
90
RC
(B) = B2 , RC
(M ) = M1 . RC
(A) =
= A1 M AB, M1 A1 B2 . CM1 k A1 B1
. (CM1 ) (CM ) , ,
(A1 B1 ) (CM ).
4.
. 177

,
m.
. O ABM N (. 39).
90
90
90
RO
(B) = A, RO
(A) = N . RO
(C) = C1 , C, A, C1 , CAB + BAN + N AC1 = 180 .
COC1 M
CC
1 = m.
2CO2 = m2 CO = m/ 2.
5. B

A B. O
,
, N

A,

C
C
A
1
, B.
. 39
. M
a
M1
a
B
M B
b
b M1 . M
O1
O
]OAO1 = f, O O1
a b (. 40).
f
A
, RA
(a) = b. f
, RA
(M ) = M1 . . 40
, AM B =
= AM1 B = AOO1 = AO1 O (
A
AB) , , M A = M1 A,
M AM1 = f.
6.
B1
C
ABC, C1
A, O1
O
B l, B

C a.
a1
a
.
ABC l
60
(. 41). , RA
(C) = B.
. 41
a1 a
. C a, B a1 .
, B l B a l. 60
. : a1 = RA
(a),
l a1 = {B, B1 }, AB AB1 178

ABC AB1 C1 , C C1
a. l a1 .
7. A
, B
f.
. ,
A.
B
B
r
f
w f
r (. 42).
P
A
w r
Q
B , , w
. 42
(B, BA).
w P Q,
A
. , B
]QBA ]P BA. w . .


1.84. F
48 O.
O 72 ? 90 ?
1.85. AB BC ABC ABP Q BCM N , ABP Q ABC AB,
BCM N
BC. , P N AC .
1.86. ,
60 . , ,
, .
1.87. . , .
179

1.88. M , P , N , Q , . , M N P Q
, .
1.89. , .
1.90. M , N , P , Q AB, BC, CD, DA ABCD . ,
AN , BP , CQ, DM .
1.91. ABCD, B = 60 . a
BC CD M N , CM + CN
. , AM N .
1.92. ABCD P .
A, B, C, D P B, P C,
P D, P A . , .
1.93. ABC O. , CO
ACB.
1.94. AB BC ABCD H K , KA = KB HC = CB. ,
KDH K, A, D, C, H
.
1.95. AB AC ABC
ABDE ACP Q. ,
1) AK ABC QE
1

AK = QE;
2
2) AM AQE BC
1

AM = BC.
2
1.96. AB ABC
M . , M C = M A + M B.
1.97. ABCD
AB1 C1 D1 A. , BB1 , CC1 ,
DD1 .
1.98. ,
. , M N . ,
M N 60 .
1.99. AB BC ABC
ABM N BCP Q. ,
AC M Q .
180


1.100. ,
A, B. ,
, A B.
1.101. ,
45 . .
1.102. , , ,
a.
1.103. ,
.
1.104. , , .
1.105. A B, , .
1.106. .
, ,
.
1.107. . , .
1.108. a A.
, M
.
1.109. ,
.


1.110. CD ABCD E. BAE BC F . ,
AE = ED + BF .
1.111. AB A1 B1 . O , A A1 B B1 , O1 ,
A B1 B A1 . , OO1
, . ,
180 .
1.112.
ABC A1 B1 C1 . A,
B, C A1 , B1 , C1 , B1 ,
181

C1 , A1 , C1 , A1 , B1 . ,
.
1.113. . , , ,
. , . ? ?
1.114. ABC P . ,
P A, P B, P C
. P
?
1.115. ABC ABC1 , BCA1 , CAB1 . ,
AA1 , BB1 , CC1 , M A1 = M B + M C, M B1 = M C + M A, M C1 = M A + M B.
1.116. , , , A, B, C, D
.
1.117. O A
B. A1 B1 , AA1
BB1 , A1 OB1 f.
1.118. ABCD S, BD. S P Q M N ,
, M (AB), P (BC), N (CD), Q (DA).
, SC M Q .
1.119. ABC C AA1 . AB
D , CD AA1 .
AD : DB.
1.120. ,
w1 (M, R) w2 (N, r), R > r.
1.121. H
A, B, C. , ,
A, B C .

7.
7.1. . A B.
M ZA (M ) = M1 ZB (M1 ) = M 0 (. 43).
182

M M1 M 0 :
M M 0 = 2AB. AB , A B .
ZB ZA 2AB. ,
M0

M
A

ZB ZA = T2AB .

(7.1)

Ta Tb . Ta (M ) = M1 Tb (M1 ) = M 0 , M M 1 = a

. 43
M1 M 0 = b, M M 0 = M M 1 + M1 M 0 =
=a
+ b. M , M M 0 ,
a
+ b:
M1

Tb Ta = Ta+b .

(7.2)

, : Tb Ta = Ta+b = Tb+a = Ta Tb .
,
.
Tr ZA (7.1): (ZB ZA ) ZA = T2AB ZA .
ZA ZA = E,
ZB = T2AB ZA . 2AB = r,
1
2

(7.3)

1
2

(7.4)

Tr ZA = ZB ,

AB = r.

ZB Tr = ZA ,

AB = r.

, A
r , B
1
AB = r.
2
, :
1) ,
2) .
7.2. .
Su Sv , u k v
(u 6= v), f = Sv Su . M Su (M ) = M1 , Sv (M1 ) = M 0 , f (M ) = M 0
183

(. 44). (M M1 ) u = P , (M1 M 0 ) v = Q,
M = M1 = P M1 = M 0 = Q . M M 0 = M M 1 +
+ M1 M 0 = 2P M 1 + 2M1 Q = 2(P M 1 + M1 Q) = 2P Q. 2P Q
M , u v:
, u
v , . ,
0
M
M
1
M
f = Sv Su . u = v
P
Q
T0 . u
v
.
. . 44
, ,
.
7.3. . . Tr
.
u r. Tr(M ) = M 0 Su (M ) = M1 . v M1 M 0 ( M1 = M 0
v M1 u (. 44). Sv (M1 ) = M 0
Sv Su M M 0 . M M 0 = r = 2P Q,
u v M , Tr.
M Tr Sv Su , (7 )
Tr = Sv Su ,

T(1/2)r (u) = v,

u r.

, Tr ,
, ,
: T(1/2)r
M0 v
B
.
M1
7.4.
. O
A u
a a/2
Su Sv u v, u v,
f = Sv Su . u v = O
M
(. 45), f (O) = O, O . 45
Su Sv . ,
f . M , O. Su (M ) = M1 ,
Sv (M1 ) = M 0 , (M M1 ) u = A, (M1 M 0 ) v = B, ]M OA = ]AOM1
184

]M1 OB = ]BOM 0 .
]M OM 0 = ]M OA + ]AOM1 + ]M1 OB + ]BOM 0 =
= 2(]AOM1 + ]M1 OB) = 2]AOB = 2](u, v).
, u v . , Sv Su
O 2](u, v). ,
.

.
7.5. . a
. O
. RO
a
u. M 6= O, RO (M ) = M 0 Su (M ) = M1 .
v M1 M 0 ( M1 = M 0
(OM1 ) = v). Sv (M1 ) = M 0 . OM = OM1 = OM 0 O v. Sv Su ,
a
Sv Su , RO
M . ,
a
a
RO
= Sv Su , u v = O, ](u, v) = .
2

,
, u v , : u v = O, ](u, v) = a/2.
a
= Sv
u1 v1 = O ](u1 , v1 ) = a/2 (. 46), RO
Su = Sv1 Su1 .
v1
, u1 , a/2
O
,
,
a/2
v
:
u
.
7.6. . A
. 46
b
a
B RA
RB
,
b
a
RB RA a + b.
, a + b = b + a, . , A 6= B.
. a + b = 0 a + b = 2p
, ,
b
a
(RB
RA
)(A) 6= A. b
a
, a + b 6= 0 a + b 6= 2p. RB RA

185

a + b,
a + b. C , a 6= 0
b 6= 0. C ,
b
a
RA
(C) = C1 RB
(C1 ) = C. C1
, AC = AC1 BC = BC1 .
u1
v1
, C C1 b
/2
AB (. 47.).
a/2
: CAB = a/2 ABC = b/2.
a/2 b/2 B
A
C u
v
a+b
b
a
RC
= RB
RA
: a/2
C
u v , RA
(u) = (AB)
b/2
RB (AB) = v.
. 47
.
b
a
RB
. RA
, a + b = 0 a + b = 2p, ,
a + b 6= 0 a + b 6= 2p. C a/2
b/2
u v , RA
(u) = (AB) RB
(AB) = v.
, :
b
a+b
a
RA
RB
= RC
,
1

C1 = SAB (C).

7.7. . Tr
a
. , ,
RO
a
a
RO
= Sv Su Tr = St Sv , u v = O, ](u, v) = t = T(1/2)r (v)
2
(. 48).
a
Tr RO
= (St Sv ) (Sv Su ) = St (Sv Sv ) Su = St Su .
a
a
u t (u, t) = (u, v) = , St Su = RA
, A = u t. ,

a/2

, a = 180 :
Tr Zo = ZA ,

O
r/2
u

a
a
Tr RO
= RA
.

a/2

1
2

OA = r.

:
v

a
a
Tr RO
= RO
Tr,
1

O1 = Tr(O).

, ,
, r =
0 a = 0.
7.8. . Sl Tr r k l r 6=
0. ,
. 48

186

:
Tr Sl = Sl Tr,

r k l.

, M Sl (M ) = M1 , Tr(M1 ) = M 0 ,
Tr(M ) = M0 , Sl (M0 ) = M 0 (. 49). (Tr Sl )(M ) = M 0
(Sl Tr)(M ) = M 0 .
M0
r
. - M

Q l
P
C
()
.
Tr Sl = Sl Tr (
r k l, M1
M0
r 6= 0) Wlr. l . 49
, r
.
.
1) ( ).
2) , ,
. , (M M1 ) l = P , (M M 0 ) l = C (. 49),
P M M1 C M M 0 .
3) r
P Q M M 0 l :
r = P Q.
7.9. .
Sl Tr r l. r r = p + q, p k l, q l
(. 50). Tr = Tq Tp Sl Tr =
= Sl (Tq Tp) = (Sl Tq) Tp. Tq =
r
= Sl Su , u l q
p
T(1/2)r , : Sl Tr = Sl (Sl Su ) Tp =
= (Sl Sl ) (Su Tp) = Wup, p k u. ,
l
q/2

Sl Tr = Wup.

7.10. . 50
. .
1)
.
2)
.
187

3) ,
.
4) ,
.

.

8.

1. A1 A2 A3 A4 A5 ,
P1 , P2 , P3 , P4 , P5 A1 A2 , A2 A3 , A3 A4 , A4 A5 , A5 A1
(. 51).
A3
. M2

f = ZP5 ZP4 ZP3 ZP2 ZP1


P3
P
2

.
A2
M3
M1
P4

(. 7.1). A4
A5 , f (A1 ) = A1 , . . A1
M4
P5
P1
. , M5
M
M5
A1
M f : M5 = f (M ).
A1 M M5 . . 51
A2 = ZP1 (A1 ), A3 = ZP2 (A2 ),
A4 = ZP3 (A3 ), A5 = ZP4 (A4 ).
2. M ABC BC, CA, AB M1 , M2 , M3 . ,
188

M1 , M2 , M3 ,
ABC.
. a1 , a2 , a3 , a (. 52), H ABC ( 1.33). . 7.4 SCA SBC =
2BCA
. a1 = RC
a2 M1 M2 , 5 6
M1 , M2 , H .
M2 , M3 , H. , M1 , M2 , M3 , H
.
3. CA CB ABC CAM N CBP Q O1 O2 . D
F M P N Q . , ABD O1 O2 F (. 53).
M1
B

a3

N
H

a1

M3

F
M

O1

O2

C
M2

a2

. 52

B
. 53

90
90
. RA
RB
.

180 , . . , P M . ,
D P M .
DAB DBA , . . 45 ,
ABD. O1 O2 F
O1 O2 90 .
4. A, B, C, D
90 . , AC
BD .
90
90
180
90
90
. RB
RA
= RO
RD
RC
=
180
180
180
= RO1 . RO1 RO ,

189

O1 = O. ]OAB = ]ABO = 45 ]OCD = ]CDO = 45 . ,


90
90
(C) = D. 90 AC RO
(A) = B RO
BD, .
5. BC, CA, AB ABC (
) BCA1 , CAB1 ,
ABC1 . , O1 , O2 , O3
B1
O2
A1 (. 54).
C
O1
.
120
120
120

RO
RO
RO
.
A
3
1
2
B
(. 7.6) ,
O3
, 360 . ,
C1
A .


. 54
( ).
120
120
120
120
120
RO
RO
RO
= E RO
RO
=
3
1
2
1
2

120
120
120
240
= RO3 , RO1 RO2 = RO3 . O3
O3 O2 O1 O2 O1 O3 , . . 60 . , O1 O2 O3 .

1.122. .
1.123. , , ( ),
. .
1.124. .
1.125. ABC. l, m, p BC, CA, AB .
, Sp Sm Sl ,
Sl Sp Sm , Sm Sl Sp .
1.126. , .
190

1.127. , ,
, .
1.128. , , , .
,
.
A
B
C
,
RA
RB
1.129. ABC. RC
ABC.
1.130. BC, CA, AB ABC BCA1 ,
CAB1 , ABC1 . , A1 , B1 ,
C1 60 . .
1.131. .
, , .
1.132. . ,
.
1.133. AC BC ABC
ACP BCQ. ,
M AB, P
O BCQ.
1.134.
. .
1.135. .
1.136. . , , , .
1.137. a b
A B.
a b, A B?
1.138. ABC A1 B1 C1 . , AA1 , BB1 , CC1 .
1.139. ABC A1 B1 C1 . ,

191

, , .
1.140. AB A1 B1 .
A A1 B B1 , A B1 B
A1 . , .
1.141.
,
?
1.142. , AB A1 B1 ?
1.143. ABCD, A = 45 .
DA
CD
BC
AB
W(CD)
W(BC)
W(AB)
.
W(DA)
1.144. ABCD, M . P Q
AB CD. , ZQ ZM ZP
.
1.145. , .

9.

f Oxy , . f (M ) = M 0 M
M 0 (x, y) (x0 , y 0 ),
x0 , y 0 x, y
, f .
9.1. . r(a, b)
. M (x, y) M 0 (x0 , y 0 ),
M M 0 = r, . . x0 x = a y 0 y = b. ,
x0 = x + a,

y0 = y + b

(9.1)

r(a, b).
S(x0 , y0 ) ZS
1

M (x, y) M 0 (x0 , y 0 ), (x0 + x) = x0 , (y 0 + y) = y0 ,


2
2
ZS :
x0 = 2x0 x,
192

y 0 = 2y0 y.

(9.2)

9.2. . O
, () f, x0 , y 0 M 0 M (x, y)
x, y f. Ox OM a (. 55), OM r.
x = r cos a, y = r sin a.
OM 0 r, Ox
y
a + f, M0
:
f
x0 = r cos(a + f), y 0 = r sin(a + f).
, :
x0 = r cos a cos f r sin a sin f,
y 0 = r sin a cos f + r cos a sin f.

M
x

. 55

r cos a r sin a x y, O f:
x0 = x cos f y sin f,
y 0 = x sin f + y cos f.

(9.3)

, f = 90 x0 = y, y 0 = x.
9.3. . ax + by + c = 0 M (x, y) M 0 (x0 , y 0 ).
x0 , y 0 x, y a, b, c. M M 0

x + x0 y + y 0
,
,
2
2
1
1
a(x + x0 ) + b(y + y 0 ) + c = 0.
2
2

, M 0 M , . .
p(b, a), :
b(x x0 ) + a(y y 0 ) = 0.
x0 , y 0 . :
 0
ax + by 0 = ax by 2c,
bx0 ay 0 = bx ay.
:

x0 = x

2a
(ax + by + c),
a2 + b2

y 0 = y 2b (ax + by + c).
a2 + b2

193

(9.4)

, y = kx,
(9.4) :

x0 =
((1 k 2 )x + 2ky),
1 + k2
(9.5)

y 0 = 1 (2kx (1 k 2 )y).
2
1+k

k = 1 x = y, y = x.
9.4. I II . ,

. ,
: ABC A0 B 0 C 0 (. 1.3). ABC O f,
A1 B1 C1
A0 B 0 C 0 .
A1 B1 C1 A0 B 0 C 0 A1 A0 .
:
x1 = x cos f y sin f,
y1 = x sin f + y cos f,
:
x0 = x1 + a,

y 0 = y1 + b.

:
x0 = x cos f y sin f + a,
y = x sin f + y cos f + b.

(9.6)

.
, ,

. Ox .
:
x1 = x, y1 = y

(M (x, y) M1 (x1 , y1 )).

M1 (x1 , y1 ) M 0 (x0 , y 0 ),
x0 = x1 cos f y1 sin f + a,
y 0 = x1 sin f + y1 cos f + b.
194

:
x0 = x cos f + y sin f + a,
y 0 = x sin f y cos f + b.

(9.7)

9.5. .
1. . , .
. ,
Ox (. 56).
A ABC a, b. yB B (9.3)
3
1
a b.
2
2
3
1

f = 120 yC = a b. 2
2

f = AOB = 120 : yB =

y B

1
1
2
2
+ yC2 = b2 + (a 3 b)2 + (a 3 b)2 =
+ yB
yA

4
4
3
3
= (a2 + b2 ) = R2 , R ,
2
2

O
C

A
ABC. .
. 56
2.
C(x0 , y0 ) f.
f
. RC
,
(9.6):

x0 = x cos f y sin f + a,

y 0 = x sin f + y cos f + b.

a b. C(x0 , y0 ) C(x0 , y0 ),
x0 = x0 cos f y0 sin f + a,

y0 = x0 sin f + y0 cos f + b.

a b . :
x0 = (x x0 ) cos f (y y0 ) sin f + x0 ,
y 0 = (x x0 ) sin f + (y y0 ) cos f + y0 .
f
.
RC
3. Ax + By + C = 0
ax + by + c = 0.

195

. (9.4).
.
x = x0

2a
(ax0 + by 0 + c),
a2 + b2

y = y0

2b
(ax0 + by 0 + c).
a2 + b2

Ax + By + C = O
x, y .
Ax + By + C = 0 ax + by + c = 0:
(a2 + b2 )(Ax + By + C) = 2(aA + bB)(ax + by + c).
x0 , y 0 .
4. , ABC, x + 2y 13 = 0
x y 5 = 0 l m, , A(7, 8).
. , A . B l, C
m. l AB BC,
A0 , A l, BC.
A1 , A m,
BC. , A1 A0 ,
(9.4): A1 (13, 2), A0 (3, 0). BC:
x 5y 3 = 0. , 3 AB AC: 23x + 11y 249 = 0 5x y 27 = 0.

1.146. ,

, 1

: x0 = x +
y, y 0 =
x y.
2
2
2
2
1.147. ?

x0 = 1 (x 3y),
x0 = 1 (x + 3y),
2
2
)
)
y 0 = 1 (3x + y);
y 0 = 1 (3x y);
2
2

1
0
0
x = (x + 3y),
x = 1 (x + 3y),
2
2
)
)
y 0 = 1 (3x + y);
y 0 = 1 (3x + y).
2

1.148. ,
A(1, 1) B(3, 2) A1 (4, 2) B1 (1, 4).
196

1.149. , Ax + By + C = 0 Bx + Ay + C = 0
y = x.
1.150. ax + by + c = 0
r(b, a).
1.151. Ax + By + C = 0 S(x0 , y0 ).
1.152. , Ax + By + C = 0
S(x0 , y0 ) f :
(x x0 )(A cos f B sin f) + (y y0 )(A sin f + B cos f) + Ax0 + By0 + C = 0.
1.153. ABCD .
AA1 , BB1 , CC1 , DD1 . ,
AA1 CC1 = BB1 DD1 .
1.154. C ABC l
A B , l A1 B1 . , CA21 + CB12
l AA1 ( ABC).

10.

10.1. . . () .
, .
, , . f f (A) = A1 , f (B) = B1 . f AB
A1 B1 , , , .
, r1 = A1 B 1 r = AB f . ,
AB, r:
197

.
(. 57), (. 58) . .
,
.
, , .
r1
A1

B1

A
l

A
r

A1

r1
B1

. 57

. 58

.
1. ,
, ,
:
f (x
a + yb + z
c) = xf (
a) + yf (b) + zf (
c).
f (
a + b) = f (
a) + f (b) f (x
a) = xf (
a). . , ,
.
, .
2. , , .
a
a
r RA
(
r) = r1 , RB
(
r) = r2 . |
r1 | = |
r|
|
r2 | = |
r|, |
r1 | = |
r2 |. , (
r, r1 ) = a (
r, r2 ) = a,
r1 r2 . , r1 = r2 .
10.2. .
1. OAB, OCD, OEF . , M , N , P BC, DE, AF
.
. BEDC :
1
2

1
2

M N = (CD + BE) = (CD + OE OB)


198

(. 59). 60 , ,
: CD OD, OE F E, OB OA. 1 M N
1
1
E
(OD + F E OA) = (AD + F E) =
2

= P N . ,
M N P .
2. AB, BC, CA
ABC ABC1 , BCA1 ,
CAB1 . ,
.
. O1 , O2 , O3
BCA1 , CAB1 , ABC1 (. 54).

N
D
O
F
C

P
A

M
B
. 59

1
3

, : O1 O2 = (A1 A+BB 1 +CC) =


1
3

1
3

= (A1 A+BB 1 ) O3 O1 = (AA1 + C1 C). A1 A =


= A1 B + BA. A1 B BA 120
BC C1 B. , A1 A
BC + C1 B =
= BC BC 1 = C1 C. ,
BB1 AA1
O3
1
C
O1 O2 = (A1 A + BB 1 ) 3

P
O4

O2

1
(C C + AA1 ) =
3 1

= O3 O1 , .
3. A
B
. , O1
, .
. O1 , O2 , O3 , O4
. 60
,
AB, BC, CD, DA ABCD, ,
M , N , P , Q (. 60).
M

1
2

O1 O3 = O1 M + M P + P O3 = O1 M + (AD + BC) + P O3 .
199

1
2

90 : O1 M M A = BA,

1
AD QO4 ,
2

1
1
BC O2 N , P O3 CD. , O1 O3
2
2

1
1
1
BA + QO4 + O2 N + CD = (BA + CD) + QO4 + O2 N =
2
2
2

= N Q + QO4 + O2 N = N O4 N O2 = O2 O4 .
, O1 O3 O2 O4 |O1 O3 | = |O2 O4 |.

1.155. AB BC ABC
ABM N BCP Q. , M Q
BE ABC .
1.156. .
, .
1.157. AB BC ABC
ABC1 BCA1 . M , N , P AC, BC1 , BA1 . ,
M N P .
1.158. AC BC ABC
ACB1 BCA1 . OM A1 , M AB, O
ACB1 .
1.159. A1 B1 C1 D1 A2 B2 C2 D2 . A1 A2 , B1 B2 , C1 C2 , D1 D2 A, B, C, D
. , ABCD ( A, B, C, D ).
.
1.160. ABCD O
90 A1 B1 C1 D1 . P , Q, R, S
A1 B, B1 C, C1 D, D1 A . , P R QS
.
1.161. AC BC ABC ACM N BCP Q. , AQ BN , ,
AB.
90
1.162. ABC D = RA
(C)
90
E = RB (C). , M DE
AB

1
AB.
2

200

1.163. O n- A1 . . . An .
,
sin A1 OA1 + sin A2 OA2 + . . . + sin An OAn = 0.
1.164. AA1 , BB1 , CC1 ABC BC = a,
CA = b, AB = c, a2 AA1 + b2 BB 1 + c2 CC 1 = 0. .

11.

11.1. . y = f (x), y = f (x + a), y = f (x) + b, y = f (x + a) + b . , (9.1),

n x0 = x a,
n x0 = x,
n x0 = x a,
1)
2)
3)
0
0
y = y;
y = y + b;
y 0 = y + b,
M (x, y)
y = f (x) M 0 (x0 , y 0 ). :
n x = x0 + a,
n x = x0 ,
n x = x0 + a,
1)
2)
3)
0
0
y=y ;
y = y b;
y = y 0 b.
,
(a, 0), (0, b), (a, b) y = f (x) y 0 = f (x0 + a), y 0 b = f (x0 ), y 0 b = f (x0 + a).
( ), : y = f (x + a), y = f (x) + b,
y = f (x + a) + b.
, y = f (x + a)
y = f (x) (a, 0), . . Ox |a|
, a (. 61). y = f (x) + b y = f (x) (0, b),
. . Oy |b| , b (. 62). y = f (x + a) + b
y = f (x) , . .
(a, b) (. 63).

-
y=

ax + b
,
x+c

201

(11.1)

y
y
b
y = f (x + a)

y = f (x) + b

y = f (x)

b>0

a>0

y = f (x)
. 61

. 62
y

y = f (x + a) + b
r
y = f (x)
x
O
r = (a, b)
. 63

b ac

+ a. ,
: y =
x+c
b ac = k 6= 0. ,
k

y = ,
x
.
,
r(c, a).

3
y=

3x + 5
x+2

y=
O
2

. 64

1
x

y =
+ a,
x+c
. . -
(11.1). . 64
x

3x + 5
, x+2
1
y = x

y =

r(2, 3).
,

y =

ax + b
x+c

(c, a) ,
.
11.2.
y = f (x) y = f (x), y = f (x),
202

y = |f (x)|, y = f (|x|) , y = f (x) (


).
n x0 = x,
n x0 = x,

0
y = y
y0 = y
Ox Oy .
y = f (x)
y = f (x) y = f (x). , y = f (x) y = f (x) Ox (. 65), y = f (x) y = f (x)
Oy (. 66).

, y = f (x) y = f (x) (. 67).
y

y=f (x)
x

y
y=f (x)

y=f (x)

O
O

x
y=f (x)

y=f (x)
. 65

y=f (x)

. 66

. 67

y = |f (x)| y = f (x) , f (x) > 0,


y = f (x), f (x) 6 0. y = |f (x)| y = f (x),
( ) Ox, ,
( ) Ox, Ox. . 68 y = |x2 4x + 3|
y = x2 4x + 3.
y = f (|x|)
y = f (x) x > 0 y = f (x) x 6 0.
y = f (|x|) y = f (x),
Oy, Oy.
y = f (x), Oy, . . 69
y = x2 4|x| + 3.
y = |f (|x|)| y = f (x) ( )
. . 70 y = |x2 4|x| + 3|.
y = f (x)
x,
203

y
3

y
3

y = |x2 4x + 3|

1
1

1
y = x2 4|x| + 3

1
. 68

. 69
y
3

1
x

O
3

y = |x2 4|x| + 3|
. 70

. x y
x = f (y), y
x: y = g(x).
(9.5) y = x
: x0 = y, y 0 = x.
y = x y = f (x) x = f (y). , ,
.
3x + 5
,
y =
x+2
. 64.
,
(, 3) (3, +) (, 2) (2, +).
x =
204

3y + 5
5 2x
, y =
.
y+2
x3

y=

3x + 5
5 2x
y=

x+2
x3

y = x (. 71).

y=

3x + 5
x+2

O
2

2
y=

5x 2
x3

. 71

1.165. y = log3 x. :
y = log 1 x,
3

y = log3 (x),

y = log3 |x| + 1,

y = log3 (x), y = log3 |x|, y = log3 (1 x),


y = log3 x, y = log3 (|x| + 1), y = 3x .
4x + 3

1.166. y =

2x 5
.
1.167. :
x3 x2 + 2x = 1,

x2 =

1
.
x+2

1.168. , 3x2 3x 4 = 0
[1; 2].
205

II


12.
12.1. .
O k 6= 0. X X 0 ,
OX 0 = k OX . X 6= O, OX OX 0 k > 0
k < 0. , k 6= 0 O,
X, X 0 X 0 X . , ,
, O, ,
() . , k OX = OX 0
.
. O k 6= 0
, X X 0 , OX 0 = k OX.
O k HOk . ,

HOk (X) = X 0 OX 0 = k OX.
, HOk (O) = O, k OO = OO. ,
.
k = 1 OX 0 = OX , X 0 = X X,
. . 1 :
HO1 = E. k = 1, OX 0 = OX O XX 0 .
, 1 : HO(1) = ZO .
k 6= 1, O HOk
. , P ,
OP = k OP , (1 k) OP =
0. 1 k 6= 0, OP = 0
P = O.

, HOk , OX =
1
= OX 0 O
k
1
: (HOk )1 = HO1/k .
k

12.2. . .
. A0 B 0 A B HOk ,
0
A B 0 = k AB.
. OA0 = k OA OB 0 = k OB, OB 0 OA0 = k OB k OA = k (OB OA), A0 B 0 = k AB.

|A0 B 0 | = |k||AB|. , , |k| =
6 1, , . . .
|k| = 1 . ,
. , , k 6= 1.
. .
. HOk a. A a, B a (A 6= B) HOk (A) = A0 , HOk (B) = B 0 (. 72).
A0 B 0 = k AB. X a,
AX = t AB, t (). a
t, . .
A0
. A
AX = t AB O
X X0
a, A B. , t = 0
B0
B
A, t = 1 B.
HOk (X) = X 0 ,
0
A X 0 = k AX. AX =
. 72
= t AB AX AB

1 0 0
1
1
1
A X A0 B 0 , : A0 X 0 = t A0 B 0 ,
k
k
k
k

A0 X 0 = t AB. a
HOk . , a0 , A0 B 0 .
X X 0
t ( ), a
a0 .
207

O a, a0 k a,
A0 B 0 k AB. O a, a0 = a, O, X, X 0
X a. , .
1. , , .
2. , ,
.
12.3. , . X X 0 t AX = t AB
A0 X 0 = t A0 B 0 a a0 , : a a0 ,
t. .
1) . ,
, ,
.
2) .
3) .
12.4. . . .
. A0 A: HOk (A) = A0 . OA0

k
,
OA
. , O
A A0 . X 0 X. , X 6 (OA) (. 72).
X 0 (OX) (A0 X 0 ) k (AX) ( 1), X 0
OX , A0
AX. Y (OA), Y 0 X X 0 (. 73): (X 0 Y 0 ) k (XY ), Y 0 (OA).
X
O
Y

A0

Y0

A
X0

. 73

208


A A0 , B B 0 , : A0 B 0 k AB, A0 B 0 6= AB
(. 74). k =

A0 B 0
, O . AB

A, A0 , B, B 0 , O = (AA0 ) (BB 0 ).
(. 75), M 0 M
/ (AB) ,
A0 B 0 M A M B.
O = (AA0 ) (M M 0 ).
B

M
B0

M0
O

A0

. 74

A0

B0

. 75

13.
13.1. . 1
, , 1 .
, AB A1 B1 , ,
AB A1 B1 .
B1
A A1 B B1 , B
A B1 B A1 .
k k
O1
(. 76). , AB A1 B1 - O
. AB = A1 B 1 ,
A
A1
AB A1 B1 ,
. 76
: A B1 , B A1 .
A1
ABC
A
A1 B1 C1 B1
O
B
. ,
A A1 , B B1 , C
C1

.
77
, C C1 (. 77).
,
. , HO1 () = E() = ().
209

HOk () = 1 , HO1/k (1 ) = (). , ,


.
ABC HOk (4ABC) = 4A1 B1 C1 (. 78). HO1/k
(4A1 B1 C1 ) = 4A0 B 0 C 0 , ABC A0 B 0 C 0 .
1

. , A
A0
.
A1
C
C0

C1
( 15).
B
13.2. 1
O1
O
. B
B0
,
. 78

. : w w1 ,
, ?
. .
. w w1 O
r r1 , O r1
r1
k1 =
k2 = , w w1 .
r
r
, O1 6= O r1 6= r. AB A1 B1 (. 79). H
A
A1
O1
O

w1

S1

B1
B
. 79

O O1 , A A1 . , H O A w O1 A1 w1 ,
H(w) = w1 ( ). H S = (OO1 ) (AA1 ),
210

r1

O1 A1
, HSk (B) = B1 . O1 A1  OA,
OA
r1
OA, k = .
r

k = ; O1 A1
r
, O O1
A B1 , w w1 .
S1 = (OO1 ) (AB1 ),
, HSk
(B) = A1 .
1
, HSk (w) = w1 HSk
(w) = w1 , k =
1

O1 B 1 O1 A1
=
= k.
OB
OA

O1 A1
, S = (OO1 ) (AA1 ),
OA


S1 = (OO1 ) (AB1 ). , HSk HSk
1
AB A1 B1 ,
w w1 . ,
, w w1 , .
w w1 S,
S. , , O O1 , S S1 , w w1 S,
.
,
.

14.
1. , , , , , (C
).
. P1 , P2 , P3 - M1
M3

B
P3
M , P1
M2
P2
ABC, BC,
M
CA, AB (. 80). M1 , M2 , M3 , M A
. 8 ( 2,
. 80
. 52) M1 , M2 , M3
1/2
. HM
P1 , P2 , P3 , , M1 M2 .
211

2. , ABC
G, H O
, GH = 2OG.
. ABC A1 B1 C1
G : A A1 , B B1 , C C1
1

G k = (. 81). 2
. OA1 , OB1 , OC1
A1 B1 C1 .
, AH, BH, CH ABC OA1 , OB1 , OC1 A1 B1 C1 ,
, H
C
ABC
O A1 B1 C1 . H
B1
A1
O O

G H

1
2

G GO = GH , -

GH = 2OG.
, G,
A
C1
B
H O . 81
, .
C
3. , ,
H2
,
K3
,
B1
A1

OH ,
O E
H1

H
.
K1
K2
. A
H3
B
C1
w,
H1 , H2 , H3 ABC
(.82). ,
. 82
H , ( 1.32),
w -

1
2
R
OH, . K1 , K2 , K3 2

H k = .

212

ABC w. ,
H A, B, C. , ,
( 1.11).
A1 , B1 , C1 w.
. ,
( ).
4. A, B, C,
l. l M N ,
(AM ) k (BN ) (CM ) (CN ).
. , (AB) l. M N . (AB) l = O,
O, A B, M N (. 83). C1 = HOAB (C), (CM ) k (C1 N ) (CN ) (C1 N ).
, N g
CC1 , , N (g l).
N . M O
(AM ) k (BN ).
g l. . 83
M N , M1 N1 . (, B
A, .) AB k l M1
A
TAB .
M
N1
- B
C
.
, , .
g
N
, C1 l
,
.
. 83
.
5. .
. , . AB1 C1
A B (. 84). , A,
AB1 C1 ABC. 213

, A, AP p ( ) AP1 AP1 = AB1 + B1 C + C1 A.


P P1 p
P1 A. , P
C1
B C ABC:
C
B = HAP1 P (B1 ), C = HAP1 P (C1 ).
.
A
B B1
,

. 84
, . .
6. , a b.
. a O a b = A
(. 85). a . a b
a1 b1 , a
a . a1 b1 = A1 AA1
. C
A
, b

S
S T AA1
a ( ). C b1
T

a
O
A1
a b SO ( T O).
a, a b, a1
, . 85
AA1 , , ,
a . , 8.
4. . 85 ,
.

, O a
a b.
214


2.01. , .
2.02. , A, B
C, D, A B , C D
. , (AB) k (CD).
2.03. .
,
.
2.04. ABC. A1 , B1 , C1 A, B,
C P .
, , A1 , B1 , C1
BC, CA, AB , .
2.05. ABC, A B , C .
2.06. , ,
. .
2.07. , , ,
, .
2.08. , . , ,
,
.
2.09. ABCD AB CD O. , ,
AOB COD, .
2.10. A,
B C. ,
BAC .
2.11. ( ) . ( ) ,
, , A B. , AB .
2.12. A. M N P .
215

, AP M AN .
.
2.13. A. M , N , P , Q,
. , M AP N AQ .
2.14. . M , N , P , Q . , M Q N P , 180 .
2.15. ,
.
2.16. P A1 , B1 , C1 BC, CA, AB ABC. , , A, B, C
P A1 , P B1 , P C1 , Q QG = 2GP , G
ABC.


2.17. M N m
n ,
.
2.18. A
, B C (B A C) , AB = 2BC.
2.19. ,
,
.
2.20. ,

.
2.21. , .
2.22. l ,
a M .
2.23. ,
.
2.24. ,
, . .
216

2.25. ,
.
2.26. ABC A1 B1 C1 .
A2 B2 C2 , (A2 B2 ) k (A1 B1 ), (B2 C2 ) k (B1 C1 ), (C2 A2 ) k (C1 A1 )
A2 BC, B2 (AC), C2 (AB).
2.27. ,
.
2.28. , .
2.29. , .
2.30. , .


2.31. a A , b B
(a 6= b A 6= B). ,
(a, A) (b, B).
2.32. B l A C. l1 , l, A1 , B1 , C1 , B1
A1 C1 A1 B1 : B1 C1 = AB : BC. A, B, C
a, b, c , A1 , B1 , C1
a1 , b1 , c1 , .
, M = a a1 , N = b b1 , P = c c1 .
2.33. ABCD A1 , B1 , C1 ,
D1 BCD, CDA, DAB, ABC . ,
A1 B1 C1 D1 , .
2.34. A1 , B1 , C1 M
A, B, C ABC . M M1 , M1 A1 B1 C1 .
2.35. A, B, C. M G ABCM (
). M G. (
.)
2.36. , . ?
2.37. ABC A1 B1 C1 .
AA1 , BB1 , CC1 A2 , B2 , C2 k.
217

S2 A2 B2 C2 ,
S S1 .
2.38. S ,
a, b, c, M , A, B, C,
AB : BC m n .
2.39. a, b, c . ,
a, b, c A, B, C,
AB : BC m n .
2.40. AB AC ABC M
N , BM = M N = N C.
2.41. ABCD, M . , M AB, M BC, M CD, M DA .
2.42. ,
.
2.43. A1 B1 C1 , ABC.
2.44. ABC. S
M N , M N CA
CB.
2.45. ABC
AB M . M1 CM1 , AB N . ,
AN = BM .

15.
15.1. .
HAa HAb A a b. X : AX 1 = aAX AX 0 = bAX 1 .
C
B A0
A
, AX 0 = (ab)AX. ,
X
X0
,
X1
. ab = 1 . 86
. .
HAa
b
HB A B. X
HAa (X) = X1 HBb (X1 ) = X 0 (. 86), . . AX 1 = aAX BX 0 = bBX 1 .
218

HBb (A) = A0 , BA0 = bBA. A0 X 0 AX


, A0 X 0 k AX. , C = (AA0 ) (XX 0 ).
AXC A0 X 0 C :
A0 X 0 = bAX 1 = (ba)AX.

A0 X 0
CX 0
CA0
. ,
=
=
AX
CX
CA

A0 X 0
= ab CX 0 = (ab)CX.
AX

CA0 = (ab)CA, C X. , C:
HBb HAa = HCab ,

C (AB).

A0 X 0 = AX, C , ab = 1 XX 0 = AA0 .
.
HBb HAa , A
0
A , A HAa HBb A0 .
.
. A B a b ab 6= 1.
, AB. ab = 1,
A 6= B A = B. .
.
,
ab 6= 1 ab = 1.
15.2. . A, B, C, A1 , B1 , C1
, A, B, C , A1 , B1 , C1
(AB1 ) k (A1 B), (BC1 ) k (B1 C), (AC1 ) k (A1 C).
. (AB) = m, (A1 B1 ) = m1 .
C m C1 m1 . m m1 m m1 = O (. 87).
C
m
B

A
O
C1

m1
B1 A1

m1
C1

. 87

B1
. 88

219

A1

: HOa (A) = B HOb (C1 ) = B1 . ,


, (HOb HOa )(A) = C (HOa HOb )(C1 ) = A1 . ,
A C C1 A1
HOab , (AC1 ) k (CA1 ).
m k m1 (m 6= m1 ), (AB) k (B1 A1 ) (AB1 ) k (BA1 ) AB = B1 A1 (. 88). BC = C1 B 1 .
AB + BC = AC C1 B 1 + B1 A1 = C1 A1 AC = C1 A1 , (AC1 ) k (CA1 ).
15.3. .
(. 13.2).
, , , ,
. .
. , ,
, .
. w1 , w2 , w3 O1 , O2 , O3 O3
/ (O1 O2 ). 0
wi wj Sij Sij
, (i, j = 1, 2, 3; i 6= j)
0 H S 0 = HS 0
(. 89). HSij HSiu = HSju HSij
(i, j, u = 1, 2, 3
iu
ju

S23

S13
0
S13

O3

w3
0
S23

O2
O1

w1

0
S12

w2

. 89

220

S12

), Sju (Sij Siu )


0
0
0
0
0
0
Sju
(Sij
Siu
). , S12 , S12
, S23 , S23
, S13 , S13
: S12 , S23 , S12 ; S12 ,
0
0
0
0
0
0
S23
, S13
; S12
, S23 , S13
; S12
, S23
, S13 ,
.
15.4. . BC, CA, AB, ABC, A1 , B1 , C1 . ,
, ,
AB 1 CA1 BC 1
= 1.

B1 C A1 B C1 A

(1)

. A1 , B1 , C1 (. 90).

A1 C
B1 A
C1 B
= k1 ,
= k2 ,
= k3 . HAk11 (B) = C, HBk21 (C) = A
A1 B
B1 C
C1 A

(HBk21 HAk11 )(B) = A. , k1 k2 6= 1,


k1 k2 = 1

A1 C
B1 C
CA1
CB 1
=
,
=
, . . A1 B1
A1 B
B1 A
A1 B
B1 A

CB CA
, C
(A1 B1 ) k (AB), A1
B1
. HAk11 HBk21 - C1
k1 k2 B
A
(AB) (A1 B1 ) = C1 , HCk11k2 (B) = A.
. 90
, HCk31 (B) = A . C1
B A , HCk11k2 = HCk31 , k1 k2 = k3 ,
,

A1 C B1 A
C1 B

=
.
A1 B B1 C
C1 A

(1).
, (1)
A1 , B1 , C1 .
, : HAk11 (B) = C, HBk21 (C) = A
(HBk21 HAk11 )(B) = A. , k1 k2 6= 1,
k1 k2 = 1 (1), k1 k2 = k3 ,
k3 = 1, . . C1 A = C1 B, A = B, . , HBk21 HAk11 k1 k2 = k3 .
, HCk31 (B) = A. k1 k2 = k3 B A, HBk21 HAk11 = HCk31 ,
A1 , B1 , C1 .
221

16.

1. ABCD AB CD. P BC, B C,


D M
D
C
AB. , (P D) (AB) = X,
Q
(P M ) (AC) = Q, (DQ) (AB) = Y , X
M XY
B
M
A Y
(. 91).
P
. HQ ,
A C, HP , . 91
C B. A B
S
(AB) (P Q) = M . M
AB, ZM M . , HQ (Y ) = D
HP (D) = X. , ZM (Y ) = X, y
M X = M Y .
2. , M
z
C
,
, a O
x , P .
A
Z
. u
E S1
X
x a
D
a1 ( O O1 )
B
Y
A B, P (AB) (. 92).

A, U

a x, a1
O1
B, x a1 . a
N
a1
F
S = (AB) (OO1 ). , AB P
P S, S
a a1
. 92
(. 13.2). X x

OA O1 B ( , SP
a a1 ). a a1 ,
222

S S1 , a a1 . , SP S1 P
. : x y S
( A, B M , N SP
) z u S1
( C, D E, F
S1 P ).
a a1 , ,
,
, OO1 .
3. a A, B,
C, D. a M N , (AM ) k (BN )
(DM ) k (CN ).
. M N (AB) a = S,
(CD) a = T (. 93), S 6= T . HSk (A) = B
HTl (D) = C. HSk (M ) = N HTl (M ) = N
HT1/l HSk M . k 6= l, M
, HT1/l HSk .
. , HT1/l (C) = D, P = HT1/l (B). A P
M = (AP ) (ST ). N .
(AB) k a (CD) k a, HSk HTl AB DC. k = l
HT1/l HSk ,
, B
. C
,

TC
SB
=

SA
TD

D
. P
a
, S = T , S
T
M
N
.
AB = DC = r r k a,
. 93
TAB TCD
M a. a,
. . 93. HTl ( TDC )
.

223


2.46. A B
AB?
2.47. Tr HOk ; HOk Tr.
2.48.
ABC ,
?
2.49. M N , P Q, RS. , M P N Q, M R N S, P R
QS , M Q N P , QR P S,
M R N S .
2.50.
.
2.51. , . .
2.52. M N AB CD ABCD, Q AC, (QM ) (BC) =
= P . , M N N Q N P .
2.53. M , AB
ABCD, M P AC (P (BC)),
B BN M D (N (DC)). ,
D, Q = (AC) (BN ), P .
2.54. BC, CA, AB ABC A1 , B1 , C1 , AA1 , BB1 , CC1 ( ). ,
HA1 (B) = C, HB1 (C) = A. HC1 (A) = B
. .
2.55. ABCDE. m AB,
BC, CD, DE, EA M , N , P , Q, S . ,
a
HS HQd HPg HNb HM
, a =

NC
PD
QE
SA
MB
, b=
, g=
, d=
, =
.
MA
NB
PC
QD
SE

2.56. w1 w2 O1 O2 w A1 A2 . M w
M A1 M A2 , w1 w2 B1 B2 .
, A1 A2 , B1 B2 , O1 O2
.
2.57. ABC Q. M , N , P BC, CA, AB .
224

QM , QN , QP m, n, p BC, CA, AB. , ,


m, n, p,
ABC.

17.
17.1. . f ,
k: f(A) = A1
f(B) = B1 , A1 B1 = k AB ( A, B). , AB > 0 A1 B1 > 0. ,
f .
. , , A B
A1 B1
A1 B1 = k AB, k () ,
.
, k = 1 .
k (k 6= 0) |k|,
(. 12.2) A1 B 1 = k AB, A1 B1 = |k|AB.
k1 k2
k1 k2 . ,
k |k|.
, , 1
k, :
k

1
AB = A1 B1 .
k

1 (: 1 ),
, 1 .
, .
, () .
. ,
. .
.
: ,
.
225

17.2. .
.
.
. ,
, .
. fk k,
fk HO1/(k) HO1/(k) O
f : fk HO1/(k) = f ,
(k

1
= 1). k

(fk HO1/(k) ) HOk = f HOk . , HO1/(k) HOk


, :

fk = f HOk .

(17.1)

HO1/(k) fk = g,
g 6= f , :

fk = HOk g.

(17.2)

, f = H f g ,
fk = f = g HOk (k = 1) .
(17.1) (17.2) , , , .
. . 1.2 . 12.2
12.3, :
1) , ,
2) ( ),
3) , ,
4) ,
5) , . . .
: AB CD , A1 B1 C1 D1
, A1 B1 = k AB C1 D1 = k CD, A1 B1 : C1 D1 =
= AB : CD.
, ( ), , , , , a , ,
.
k

k
O

226

18.
18.1. . A, B, C A1 , B1 , C1 ,
A1 B1 = k AB, B1 C1 = k BC, C1 A1 = k CA,
,
A, B, C A1 , B1 , C1 .
. HOk O. HOk (A) = A0 , HOk (B) = B0 , HOk (C) = C0 ,
A0 , B0 , C0 A0 B0 = k AB, B0 C0 = k BC, C0 A0 = k CA.
: A0 B0 = A1 B1 , B0 C0 = B1 C1 ,
C0 A0 = C1 A1 . (. 1.3) f , f (A0 ) = A1 , f (B0 ) = B1 ,
f (C0 ) = C1 . f HOk = fk , A A1 , B B1 C C1 .
.
. f y
, f(A) = A1 , f(B) = B1 , f(C) = C1 y(A) = A1 , y(B) = B1 ,
y(C) = C1 . ,
. y1 f ,
A, B, C . . 1.3 y1 f
: y1 f = E, y = f.
: ,
.
,
, .
18.2. . .
, ABC A1 B1 C1
, .
ABC A1 B1 C1 ,
. , : ,
.
, , . , , .
.
227

, : A A1 , B B1 .
M M1 .
M
/ (AB),
A1 B1 A1 B1 M1 , ABM
ABM .
M (AB), M1 (A1 B1 ),

A1 M 1
AM
=
= l.
B1 M 1
BM

A1 B1 l. : N1
N
/ (AB) , N N1 , M1 M (AB).
, ,
, .


, . 17.2 :
1)
,
2)
.

19.
19.1. ( ) .
, , ( 2.47).
.
0
M0 M

:
a
O

M1

a
a
RO
HOk = HOk RO
,

. 94

. a
, HOk (M ) = M1 RO
(M ) = M0 ,
M M 0 (. 94).
228

( ).
,
.
. , , .
. f a
, k 6= 1, a 6= 0, a 6= p,
HBk RA
f , . A = B ,
A 6= B. HBk (A) = A0 , f(A) = A0 . B0 B
a
a
(B0 ) = B, f(B) = B. , , . . RA
RA
f : A A0 , B0 B (. 95).
w,
A A0 AB0
w
A. ,

A
A, A0 , B0 . O
A0 a
, a
a
A1
A, B, B0 .
A O.
,
|a| < p. ]AOA1 = ]B0 AB = ]B0 OB = a.
f(AB0 ) = A0 B, ,

A0 B
= k. AB 0

a
a
HOk RO
. RO
(A) =
a
0
= A1 RO (B0 ) = B1 , A1 (OA ), B1 (OB),
A1 B1 = AB0 ]AB0 O = ]A1 B1 O. , , ]AB0 O = ]ABO ]ABO =

B0

B1

. 95

= ]A1 B1 O. , A1 B1 k A0 B

AB
= k, A1 B1

, HOk (A1 ) = A0 HOk (B1 ) = B. , a


HOk RO
A1 A0 B0 B,
f. ,
a
a
f = HBk RA
= HOk RO
,

.
, ,
( ), .
a = 0 k = 1.
229


. ,
a
p
f = HOk RO
, , E = HO(1) HO(1) = HO(1) RO
:
a
p
a
p+a
f = HOk E RO
= (HOk HO(1) ) (RO
RO
) = HOk RO
.
p+a
a
= HOk RO
.
, f = HOk RO
, , ,
, : k k
k. .
19.2. .
. ,

. .
, , :

Sl HOk = HOk Sl ,
k
O

O l.

, H (M ) = M1 Sl (M ) = M0 , M M 0
(. 96).
M0
,
M0
, O
l
.

M
.
M1
.
, , . 96
.
. f , , f = HOk Sl
(k 6= 1). O l . , O
/ l,
O m l. m l = A, Sl (O) = O1 ,
230

HOk (O1 ) = O0 , HOk (A) = A0 (. 97). f(O) = O0 f(A) = A0 . HOk HA(1) = HTk . A
A0 , O O0 . T
m
. k
O
HT Sm
A A0 , O O0 f. , f = HOk Sl = HTk Sm ,
A0
l
T m.
A
P0
P
O0
O1
.
, f = HTk Sm , T m,
, E = HT(1) HT(1) = HT(1) ZT =
= HT(1) St Sm , t m t m = T , . 97
: f = HTk Sm = HTk E Sm = (HTk HT(1) )
(St Sm Sm ) = HTk Sl . ,
O

f = HTk St = HTk Sm ,

t m t m = T .

M0

- M0
M
T
l
f = HOk Sl = HTk Sm . A
M Sl (M ) = M1 , HOk (M1 ) = M 0
M1
m
Sm (M ) = M0 . (M0 M 0 ) m = T ,
k
0
HT (M0 ) = M (. 98).
. 98

(. 7.10)
. .

231

20. ,
20.1. . .
, , ,
.
,
: . ,

. ,
0 180 .
.

.
.
.
f , l m
, a
Sl (m) f(m) m
l f(l).
. f = y Sl , y .
f(m) = (y Sl )(m) = y(m1 ), m1 = Sl (m).
, a m1 y(m1 ) y
, . . a Sl (m) f(m)
f l. ,
m = l Sl (l) = l a = ](l, f(l)).
, ,
a,
l. .
20.2. . 19 ,
, , . k 6= 1 ,
, , A B, AB = k AB,
k = 1.
. (,
), ( , ).
.
232

. 19.1
(. 95). . 19.2
, .

Q
C
N
D

C1
.
D1
O
f
B1
A A1 B B1 .
A1
M
B
AB A1 B1
A
ABCD A1 B1 C1 D1 (. 99). (AB) (A1 B1 ) = M ,
P
(CD) (C1 D1 ) = N , (BC) (B1 C1 ) =
= P , (AD) (A1 D1 ) = Q, O =
= (M N ) (P Q) . 99
.
20.3. .
O A B A1 B1 .
OA1

OB1

=
= k ]AOA1 = ]BOB1 =

OA
OB
= f . AOB A1 OB1 ,
OB

OB1

]AOB = ]A1 OB1 = y


=
= k1 . , B B1
OA
OA1
A A1 O,
k1 y. .
O A A1 B B1 ,
O, A B
A1 B1 .
20.4. . ,
, . , , . , . 19.2
m t, , , . k 6= 1
,
HTk ( HTk ) St ( Sm )
m t.
, m t

233

, . .
,
.

21.
1. ABC, C
CD. , AM CN
ADC DBC .
. CDB ADC (C A,
D D, B C), (. 100).
D
C
, AC

M
90
k
.
RD
HD
, k =
CB
C N A
D
N
B
A M . . 100
AM CN .
2. O
f
ABCD. RO
, f 6= 180 ,
A1 B1 C1 D1 . ,
A
AB A1 B1 , BC B1 C1 , CD C1 D1 ,
Q
D1
A1
F
DA D1 A1
D
.
E
. (AB) (A1 B1 ) =
P
M
=M
, (BC)(B1 C1 )=N , (CD)(C1 D1 )=
H
f O
C1
S
= P , (DA) (D1 A1 ) = Q (. 101). E, G, H, F
G
B
N
C
O , ABCD, B1
, . 101
, EGHF .
f
RO E S. : OE = OS,
OEM = OSM = 90 , EOS = f, ,
f
1
EOM EOM = OM = OE
. 2
cos(f/2)

f/2
M E HOk RO
, k =

234

1
.
cos(f/2)

, N , P , Q
G, H, F , . . .
, M N P Q .
3. a a1 A B.
, A. , ,
M1
, B a1
.
N1
O1
. O O1
f
a a1 (. 102). HAk RA ,
f = OAO1 k =

AO1
, O O1 , A
AO

, , a a1 . M a, A B, M1
. ,
M , B, M1 . , ABM + ABM1 = 180 . 1
2

1
2

, ABM = `AM , ABM1 = `AM1 . -

M
N
. 102

AM
ABM1 , . ,
1
2

1
2

1
2

ABM + ABM1 = `ABM1 + `AM1 = (`ABM1 + `AM1 ) = 180 .


,

D
,
w1

. , C

C
w
D ,
O

B
.
A
,

B
, M M1 N N1 , . 103
, M N B M1 N1 B, B = (M M1 ) (N N1 ).
4. ABCD , A , B C .
. w = (O, R) , ABCD
45
(. 103). RA
HA 2
O1

235

B C. w1 w , C w w1 . C,
ABCD. w1 . O1
1
OA, R . 2

w w1 ,
2,
1 0, OA , R(1 + 2).
5. , , , .
. AA1 CC1 , ABC, H. CHA1
ABA1 ( ) , , A1 A1 , H B, C A.
, A1 A1 , H C, B A
(. 20.3). A1 H HB A1 C CA. (A1 H) (A1 C), (HB) (CA),
. . B
C
H.
6.
ABCD BK
N AC. M N K
AK CD .
, BM N M
A
D (. 104).
. . 104
BKC DCA :
BK
DC
=
= l.
KC
AD

M N =

1
= (AD + KC), AD KC 2

l 90 ,
. AD CD = BA, KC BK ( 10)
1

M N (BA + BK) = BM . , M N BM .
2
7. BC, CA, AB ABC B1 C1 , C1 A1 , A1 B1
A1 B1 C1 . , AA1 , BB1 , CC1
.
. ,
. , A A1 , B B1 ,
C C1 , 90 O.
AA1 , BB1 , CC1
Q = (AC) (A1 C1 ), P = (BC) (B1 C1 ), L = (AB) (A1 B1 )
236

(. 105). 3 O .
C1
P
B
B1

C
A1

. 105

8. ABC C . A1 B1 AA1 BB1 . ,


A1 B1 C ABC,
A2
.
. ,
B
l
A1
A1 B1 C ABC
(. 106 107).
, . A B
A B2 B 1
C
, . . 106
, ,
: 1) C , 2) C .
A2
l
l
C ABC A2 B2 C,
B
A2 (BC), B2 (AC). A
B1
AA1 C BB1 C : A1 C = AC|cos C|
B2
C
B1 C = BC|cos C|. AC = A2 C BC = B2 C,
A1 C = A2 C|cos C| B1 C = B2 C|cos C|. A1

. 107
, :
CA1 = CA2 cos C CB 1 = CB 2 cos C. , HCcos C (A2 ) = A1 HCcos C (B2 ) = B1 . ,
HCcos C Sl ABC A1 B1 C
|cos C|.
237

9. ABCD . A1 C1 A C
BC, B1 D1 B2
B D A2
AC. , C
D
A1 B1 C1 D1 A1 B1
ABCD (. 108).
O
l
. D1 C
1
l f D2

C2
ABCD A2 B2 C2 D2 . A
B
OA1 = OA cos f, OB1 = OB cos f,
. 108
OC1 = OC cos f, OD1 = OD cos f OA =
= OA2 , OB = OB2 , OC = OC2 , OD = OD2 ,
OA1 = OA2 cos f, OB1 = OB2 cos f, OC1 = OC2 cos f, OD1 = OD2 cos f.
, HOcos f A2 B2 C2 D2 A1 B1 C1 D1 . , HOcos f Sl ABCD A1 B1 C1 D1 , .
10. ABC AA1 , BB1 , CC1 H. , HAC1 BCC1
.
. HAC1 HCA1
(. 109). , H H, A C, C1 A1 ,
l
HAC1 f
C
. AC0 H0
A1
AC1 H l, B1
f l
C0
AC0 H0 CA1 H . H
, f 90 . l m m
A
C1 H0
B C0 AH0 A1 CH
. 109
.
11.
ABCD, A, AB AD, AC B D.
238

. ABCD
(. 110). A,
AC D B.
AB

k =
. C1
AD
C , ABC1 =
B
C
= ADC C1 BC
ABC ADC.
C1
AC1 = k AC, AC1 D
A
AD : AB =
= AC : AC1 . . 110
B,
: AB , CC1 B . , B
(A, AB) , , B
D. D .
(A, AB) ,
. AC.
.

2.58.
, .
.
2.59. ABC C
CD. ACD ABC?
2.60. M , A B, C D.
AC BD?
2.61. . ?
2.62.
A B, B C (AB BC). , OC : OA = k 2 , O
k .
2.63.
.
2.64. ,
.
239

2.65. OABC
OA1 B1 C1 . , AA1 , BB1 , CC1
. AA1 BB1 , AA1 CC1 .
2.66. ABC AB. CD DE BC (E (BC)).
M DE. , AE CM
.
2.67. ABCD. P Q AB BC
, BP = BQ. BK
BP C. , (DK) (QK).
2.68. M ,
ABC, M A1 M B1 BC AC. P Q AB A1 B1 . , P QM = 90 . ,
AB A1 B1 OM C, O ,
ABC.
2.69. A1 B1 C1 ABC
a < 180 . , , ABC.
.
2.70. ABC P
A1 B1 C1 . ,
.
2.71. , w w1 , A A1 . ,
A A1
.
2.72. A B,
M N P Q,
M P , N Q . , M P N Q .
2.73. m n S f. S a b. A B
M m a b, A1 B1 N n a b. AB
A1 B1 .
2.74. A, B, M . AM M B AB.
, N .
N M AB.
240

2.75. AC BD
m n, AB CD .
, (m, (AC)) = (n, (BD)).
2.76. f f(a) = b f(b) = c. ,
a c .
2.77. ABC AC > CB. C AB D. M N BD CD .
, ADC AN CM .
2.78. ABCD AB CD O. A1 B1 A B AOB. , ACA1 BDB1
.
2.79. A(2; 1) B(3; 2),
.
.
2.80. ,
, .
2.81. ,
.
2.82. , , ,

.
2.83. M1 , M2 , M3 M
a
a
a
RA
, RB
, RC
. , M1 , M2 , M3
, A, B, C.
b
g
a
2.84*. RA
, RB
, RC
. b
g
a
M , M1 = RA (M ), M2 = RB
(M ), M3 = RC
(M )

, a = b = 180 , g = 90 .
2.85. , , ,
, .
2.86.
, , ABCD.
2.87. , l A.
ABC , l
A, B C
AB AC .
241

2.88. ,
. .
2.89. . .
2.90. .
. ,
.
2.91. AB A1 B1 . ,
A A1 , B B1 .
2.92. , ,

(8 ), .
2.93. ABC AA1 BB1 .
, A A1 ,
B B1 .
2.94. ,
.
2.95. ABC A1 B1 C1 . , AA1 , BB1 , CC1 , .
2.96. ABCD RP BQ , P Q AB BC . , , AR, BP ,
CB, DQ , .
2.97. ABC A1 B1 C1 . , , AA1 , BB1 CC1 ; AB1 , BC1 CA1 ; AC1 , BA1 CB1
k = AB : A1 B1 , , .
2.98. ABC ABC1 , CBA1 , ACB1 . ,
C1 A1 CB1 .
2.99. () ABC A1 B1 C1 . AA1 , BB1 CC1 P . , A, B, C, P A1 , B1 , C1 , P .
2.100*. ABC, A1 B1 C1 , A2 B2 C2
. A, A1 , A2 ; B, B1 , B2 ; C, C1 , C2 242

. , ,
, .
2.101. , A, B,
C A1 A2 , B1 B2 , C1 C2 .
2.102. AB, BC CA
ABC
ABP , BCQ, CAR, P = Q = R = 120 . , P , Q, R k
120 B.
2.103. ABC k A1 , B1 , C1 . A1 B1 C1 1/k
A2 , B2 , C2 . , ABC A2 B2 C2 . .
.
2.104. , ,
, ( ).
2.105. AD BC ABCD
90 , A1 D1
B1 C1 . , A1 B1 = C1 D1 .
2.106.
ABC, A , B
. C .
2.107. S .
2.108.
.
2.109. a A B
ABC. X AM X,
ABC ,
M a.
2.110.
ABC A1 B1 C1 . AB A1 B1 , BC B1 C1 , CA C1 A1 M , N , P . ,
(M AA1 ), (N BB1 ), (P CC1 ) .
2.111. A1 B1 C1 A2 B2 C2 . A1 A2 , B1 B2 , C1 C2 A,
243

B, C . , ABC
A1 B1 C1 A2 B2 C2 .
2.112. w M . Q
M RPa , P w a .
2.113. I ABC
A1 B1 C1 .
, A2 B2 C2
A2 = (BC) (B1 C1 ), B2 = (CA) (C1 A1 ), C2 = (AB) (A1 B1 )
.

22.

a a1 l. M a M1
a1 , M M1 l.
a a1 , a a1 l. ,
l,
. .
1. .
, , A, B, C, . . . m a, C

. a

m1
l
B
a1
A
() m (. 111).
2. m
.
, , m1
A1 B1
C1
a1
a, . . 111
a1 .
3. ,
, .
244

, AB CD
(. 112). AA1 , BB1 , CC1 ,
DD1 . AB

A1 B1

=
.
:
CD
C1 D1
AB M N D
.
B C
A
M N DC,
N
C D M
AB. 1
2 A1
B 1 C1
D1
M1 N1 D1 C1 , C1 D1
M1
N1
A1 B1 . AB

A1 B1


=
.
CD
C1 D1
CD = M N C1 D1 = M1 N1 ,

. 112

AB
A1 B1
=
.
MN
M1 N1

4. .
. F a S(F ), F1 a1 S(F1 ). b,
l . ( l) F F1 b . S0
,
S0 = S(F ) cos (a, b) = S(F1 ) cos (a1 , b),

S(F1 ) = S(F )

cos (a, b)
.
cos (a1 , b)

F a.
1 a1
S(1 ) = S()
,

cos (a, b)
.
cos (a1 , b)

S(F1 )
S(F )
=
.
S(1 )
S()

245

23.
23.1. .
,
. , ,
.
14, . ,
,
, , .
( ). A,
B, C A1 , B1 , C1
,
, A A1 , B B1 C C1 .
ABC A1 B1 C1 , , (. 113). . . ,
M1 M
A A1 , B B1 , C C1 (. 114).
C0

C0

A0

B0

M
P

A0

B0

C1

B1

P1

A1
A

A1

C1

B1

M1

. 113

. 114

(M C) (AB) = P . P P1 A1 B1
246

, A1 P1 : P1 B1 = AP : P B. M1 M
C1 M1 : M1 P1 = CM : M P .
23.2. . ABC A1 B1 C1 .
, , C
(. 18.1).
AB (A1 = A, B1 = B).
M
A
P
B

A1
B 1 M1
AM : M B M
AB C1
: M1 = M (. 115).
. 115
, , , , . ,
.
:
1 . .
2 . , C1 ,
C
.
3 . , , .
A
B
A1
B1
.
. 116
, M
(. 116). ,
N1
,
l
.
AB C N
C1 (CC1 ) (AB) = P (. 115),
M1

k =
. 117

P C1

PC

C ( )
. k = 1 (. 117).
247

l A A1
(. 118). 1 3 X1
X: (AX) l = K, (XX1 ) k
k (AA1 ), (KA1 ) (XX1 ) = X1 .
A
23.3. . X
K
l
1) ( ), 2)
X1
A1
, 3)
. 118

(), ,

AC
CB

A, B, C , 4) . : ,
,
. .
,
, . ,
, , , .
(
), . , () . . , . ,
, , . , C AB ,
AC/CB = 1. , , ,
.
,
15.
() ;
. . , , .
248

. : ,
. -
. . ,
ABCD M N P Q. , A M , B N , C P .

(CD) (P Q), (AD) (M Q) , , D Q.
, , , .
ABCD M N P Q ( AB
M N ) . D Q , AB : CD = M N : P Q.
.
,
, .
,
. .

24.

, . . ,
. , .
.
0 ,
() , .
0
.
,
.
249

0 . , , .
.
1. ,
, , ,
.
. . , . ,
,
.
, , .
2. M N BC
CD ABCD. , P
AM BN .
. .
. ABCD A1 B1 C1 D1 , M1 N1
B1 C1 C1 D1 M N
, ABCD
. P P1 A1 M1 B1 N1 .
B1 P 1
BP
A1 P 1
AP
. A1 B1 C1 D1
=

=
P1 N 1
PN
P1 M 1
PM

, A1 M1 B1 N1 (
90 ). A1 B1 = 2. A1 B1 M1 :

A1 P 1
22
= 2 = 4. Q1 P1 M 1 1

C1 (B1 N1 ), B1 Q1 : Q1 N1 = 4. P1 B1 Q1 , B1 P1 : P1 N1 = 2 : 3. , AP : P M = 4
2
BP : P N = .
3
3. BC, CA, AB ABC
A1 , B1 , C1 1 : 3.
ABC
AA1 , BB1 , CC1 .
. .
ABC . ,
ABC (. 119). (AA1 ) (BB1 ) = C0 , (BB1 ) (CC1 ) = A0 ,
250

(AA1 ) (CC1 ) = B0 , A0 B0 C0 , ABC 120 . , S(A0 B0 C0 ) = S(ABC) 3S(ABA1 ) + 3S(A1 BC0 ).


S(ABC) = 3S(ABA1 ), S(A0 B0 C0 ) = 3S(A1 BC0 ).
=

S(ABC)
=
S(A0 B0 C0 )

S(ABA1 )
. ABA1 A1 BC0 . AB = 1.
S(A1 BC0 )

, ABA1 AA21 = 7/9. ,


S(ABC)
S(ABA1 )
=
=
S(A0 B0 C0 )
S(A1 BC0 )

AA21
BA21

= 7.
B0

C1

4. A1 , B1 , C1 B1
BC, CA, AB A0
ABC . C0
,
C
B
A1
ABC, A1 B1 C1
AA1 , BB1 , CC1 .
. 119
. f A B, B C, C A. ABC . G
ABC f. f
, , AB,
BC, CA . ,
A1 , B1 , C1 BC, CA, AB, A1 , B1 , C1 B1 , C1 , A1 . A1 B1 C1
, ,
G. A0 B0 C0 A0 = (BB1 ) (CC1 ), B0 = (CC1 ) (AA1 ), C0 = (AA1 ) (BB1 )
: f(A0 ) = B0 , f(B0 ) = C0 , f(C0 ) = A0 .
, , G.

2.114.
.
2.115. ,
.
2.116. , ,
, .
2.117. , . .
251

2.118. ,
,
.
2.119. , AC ABC, AB BC M P . ,
M C AP ABC.
2.120. S
. , , , S .
2.121. ABCD P K
BC CD 2 : 1, B C.
, P D AK .
2.122. AC ABC L K ,
AL = KC l k AB BC. , , B
l k, ABC.
2.123. M AB ABCD.
P DM AC , , BD K. , P K P K,
, .
2.124. .
. , , , .
2.125. G ABC. , GAB, GBC, GCA .
2.126. ABCD , AB, BC AC N K.
, ADK ABN .
2.127. A1 , B1 , C1 , D1 AB, BC, CD, DA ABCD. ,
AB1 , BC1 , CD1 , DA1
.
2.128. AB, BC, CA ABC M , N , P M1 ,
N1 , P1 . ,
M N P M1 N1 P1 .
252

2.129. AB, BC, CA ABC M ,


N , P . CA, AB, BC M1 ,
N1 , P1 , (M M1 ) k (BC), (N N1 ) k (CA), (P P1 ) k (AB). ,
M N P M1 N1 P1 .
2.130. . ,
.
2.131. , , ,
a b, pab.
2.132. ABCD. AB, AC, AD P , Q, R. ,

AD
AC
AB
+
=
.
AP

AR

AQ

2.133. P , ABC, l, m, n AB, BC, CA .


l (BC) = A1 , m (AC) = B1 , n (AB) = C1 ,
P A1
P B1
P C1
+
+
= 1.
AB
BC
CA

.
2.134. S. ,
. . .
.

253

III

25.

25.1. . .
w O R.
w , M , O,
M 0 , OM

OM OM 0 = R2 .
(25.1)
w , O , R .
M 0 M
. M , M T w
T OM (. 120).
T
w
M 0 M w. O
M
M0
OM T OM 0 T : OM : OT =
= OT : OM 0 , OM OM 0 = OT 2 = R2 .
M M 0
. 120
. , M M 0 ,
0
M M , . . , ,
. , (). M M 0
.
M w, OM OM = R2 M
. , w ( ).
.
O , (25.1)
. O , O.

(25.1) , OM < R OM 0 > R .


.
3.01.
.
25.2. .
O .
M (x, y) M 0 (x0 , y 0 ), OM 0 = lOM l > 0. OM OM 0 = R2
lOM 2 = R2 , l =
OM 0 =

R2
. ,
OM 2

R2
OM .
OM 2

(25.2)

.
R2 x
R2 y
x0 = 2
, y0 = 2
.
(25.3)
2
2
x +y

x +y

.
M 0 M ,
x=

R2 x0
,
+ y 02

y=

x02

R2 y 0
.
+ y 02

x02

(25.4)

, .
Ax + By + C = 0 C 6= 0 , . .
.
25.3. . , O . ,
O ( O), .
l
, - w
g
OM , w
S
A
A0
O
,
OM 0 .
M
M0
g, N
(. 121). A0 A OA g
. 121
A0 l OA. M g (M 6= O)
OM l N .
255

OAM OA0 N : OA : OM = ON : OA0 ,


OM ON = OA OA0 = R2 . , N M
. , g l
w.
, , , ,
. , , , .
OA, A , .
g (. 122). AB,
w g. M 0 M g A0 ,
B 0 A, B, OM OM 0 = OA OA0 = OB OB 0 = R2 (R
OA0

OM

OB 0

OM

=
. w).
OA
OM 0
OB
OM 0
, 4OM A 4OM 0 A0 4OM B 4OM 0 B 0 , OM A = OA0 M 0
OM B = OB 0 M 0 , BM M 0 = M 0 B 0 A0 .
M A0 B 0 M 0
AM B ( AB), A0 M 0 B 0
. , M g,
M 0 g0 , A0 B 0
.
g0

M0

g
O

M
S

B0

A0

. 122

, g, , g0 , . w, g, g0 .
, S Q g g0
.
256

26.
26.1. .
( , )
.
( ), (. 123).


w
T
g

R
.
O
S
. g, M
, (
M0
).
M . 123
g OM g
M 0 , OM OM 0 = OT 2 = R2 , . .
M M 0 w (. 123).
, g .
(). g, , ,
.
. M M 0 g
O, ,
w (. 123). g w. T . , OT g. OT g T1 , OT OT1 = R2 .
OT = R OT1 = R, . . T T1 , OT
g T , w g .
26.2. .
.
. , , (, ).
. g A A0 ,
w, O
AA0 , . . g (. 124).
M g OM g 257

M 0 . OM OM 0 = OA OA0 = R2 .
M M 0 , g .
g
.
0
A

T
w
,
A
.
S
O
, A M
a b, M0
w ,
OA
. 124
a, b A0 A
(. 125).
a
, A, w
, O
, A A
.
A0

.
b
w w
. 125
( ).
a b, w,
. ( ) , A A0 a b w (. 126).

a
A0

w
A

. 126

258

27.
27.1. .
: .
. .

,
. , .
a b O , ,
,
0
, O
b =a
a
w
(. 127). O
b0
,
O
f
(a, b) = (a0 , b0 ). f
O a b,
a0 b0 ,
O P 0
. 127
P a
b. a0 b0 O P 0 . O f
a0 b0 O.
a0
a b (. 128).
a
, ,
P0
b0
, , b
f
. P
, . 128
,
, .
27.2. . O R A B
OA

OB 0

A0 B 0 , OA OA0 = OB OB 0 = R2 ,
=
.
OB
OA0
O, A, B , OAB OA0 B 0 . :

A0 B 0
OA0
OA0
R2
=
, A0 B 0 = AB
. OA0 =
AB
OB
OB
OA

259


A0 B 0 = AB

R2
.
OA OB

, O, A, B . .

28.
a a0 w.
, (. 13.2). ,
.
. O, O.
. O
a a0 (. 129), M N
a M 0 N 0
a0 . A A0 , B B 0 O
.
M AO M 0 A0 O M AO A0 M 0 O.
A0 M 0 O + N 0 B 0 A0 = M AO + M AA0 = 180 ,
A
N 0 B 0 A0 = M AA0 AM
M
B 0 N 0 .
BM k A0 N 0 . , AM B
a
B0
N
B 0 N 0 A0 O
w
0
N
(A B 0 , B A0 , M N 0 ). ,
B
a a0 .

M0
, .
a0
. 0
A
, ,
. 129

. ,
,
.
. S a a0 , A A0 , B B 0 (. 130). SA0 = |k|SA
260

SB 0 = |k|SB, k . SA0 SB =
= SA SB 0 = |k|SA SB. a S
SA SB AB
. , S a, , S a, S
a, , S , S. S a .
|k|SA SB = R2 .
SA0 SB = R2 SA SB 0 = R2 , A0
B, A B 0 S R, S A0 B
AB 0 . (. 130)
(. 131)
.
A
B0

A
B
B10

A0

a
B0

S1

a0
S
A0

a0

B1
A01

. 130

. 131

29.

. w , w , .
,
.
261

1. ,
A B a.
. , ,
, A B a, . w
(B, BA).
a
A
T
a0 , a
y
y0
x x0 . A
. 0
B
a
x0 a
w
a0 .
: A
0
x0 , a0 .
x
x
,
x. . 132
. 132.
g
a
b
A

a0 , . . 2, 1
A
B
0. . 132 .
2.
D
C
. ,
d
(. 133).
.
. 133
A a b. b0
B0
0
a0 b0 ,
g
g d g0
0
d
C0
d0 , C 0
b0
a0 B 0 D0 (. 134). ,
D0
a0
B 0 , C 0 , D0 . , , . 134
, B 0 ,
0
0
C , D , , B, C, D
A .
C 0 , d0 g0 .
a0 d0 D0 262

b0 g0 . D0 B 0 C 0 , , C 0 .
3. , ( ).
. A
ABCD. , B 0 , C 0 , D0 B, C, D,
C 0 B 0 D0 (. 135).
D0
B 0 D0 = B 0 C 0 + C 0 D0 . 27
B 0 D0 = BD

R2
,
AB AD

B 0 C 0 = BC

R2
,
AB AC

D
C C0

C 0 D0 = CD

R
,
AC AD

R . :
BD

R2
R2
R2
= BC
+ CD
.
AB AD
AB AC
AC AD

B0

. 135

AB AC AD
R2

AC BD = BC AD + AB CD.
4. ,
, (
).
.
,
. , . ( 6 14).

3.01. w O. M A AB, OM ,
P M A w B. BP
OM M 0 . , M M 0
w.
3.02. , .
263

3.03. ( ) .
.
3.04. ,
. .
3.05. ,
.
3.06. , .
3.07. , , .
3.08. , , .
3.09. , A
a m.
3.10. , .
3.11. ,
.
3.12. , , , .
3.13. . , .
3.14. ,
,
, . .
3.15. A C
ABCD 90 . , (AB CD)2 + (AD BC)2 = (AC BD)2 .
3.16. , A, B, C, D

AB CD + BC AD > AC BD,
, A, C B,
D.
3.17. . ,
, ,
. .
264

3.18. O A, B, C. ,
ABC, , 180 .
3.19. , , , .
3.20. w a b, , a, , b.
.
3.21. , .

265

, ,
I
1.2. A.
1.3. S. 1.8. DAI DIA.
2.2. .
2.5. : , ,
p
, (. 2.6). 2.6. b(a + b),
ab2
ab
abc
p
, 2
. 2.8. . . 2.1. 2.9.
. 2.7. 2
2
c b c b2
b(a + b)

, . 2.10. P w. 2.12.
.
2.13.
r

1+ 5

. 2.14.
2

5 5
. 2.15. . 2.14.
2

2.16. A , BC, BP CP . 2.18.


,
p
. 2.19. 8 : 5. 2.20. r12 + r22 . 2.21. 4/5.
2.22. , O M N K
. 2.23.
.
a 10

3.4.
. 3.7. . 3.8.
4
,
. 3.9. ABC
, AM AN AB AC. 3.10. . 3.1.
, . 3.26.
a
b
g
2R2 sin3 a sin b
. 3.29. 2r cos , 2r cos , 2r cos . 3.31. 4 2.
. 3.27.
sin(a + b)
2
2
2
4.2. , . . , , ,
. 4.3. .
4.4. , -

, . 4.5. OM = OB + OC = OH OA = AH. 4.8. 4A1 B1 C 4ABC


(. 4.3). 4.9. . 4.3 . 4.12. . 4.10 . 4.17.
. 4.10 (3.15). C ,
AH + BH CH = 2(R + r). C = 90 ,
: a + b = 2(R + r). 4.18. . . 4.2 . 2.5. 4.19. , ,
1
AH AA1 = (b2 + c2 a2 ). 4.21. 2

bc
ac

,
2p
2p

I. 4.22. . 4.1. 4.28.


(4.4). 4.29.
COH, . 4.9 , OCH = |A B|.
C .
r
4.30. . 34. KCI CI =
. sin(C/2)

. 3.17. 4.31.
(. 4.15) .
5.2. . . 5.3,
. 1.8. 5.5. BCI ACI3 . 5.6. (4.4). 5.7. . 4.4. 5.12. . 5.1 4.8. 5.13. I I1 I2 I3 . 5.14. . 5.2 (. 4.4).
5.15. CI1 Y1 CI2 Y2 (. 37)
. 5.12. 5.20. . 5.2 5.5.
7.2. ,
. 7.5. ,
AC . 7.6. . 7.7. (CI) (DE) = M (BI) (DE) = N . B M
N C . 7.9.

(a + b c)c
. 7.12.
a+b+c

AOB COD (O
, ABCD). 7.13. . 7.3. 7.14. , ,
,
. 7.15. ,
.
. 7.16. ,
267

. . 7.18.
, . 7.19. M N AB CD (I, r), AM I CN I
, AM N C = IM IN = r2 . BM N D = r2 .
a2 + a(n m)
mn. 7.24. M = (AD) (BC).
7.23.
am

M AB M CD . ABCD.
.
A, B D. C1 C
, C, D, C1 .
8.3. P ABC
P DAB. 8.4.
AB1 C1 D1 . ACD
B1 C1 D1 . 8.5. ACDE, E
D . 8.6. AB1 CA1 BC1
. ACA1 C1 , A1 BC1 B1 , AB1 CC1 , AB1 CA1 . 8.7.
, ,

a+b

c

,
. 8.9. ,
, M A, M B, M C,
, M AB, BC, CA.
8.10. ,
,
. 8.11. . 55,
. 8.12. . 8.11.
9.5. , B1 C1 AC
AB . 9.7.
OAB HAC. 9.8. . 1 9. 9.11.
BEF . 9.12. 2 9.
ABC AA1 , BB1 , CC1 .
10.3. . . 9.3. 10.5. T ABC AB, H3 , Z3 AB,
L = (CT ) (IE), E CH3 .
CT H3 , , L, E, Z3 , I, E, Z3 .
268

(3.12) (5.2) . 10.6. . 10.8.


(
) ACC1 A1 B1 B. 10.9. . 10.5. 10.11. ( 10, 1). , ,
, . 10.12.
. 10.13. . 10.14. . 71.
11.3. d = a cos A + b cos a + c cos D, a AD BC. 11.4. .
. 3.2. 11.5. , .

1 2
(a + b2 + c2 4 3S). 11.10. (11.5). 11.13.
6

. 11.14. ACBD.
12.3. 2S. 12.4.
. 12.8. 4. 12.9. , , ,
, . 12.10. , ABN
AM D M BC. 12.11. A C F BD.
ABCF , ABCT , ADCF . 12.12. ,
. 12.18.
, ABCD AC. .
.
13.3. . 13.4. , r =
1

ab
. 13.6. . . 13.1
a+b+c

R + r = (a + b). 13.8. . 13.2. 13.11.


2
O G ABC. , O 2
2
ABG. AO + BO 6 AG + BG = ma + mb ,
3

269

ma + mb > 3R. , mc > R. 13.12. A1 ,


B1 , C1 BC, CA, AB ABC, O . AA1 6 R + OA1 ,
1
1
1
1
+
+
=

BB1 6 R + OB1 , CC1 6 R + OC1 . ,


ha
hb
hc
r
OA1
OB1
OC1
+
+
= 1. 13.17.
ha
hb
hc

(13.21). 13.19.
1
1
1
9
+ + >
.
a
b
c
a+b+c

x
y
+ > 2 y
x

x, y. 13.20. (13.10) (13.22).


. 13.21. . 13.22. 13.22. (. 4.15). 13.23. . 13.17. 13.24.
. 13.25. (13.22). 13.25. (3.18),
A

(5.3), (5.4). 13.26. a2 = (b c)2 + 2bc(1 cos A) > 4bc sin2 . 13.27. 2
,
. 13.28. . 13.26.
13.29. (3.15). 13.30. (13.20). 13.31. (13.18)
pA pB

,
. 13.32. ,
2
2
. 13.31. 13.33. (13.22)
(13.10). 13.37. ABC
ABD . , , |CAB CBA| < |DAB DBA|. ,
AB,
, C AB, D. 13.39.
R + r = R(cos A + cos B + cos C), R + r < c
(c )
1
2

sin C > (cos A + cos B + cos C). 60 6 C 6 90 ,


(13.10) . 13.42. A1 B1 C1 90
B

A
,
2

90 , 90 . 2
2
. 13.28. 13.43.

a
b
c
+ + > 3. 13.44. ,
b
c
a

270

. 11.5. (13.11).

14.1. , a = b = h 2. 14.2. , A B.
14.4. . R : r = 1 + 2.
14.5. , M
. 14.6. . (. 4.15). 14.7. .
14.8. . 14.9. ,
AM
SM CA + SM AB
=
.
M A1
SM BC

6, 8. 14.10. M ABC. .
. 3.1. 14.11. A , AM . , BC. , A ,
. 14.12. CD M K,
K = (AD) (BC). 14.13. . 3 14.
.
16.2. A
. 16.3. (A, a), (B, b), (C, c). 16.4.
AB
. 16.6. P a, x w
, . O
a, P O
. 16.9. AB.
,
H ABC.
17.1. 17.
17.2.
A B. ,
. 17.4. A B R r, M

MA
R
=
( ).
MB
r

271



3 3( 13 1)
1 2
1 2
60
2
. 6.
.
1. |a b | tg a. 2.
m 4S. 3. 72. 4. 13. 5.
2
2
37
32p

2
2m sin a sin b sin g
2 3
1
11
7.
. 8.
. 9. (7 + 9 tg a ctg b). 10. 5 : 2. 11.
.
9
2
12
2 sin2 b + 2 sin2 g sin2 a
p
ab(a + b)
12.
tg a. 13. 8a2 sin a cos3 a. 14.
p(p e)(p f )(p 2m),
2(a b)
r

1
5
p = (e + f + 2m). 16.
. 17. 1 n + n2 . 18. R : r = 3. 19. 5 : 10 : 13.
2
8

b 2
20.
a + b2 . 21. 2. 22. 3 3. 23. 4a
4
3
14. 24. 90 , 60 , 30 . 25. cos ABC = . 26.
, R : r =
5
5

80(1 + 3 7)
20 + 5
. 31. 2 3. . . 4.3. 32. CK k AM

1701
24 5

CM
AC 2
6
6
,
=
.
34.
(21

105)

(14 70). 36. M B AB 2


8
6
1
CA1 DB1 A1 B1 h, h 2

ABC AB. 37. K = (DA) (M B)


1 2
m + n2 . 39. ABC
, KM = M B. 38.
2
B = 2C, R = 12, 5. 42.
2
2
2
CO CDI.
= r2 (p c)2 .
OI = R 2Rr, CI
51

CD2 = 4R2 ab. 43.


. 44. 3R2 . 45. cos C =
. 47. .
2
2
r
2m
(3.15). 48. h
. 49.
m+h

R2
(4.5). 52. r1 r2 + r2 r3 + r3 r1 = r32 . 54. 2 3 3. 55.
. 56. ab. 58. Rr.
a

1
59. 3 2. 60. sin DEF = . 62. h 5. 64. 17
2Rr
, R r , d
r+R+d
p
1
. 65. (b2 a2 ). 66. 2 S tg b. 69.
2

ABCD A1 B1 C1 D1
, AA1 = BB1 = CC1 = DD1 .
: 1) AC A1 C1 AC = A1 C1 , 2) AC k A1 C1
. 70. P Q =
1
= |AP AQ| = |a + c b d| = M N . 71.
2
ABM Q AN P D .
C, Q, N p

P M . 74. 2 + 5. 76.
272

BD2 . 78.
. 79.
A1 A2 A4 A5 . 80. . (12.5). 81.
ad + bc p
(a + b + c + d)(a + b c d)(a + d b c)(b + d a c).
S=
4|ad bc|

82. (12.5). 83. B , AC, , , BD AP CQ. 84. 2 ctg2 a.

1
85. 2 cos A cos B cos C. 86. 2 2 (2 2). 87. 2 sin A sin B
2
cos(A B) = 1. CC1 ABC, OCC1 =
= |B A|. 96. ABC CEF

c
. a + b 6 c 2. 97. . (13.5). 98.
a+bc

(13.19).
e1 + e2 e3 . 99. m
O h O 2f,
a + h = R(sin 2f + cos 2f) =

 2

1
= R 5 sin f + cos f R 5 sin(f + a) 6 R 5.
5

1
a+b 2
(a + b2 + 3ab),
8ab
5
a2
AA1
=1 2
. a2 + b2 > 2ab. 101. ,
AA2
2(b + c2 )
a2
b2
c2
3
2
+ 2
+ 2
> . 102. AOB = f,
b + c2
c + a2
a + b2
2

cos f = . 100. R1 + R2 =

COD = y.
p f + y = 180 , AB > 2r tg f, CD > 2r tg y, tg f tg y = 1,
tg f + tg y > 2 tg f tg y = 2. , AB + CD > 4r.
. 105. : n(a1 b1 + a2 b2 + ... + an bn ) > (a1 + a2 + . . . + an )(b1 + b2 + . . . + bn ),
a1 , a2 , . . . , an b1 , b2 , . . . , bn ( ). 108.
aM A + bM B + cM C.

. 116. , 2R1 sin > x2 + x3 .


2
4a cos2 a
1 2
4
. 3.21. 117.
.
118.
a
sin
2a. 119. R. 120.
1 + 4 cos2 a
4
3

ACB = 135 .

2(a2 + b2 + ab 2). 121. O


l .
273

l , . 124.
AC1 BC (C1
AB) ( 7, 3). 125. S,
XY , , A OA. , ( 2)
XS XZ
= 1, Z = (OA) (XY ). 127. :
SY ZY

AB
|a b|. 133. ,
. 135. C1 w AC. O, B, C,
C1 .
( 16). 136. (12.18)


(a + c) + (b + d) 2
4S 6 ab + cd + ad + bc = (a + c)(b + d) 6
= p2 .
2

II
I
1.03. ADCH . ZA (D) = M
ZC (D) = N , AH CH
M N ( ). , H AH CH.
1.04.
, ,
. , .
1.05. O. O. .
1.06. O AC. , ZO (B) = D.
ZO (B) = B1 , AB1 C = CBA = ADC. B1 6= D ( ).
1.07. AOB COD, BOC DOA O ,
. , , , .
274

, ABCD
.
1.08. ,
, . .
1.09. M
.
1.10. M , N , ZO (M ), ZO (N )
.
.
1.11. , .
1.12. OM
ZM .
1.13. , .
1.14. , , , O .
. ,
. O, .
, O,
.
1.15. , .
1.16.
.
1.18. ,

.
1.20. ,
, . ,

.
1.21. .

. 275

. .
1.22. 1.21. , ,
.
.
1.23. A1 = ZP (A). f AM B AB. (CD) (M B) = K,
(KA1 ) k (AM ) () M KA1 = f. K
, A1 B
p f. .
1.24. ,
a/2 ,
.
1.25. A, C, M , K w.
, w M K.
1.26. ABC. O ABC. Q1 , Q2 ,
Q3 Q = ZO (P ).
1.27.

.
1.28. AB, CD, EF
.
1.29. .
. , .
1.30. .
, ,
.
1.31. M
.
1.32. , .
276

1.33. 1.32 , ABC


, .
1.36.
, , , .
,
.
1.37. ,
,
. , ,
,
.
1.38. m a b,
a b1 , b1 = Sm (b).
. m a b,
, , . .
1.39. a b1 b
.
a b1 , . . 2, 1, 0.
1.40. P = a (M1 N ), M1 = SA (M ).
1.41. B1 = SAC (B), B1 CD . A B.
1.42. ABC AC
BC, h AB ABC ABC1 .
ACC1 CAC1 ,
A B. h
CC1 , B = Sh (A).
1.43. ABC AB l C. A1 = Sl (A),
AA1 B ,
.
1.44. C1 , C AB. C1 M D . C1 D
M .
1.45. AB CD ,
, S = (AC) (BD)
O = (AD) (BC).
277

1.46.
. , ,
.
1.47. ,
(. 11), , .
1.48. , .
m , M a1 , a m.
1.49. l t a ( r),
X m, l t
m, a1 = Sl (a) a m. a1
b 2r, a, ,
m r. , X m
l a a1 . 4, 2, 0.
1.50. BC
P1 = SBC (P ) AB.
1.51. .

.
1.52. ABC AB, |A B| = a, h AB. C l AB.
B1 = Sl (B), ACB1 = 180 a ().
: AB, l, AB1 , C
l ,
AB1 180 a. . ABC ABC1
AB.
1.53. O ABCD . AD DC BC
OA. .
.
1.54. P1 = SOC (P ). P1 Q C
P1 CQ, .
1.56. AB BN ABC
TN M (A) = A1 , TN M (B) = B1 . AM B1
278

AA1 = A1 B = BB1 . AM B1 A1 M B, A1 M = M B
AC = CB.
1.58. M BA.
1.59.
ABC, .
1p 2
2c + 2d2 (a b)2 , a b ,
1.60.
2

c d .
1.61. KC , B.
(BC) k (AK) TKC (A) = B. AB = KC = 2r.
1.62. TAC , P1 = TAC (P ). , AP C =
= P CP1 . P CP1 P P1 .
1.63.
. A1 C1 = AC, B1 D1 = BD
(A1 C1 ) k (AC), (B1 D1 ) k (BD). ABCD A1 B1 C1 D1 .
1.64. ABC ABH ( 1.33). CH
. TCH
ABC ABH.
2 . 4.2.
1.65. OM TM N , N , O.
1.66. ABCD AB CD
TDC (A) = A1 , A1 BC .
.
1.67. . 1.66.
1.69. ABCD AB CD
TDC (B) = B1 , AB1 C . D .
1.70.
.
1.71. A B a b m
. A1 B1 A B m,
b
a1 = TA1 B1 (a).
1.72. , ,
. 279

. a h , , a = h , a < h
.
1.73. ,
,
.
1.74. a, b, c, d , , A a, B b, C c, D d. TAB (D) = C,
C = c d1 , d1 = TAB (d).
1.75. ABC
M TAM (M ) = N , M N C . . A1 B1 BC CA
, AA1 K, K = TBA1 (B1 ),
.
1.76. B1 = TDC (B), AB1 C , , .
ABB1 , D = TB1 B (C). , AB1
.
1.78. , .
.
1.79. 4.
1.81. ,
, a/2. .
1.82. l , ,
,
A .
1.83. ABCD AB CD. D1 = TCB (D), ABD1 .
ADD1 ( ), C = TD1 D (B).
.
1.84. 48 n = 72 + 360 k, n k ,
n = 9 k = 1. ,
48 , , 48 9 = 432 ,
72 . 48 n = 90 + 360 k
280

(). , .
90
1.85. RB
P N AC.
1.86. 120 .
1.87. , , ,
90 . , ,
, .
1.88. .
, M N P Q,
. M N P Q.
1.89. 360 : 5 = 72
,
( ).
.
.
1.90. 1.89.
1.91. ABC ACD BM = CN . 60
RA
B, C, M
60
C, D, N . RA
(M ) = N , AM N .
1.92.
90
P .
90
(C), B (CC1 ) COC1 1.93. C1 = RO
.
1.94. CD DA DN KD.
1.95. ABC B1 =
90
= RA
(B). K1 B1 Q AK1 AK, AK1 k QE
AK = AK1 = (1/2)QE. BC B1 Q, . AM k QB1
AM = (1/2)QB1 , AM BC AM = (1/2)BC.
60
1.96. , M1 = RB
(M ) M C.
1.97. 5 6 BB1 , CC1 , DD1
, .
1.98. 60 , M N
281

( 5 6). , 60 .
1.99. M C AQ (1/2)M C (1/2)AQ.
,
B 90 .
1.100. (. 5.3): A B O1 O2 ( ).
1.101. , A B .
, , AB 45 .
.
1.102. O g w A B, ,
AOB = a. A ( B) g
a
(w).
w1 = R O
1.103. A1 A
. AA1 .
1.105. O
60
60
(B) = B1 , A1 B AB1
(A) = A1 , RO
RO
.
1.106. , A, ABCD
90
a b. A a B b, RA
B
D b. D b a1 a .
1.108. O w (O, OM )
a P Q. O
P OM QOM a w,
M . A
.
1.109. - , ,
. 1.108.
1.110. ABCD E1 =
90
= RA
(E). E1 (BC) BE1 =DE. , F AE1 =AF E1 ,
AE1 = F E1 AE = AE1 = F E1 = F B + BE1 = F B + DE.
1.111. AB A1 B1 , OO1
282

, AB A1 B1 . AB
A1 B1 , .
AB B1 A1 . ,
. ,
A1 B1 B1 A1 , . . 180 .
1.112. O
ABC O1 , A1 B1 C1 .
OO1 , O = O1 .
1.113. ABCD A1 B1 C1 D1 . , ,
: 1) A A1 B B1 , 2) A B1 B C1 ,
3) A C1 B D1 , 4) A D1 B A1 .
.

, . a,
p
3p
a + , a + p, a + . 2
2
p
3p
, , p, .
2
2
1.114. P A > P B P A > P C, ACP1 60
ABP RA
. AP P1 .
P P1 6 P C + CP1 , P A 6 P B + P C.
, C P P1 , . . - B1
, ACP + ACP1 = 180 ,
C
A1
ACP + ABP = 180 .
, P
M
ABC A
.
B
60
1.115. ABC 60
M1
. RC
(B) = A1
60
RC (B1 ) = A (. 136). ,
AA1 BB1
60 .
C1
60
CC1 BB1 RA
. . 136
(AA1 ) (BB1 ) = M , M (CC1 ). 60
, RA (M ) = M1 , M1 (CC1 )
( (BB1 ) (CC1 )), AM M1 , AM1 C1 =
= AM B = 120 . , AM1 M + AM1 C1 = 60 + 120 = 180
283

M (M1 C1 ), . . M , M1 , C, C1 . ,
M C1 = M M1 + M1 C1 M C1 = M A + M B.
1.116. A, B, C, D , . P , AP BD
AP = BD. 1.88 P , C.
B D P C, . A, . ,
.
1.117. M A O f, A1 A B1 M f.
BB1 M ( ) f.
B1 : , M B
f ( p f).
90
1.118. RO
O M BP S
M Q SC.
90
A B A1 B1 . 1.119. 1. RC
(AA1 ) (BB1 ) , , (CD) k (B1 B).
AC : CB1 = AD : DB = 2 : 1.
90
2. RM
M
C B, B K, (CB) (BK), A1 N , N BK,
A C. (AA1 ) (CN ) (AA1 ) (CN ), N (CD).
CA : BN = 2 : 1, AD : DN = 2 : 1.
1.120. l1 w1 (M, R) a
A. RM
, l1 l2 w2 (N, r). M A
M P , M P = R r. P l0 ,
a
l1 . a RM
, l0
l, N ( 1.108).
1.121. a, b, g b g = {A, H}, g a = {B, H},
a b = {C, H}. 1.33, , H ABC. ,
D AH a,
C, b a, A D ( 5 6). , CA = CD. B, g a,
A D, BA = BD.
A D (BC). , (AH) (BC).
(BH) (AC) (CH) (AB).
1.122.
.
284

1.123. a b c = O, O l ,
(a, b) = (l, c) ( ). Sb Sa = Sc Sl ,
Sc Sb Sa = Sl . a, b, c
.
1.124. .
1.125. l, m, p O (ABC),
( 1.123), O. ,
Sp Sm Sl B . , Sp Sm Sl = SOB .
Sl Sp Sm = SOC Sm Sl Sp = SOA .
1.126. I ABC.
SIC SIB SIA , I ( 1.123).
AC , AC. ,
AC, I .
1.127. ,
180 . , a, b, c, d , 2(180 a)
, (Sd Sc ) (Sb Sa ) = RB

2a
RA
= Tr, A = a b, B = c d, a = (a, b), (c, d) = 180 a.
1.128. a, b, c, d , , Sd Sc Sb Sa = Tr,
Sb Sa = (Sc Sd ) Tr. Sb Sa Sc Sd :
2(a, b) = 2(d, c). (a, b) ,
(d, c) ,
. ,
180 .
1.129. A + B + C = 180 ,
.
M .

B
A
(. 7.6), , RB
RA
= RIA+B = RI180 C , I
ABC .

C
RC
RI180 C , , M
I AC,
M IC = 90 (1/2)C M CI = (1/2)C.
1.130. 180 ,
. ABC
, B
.
285

1.131. 90
. 4
8 .
1.132. ABC O1 ,
O2 , O3 , BC, CA, AB
90
90
. RO
RO

3
1
K AC, 180 C A. , O1 KO3
90
90
, K = 90 (. 7.6). , RK
(C) = O2 RK
(O3 ) = O1
CO3 O1 O2 .

120
180
RM
1.133. RP60 RO
1
, 360 A . M OP 90 ,
60 , 30 .
1.134. , .
AA1 ,
, r 6= 0.
AA1 k r AA1 6= r AA1 r .
1.135. Wlr = ZO Su = Sv ZP , u l, v l, u l = P , v l = O,
P O = (1/2)
r.
1.136. ,
, .
, .
1.137. , . A B
O O1 ( ).
1.138. , A A1 , B B1 ,
C C1 .
1.139.
, .
1.140. AB1 BA1 .
.
1.141. Wlr = Sl Sv Su , ZA = St Sl , u k v, u l, t l.
ZA Wlr = (St Sl ) (Sl Sv Su ) = St Sv Su . u,
v, t , , p
t ,
u v ( 1.123).
286

1.142. A A1 B B1
AA1 BB1 .
, A B1 B A1 ,
AB1 BA1 .
AB1 BA1 , .
.
1.143.
. ,
A AB .
, , ,
A.
1.144.
. , O ,
P M QO . , OQ = P M .
,
1
2

1
2

1
2

OQ = (OC + OD), OP = (OA + OB), OM = (OA + OB + OC + OD).


1

: P M = OM OP = (OC + OD) = OQ.


2
1.145. O a1 , a2 , a3 , a4 ,
a5 , AB, BC, CD, DE, EA .
( ) A0 . l ,
. l
.
1.146.
( x0 = x,

y 0 = y) x 3y = 0 ( ).
60
120
240
1.147.a) RO
, ) RO
, ) RO
, )
3x + y = 0.
1.148. :

x0 = 1 (12x + 5y + 59),
13

y 0 = 1 (5x 12y + 43).


13

:


x0 = y + 3,
y 0 = x + 1.
287

1.150.

x0 = x

2a
(ax + by + c) b,
a2 + b2

y 0 = y 2b (ax + by + c) + a.
a2 + b2

1.151. Ax + By (2Ax0 + 2By0 + C) = 0.


1.153. ,
y = 1. A
a, b. OA = 2, . . a2 + b2 = 2
( , x2 + y 2 = 1).
90
B, C, D.
RO
1.154. , Oy AA1 BB1 , C . l y = kx.
A a, b, A1(a, k a
), B1 (b, k b).
p
1
2
2
2
2
CA1 + CB1 = CA (1 + k ). k = tg f = ctg f, 1 + k 2 = 2
2
sin f
1
CA21 + CB12 = CA2 2 .
sin f
1.155. ABC .
1
BE = (BA + BC) 90
2
1
1
(BM + QB) = QM , .
2
2

1.156. O1 , O2 , O3 , O4 , AB, BC, CD, DA ABCD (


). O1 O4 = O1 A + AO4 .
90 O1 A O1 B AO4 BO2 , O1 O4 O1 B +
+ BO2 = O1 O2 . , O1 O4 O1 O2
. ABCD O1 O2 O3 O4 , .
1
1.157. M N = (AC 1 + CB) 60
1

(AB + CA1 ) = M P . , M N P
2
.
1.158. 60 A1 O = A1 C + CO
A1 B +OA=(M B M A1 )+(M AM O)=(M A1 +M O)=2M D,
D OA1 . , M OD ,
M OA1 = 60 . OA1 M = 30 , OM A1 = 90 .
1
1.160. P R = (A1 C1 + BD) QS =
1
2

= (B1 D1 + CA) 90 .
288

1.161. ABC

R90 (AB) = CD. AQ = AB + BQ 90 CD + BC = BD N B = N A + AB AC + CD = AD.


, CD, AQ, N B
ABD, .
1
1.162. SM = (AD + BE) 90 .
2
1.163. B1 B1 . . . Bn . , B1 B 2 + B2 B 3 + . . . + Bn1 B n + Bn B 1 = 0.
90
RO
B1 B2 , B2 B3 , . . . , Bn B1 C1 C2 , C2 C3 , . . . , Cn C1 ,
OA1 , OA2 , . . . , OAn . C1 C 2 + C2 C 3 + . . . + Cn1 C n + Cn C 1 = 0, C1 C 2 = sin A1 OA1 , C2 C 3 = sin A2 OA2 , . . . , Cn C 1 = sin An OAn .
,
sin A1 OA1 + sin A2 OA2 + . . . + sin An OAn = 0.
1.164. 90 ABC A0 B0 C0 . ,
B0 C 0 = xAA1 , C0 A0 = yBB 1 , A0 B 0 = zCC 1 , x, y, z ( ):
x=

B0 C0
BC
BC 2
a2
=
=
=
,
AA1
AA1
2S
2S

y=

b2
,
2S

z=

c2
2S

(S ABC). B0 C 0 + C0 A0 + A0 B 0 = 0,
xAA1 + yBB 1 + zCC 1 =
0 a2 AA1 + b2 BB 1 + c2 CC 1 = 0.

II
2.01.
.
, .
2.02. . AB CD .
2.04. A1 B1 C1
ABC.
2.05. D AB M ABC.
1/3
HD
(C) = M . C , M , , .
2.06. ,
.
289

2.07. : 1) O
AC BD ABCD, A C,
2) AD BC, A D.
2.09. O,
. O , O.
2.10. w w1 O
O1 , B w, C w1 (BC) (OO1 ) = S. HSBC (w) = w1 . AD
w1 , A D , ,
(AB) k (DC), BAC = ACD = 90 . w w1 ,
.
2.12. ,
, M M1 N N1 , (M1 N1 ) k (M N ), `M P = `P N
M AP = P AN .
2.13. A, , , P AQ =
= M AN , M AP = N AQ.
2.14. A, ,
M N , , P Q. M M1 N N1 .
A, :
M AQ + N AP = M AN + P AQ + 2N AP =


1
1
1
1
1
= `M N + `P Q + 2 `AN + `AP = `M1 N1 +
2

2
2
2
2
1
1
+ `P Q + `AN1 + `AP = (`M1 N1 + `P Q + 2`N1 AP ).
2
2

(M1 N1 ) k (P Q), `N1 AP = `M1 Q.


, , M AQ + N AP = (1/2) 360 = 180 .
2.15. S
,
A A1 , B
B1 , C C1 , D D1 .
SA SB1 = SC SD1 ACS D1 B1 S.
2.17. M M1 N N1 . M1 N
, M ,
. M N1
, N .
290

2.18. a O r.
HA2/3 (C) = B. B a1
a , ,
. OA < 5r ,
OA = 5r , OA > 5r .
2.19. A
a b M N (M a, N b) , M A : AN = m : n,
m n . HAk k = n/m M
N a a1 , N .
N a1 b.
, ; ,
.
2.20. . , k 1/k .
2.21. - w,
O, O
. w (OA) = {P, Q}, HOP A HOQA . ,
A .
2.22. , M a.
.
2.23.
. ,
,
, . A
S1 S2 , AS1 AS2
, .
, ,
A.
2.24. t
S. t, t,
S. S . ( ). a
R ,
(2R a)a = (a/2)2 , a/R = 8/5.
291

2.27. ABCD O
AC : BD = m : n, m n . OA OB A1 B1 , OA1 = m OB1 = n.
(A1 B1 ) k (AB) (). OA1 B1
. AB
AOB ABCD.
2.28. , , . ,
, .
2.29. w a
S m A. S,
w a, m t a.
t k m, T t a .
T A a S. . , A
/ a. A a
.
2.30.
, 2.29.
. ,
.
2.31. , a k b . (AB) a (AB) a =
= A0 , (AB) b = B0 . AA0 6= BB 0 , , A B a b (A0 B0 ). AA0 = BB 0 ,
, . (AB) k a k b,
AB ,
. A B , a b
.
.
2.32. M , l
l1 , , N P
M .
1
2.33. O : OA1 = (OB + OC + OD), OB 1 =
3

= (OA + OC + OD). A1 B 1 = OB 1 OA1 = AB.


3
3
. ,
A1 B1 C1 D1 ABCD
292

k = . AA1 , BB1 , CC1 , DD1 ,


3
G ABCD 1
4

( 1.21). OG = (OA + OB + OC + OD).


2
2.34. HM
ABC
A1 B1 C1 , , G M1
. G M M1 . , M M1 G.

1
4

2.35. O : OG= (OA+OB+OC +OD) OG0 =


1
(OA+OB +OC), G0 ABC. OG=
3
3
1
1
= OG0 + OM G0 G=OGOG0 = G0 M . , 4
4
4
1
M G G0 .
4

2.36. (AB)k(A1 B1 ), (BC)k(B1 C1 ), (CD)k(C1 D1 ), (AD)k(A1 D1 ),


(AC) k (A1 C1 ), (BD) k (B1 D1 ), ABCD A1 B1 C1 D1 .
, (AB) k (C1 D1 ), (BC) k (A1 D1 ), (CA) k
k (B1 D1 ), (CD) k (A1 B1 ), (AD) k (B1 C1 ), (BD) k (C1 A1 ).
,
.
,
D0
. ABCD , A0
. D

C0
. B0

C
90
B1
O
A
1
, A1 B1 C1 D1
D
C1
(. 137). - 1
ABCD,
A
B
. 137
ABCD.
2.37. O k1 , OA1 = k1 OA.
OA2 =

OA + kOA1
1 + kk1
S1
. OA2 =
OA.
=k12
1+k
1+k
S

293



 1 + kpS /S 2
p
1 + kk1 2
S2
1
=
, k1 = S1 /S S2 = S
=

S
1+ k
1+k

2
S + k S1
=
.
1+k
2.38. a A0 g, b c B0 C0 ,
A0 B0 : B0 C0 = m : n. A0 . ,
. l S g.
M l.
2.39. g, p S = a b. g a = A0 , g b = B0 .
g C0 , A0 B0 : B0 C0 = m : n (
). (SC0 ) c = C, l, C g, . (SC0 ) k c .
2.40. HBk (k > 0).
A
N
BM N C BM1 N1 C1 ,
M
P
M1
BM1 = M1 N1 = N1 C1 , M1 [BA),
N1 m
C1 [BC), N1 C1 k AC (. 138). BM1 N1 C1 .
B
C
C1
M1 [BA). . 138
N1 (M1 , BM1 )
m, BC
P [CA), CP = BM1 . HB1/k BM1 N1 C1 BM N C.
1

AB < AC < 2AB.


2
2.41. , . 2/3
HM
,
.
2.42. , w(O, R), ,
. H, G O
, OH = 3OG
( 2 14). , ,
HO3 .
2.43. A1 B1 C, C1 AB, A1 B1 C1 M N , AB HC1/3 . ,
ABM N . 294

M N P QRS,
(1/3)BC, (1/3)CA, (1/3)AB
.
2.44. M N , CB CA. P , P ACBD, A0 CB0 D0 ,
ACBD HC1/2 (A0 , B0 , D0
CA, CB, AB ).
2.45. M1 , CA A1 , CB B1 .
k = CA : CA1 M1 N . A1 M1 = p1 CA1 ,
p1 A1 B1 C. : AN = p CA, p ABC.
BM = p CA, AN = BM .
1b
AB
2.46. . , HBb HAa =HCab , AC =
1 ab
(). a = b = k AC =

1
1k
1
AB
= . ,
2
1 k2
2

k 6= 1, (. 15.1).
k = 1.
1
k
2.47. Tr HOk = HAk , OA =
r. HOk Tr = HBk , OB =
r.
1k

1k

2.48. HAk , HBk , HCk (k 6= 1). HBk HAk =


k2
k2
= HM
, M (AB). , HCk HM

G ABC, , M , C, G
. M AB, ( 2.46).
2.49. : H M P (N ) = Q, H P R (Q) = S. H M R , N S.
((M P ) (N Q), (P R) (QS), (M R) (N S)) . .
2.50. HAk1 HBk2 . HAk1 (M ) = N
k2
HB1 (M ) = N , HB1/k2 (HAk1 (M )) = M . k1 6= k2 HB1/k2 HAk1 M . k1 = k2 , HB1/k2 HAk1
M .
2.51. .
, 2.50, 2.47.
295

2.52. (N P ) (AB) = X (N Q) (AB) = Y ,


1 16 , M XY .
, (M N ) N X N Y .
CN
2.53. HPBC HD
, , , B N . k1 k2
(. 139):
k1 k2 =

P C DN
MA MB
MA
CN
QN

=
=
=
.
P B DC
M B AB
AB
AB
QB

Q ,
, Q (P D).
B

D
. 139

2.54. B .
( ):


CA1 BC1 AB1
A1 C B1 A C1 B

= 1.
A1 B

B1 C

C1 A

A1 B

C1 A

B1 C

B.
2.56. , w1 w2 w -

A1 O
, : w1 w, O1 O,
A1 O1
A2 O
B1 M . HAk22 , k2 =
, : w w2 , O O2 ,
A2 O

. HAk11 , k1 =

M B2 . ( )
HSk , k = k1 k2 , . k 6= 1 S A1 A2 . HSk (O1 ) = O2 HSk (B1 ) = B2 , O1 O2 B1 B2 S A1 A2 . k = 1,
. (A1 A2 ) k (O1 O2 ) k (B1 B2 ).

.
2.57. , m, n, p,
HQ2 ABC, 296

( 1/2)
. 1, .
2.58.
, (. 18.2),
.
2.59. A, AC AB k = AB : AC k,
k ,
CAB.
2.60. AC BD . AB CD AD BC,
.
2.61. r1 r2 w1 w2 , O1
O2 , ,
w1 w2 , , M
M O1 : M O2 = r1 : r2 .
2.62. : OB : OA = k, OC : OB = k.
OC : OA = k 2 .
2.63. m f ,
. f(m0 ) = m f(m) = m1 , m0 m
m m1 f.
2.64. f k, O f(O) = O1 , OO1 1 : k.
2.65.
O, 45
2 A B A1 B1 , ,
AA1 BB1 . 45 .
90
90
, RO
(A) = C RO
(A1 ) = C1 . AA1 CC1 .
2.66. N DE, 1 21 (CM )
(DN ). (AE) k (DN ), (CM ) (AE).
2.67. K 90 , B P C B, D Q,
ABCD ,
P B BQ. (DK) (QK).
2.68. M , A1 , C, B1 g
M C. f M , ABC g ( 3 21). A B1 , B A1 , , P Q. P M Q = BM A =
297

MQ

M A1

= OM C = a ( )
=
= k ( ).
MP
MB
, P M Q M BA1 , P QM = BA1 M = 90 . AB
B1 A1 a , a = OM C.
2.69. ABC. 1
2 21.
.
2 cos(a/2)
2.70. A0 , B0 , C0 P
BC, CA, AB ( 1 14). A2 = (BC) (B1 C1 ),
B2 = (CA) (C1 A1 ), C2 = (AB) (A1 B1 ) A0 ,
B0 , C0 O
a
1


.
2
|cos(a/2)|
A2 , B2 , C2 .
2.71.
w w1 , w w1 , A A1 ( 3
21) w A w1
A1 . ,
.
2.72. a b M a, P a, N b, Q b,
A, a b, P Q ( 3 21) , , ABP AN Q. 1

: P M A + AN Q = `ABP + `ABQ = `AN Q + `ABQ = 180 .


2
2
2
2
(M P ) k (N Q).
2.73. S, B, A, M w SM .
S, B1 , A1 , N w1 SN . O
,
O, w w1 , A B
A1 B1 ( 3 21). ,
AB A1 B1 f .
2.74.
90 N .
N , AB
. N = M .
2.75. (AC) (BD) = M . A D, B C, M M
, m n (AB) (CD). m n AM BM
( ).
2.76. , , . a b
298

b c, . , (a, b) + (b, c) = 0.
, (a, b) + (b, c) = (a, c). , (a, c) = 0
, , a k c.
2.77. ACD CBD
D , ADC. AN CM
.
2.78.
AOB O k = OB : OA = OD : OC ACA1
BDB1 .
2.79. A0 (2; 1) B0 (3; 2). ,
A0 A, B0 B,
. .
2.80. a(O, R) ,
ABC , A
a, B a D B AC.
BD : AD = 3, f D 90
3 A B. B

1
a f (a) = a1 . DO < (1 + 3)R
2

1
1
, DO = (1 + 3)R , DO > (1 + 3)R
2
2
.
2.81. ABCD . f A, B D.
f (C) = C1 , C, D, C1
C1
D
(. 140), ABC + ADC =

C
= 180 ADC1 + ADC = 180 . DC1 AD : AB = DC1 : BC. D1
A
B
A C1 C
k = AD : AB f , DA . . 140
, C, D, C1 ,
A (D, DA) , CC1 k.
ACB .
2.82. A1 B1 C1 a,
b, c. ABC,
A1 B1 C1 , A a, B b, C c. f
A a A1 k = A1 C1 : A1 B1
299

B C b b1 . , C = c b1 .
B = f 1 (C). A , :
f = B1 A1 C1 ,
f.
2.83. M1 A a
a
M 90 2 sin .
2

M2 M3 B C
. M1 , M2 , M3 ,
A, B, C.
2.84. M , A B M M1 M M2 , (AB) k (M1 M2 ).
M3 (M1 M2 ). (M M3 ) (AB) = D. g = 90 ,
CD = M D = DM3 . M
(. 141): D (AB) DC
DC, M1
M3
M2
CDM ,
CDM = 90 . M
D A
D

C 45 2, M , C
M
, ,
AB .
2.85.
2.82.
. 141
2.86. ABCD
M N P Q (M (AB), N (BC), P (CD),
Q (AD)) O ( 1 . 2.2). M ON =
= a ON : OM = k . f O
a k M N , N = (BC) f (AB).
a 6= A a 6= 90 , ,
a, a, 180 a a 180 . a = 90 6= A
. a = A 6= 90 k 6= AB : BC
. a = A k = AB : BC, ,
a = A = 90 k 6= AB : BC .
2.87. B C f
A, l k = AC : AB = m : n,
m n . a b B a,
C b, f (a) = a1 , C (a1 b).
a1 b.
300

2.88. A A1 a k, a O , AOA1 = a OA1 : OA = k.


O , A1 A k, , a () AA1 .
2.89. m f f(A) = A1 . M M1 m A A1 , f(M ) = M1 .
O (. 20.2).
. OA1 : OA = AM1 : AM = k, O
m w
A1 A k. (A1 A) m = M0 , m w = {M0 , O}, M0 O.
2.90. A, B, C, D , 12
, , (AB) (CD), (AC) (BD), (AD) (BC) .
2.91. a = (AB) .
O a. O , a
O,
.
2.92. .
,
. ,
.
2.93. C , CA CB
.
2.94. A c, B g. A B
f = g c: f(A) = g(c(A)) = g(A) = A1 , f(B) = g(c(B)) = g(B0 ) = B1 .
B0 (AB), B1 (A1 B).
( 3 21), , Q ABA1
, A1 B1 AA1
A1 .
2.95. A1 B1 : AB = k, AA1 , BB1 , CC1
1

, A A1 , B B1 ,
k
C C1 . ,
1

. k
.
301

2.96. O 90 . A R, B P , C B, D Q
.

O .
2.97.
, ABC
A1 B1 C1 .
O O1
, O1 O A1 B1 : AB (. 20.4).
2.98. C1 B1 A1 C ,
BC A C
.
2.99.
ABP A1 B1 P , BCP B1 C1 P Q
ABC A1 B1 C1 . ABP
BCP B, P , Q .
A1 B1 P B1 C1 P .
2.100. (AA1 ) (BB1 ) = P , (BB1 ) (CC1 ) = Q (. 142). , ABC
A1 B1 C1 , (ABP ) (BCQ).
(A1 B1 P )
(B1 C1 Q). , ABC A2 B2 C2 , (ABP )
(BCQ). A1 B1 C1 A2 B2 C2
(A1 B1 P ) (B1 C1 Q). , .

C2

C1

B1

B2

A1 A2
. 142

2.101. . 20.3 , A B
A1 B1 , S, .
AA1 : BB1 = SA : SB. S, A B A2 B2 . , AA2 : BB2 = SA : SB , ,
AA1 : BB1 = AA2 : BB2 , AA1 : AA2 = BB1 : BB2 .
BB1 : BB2 = CC1 : CC2 .
302

2.102. B . 360 . , B
k 3 .
2.103. O ABC 120
A, B, C B, C, A,
A1 , B1 , C1
B1 , C1 , A1 , A2 B2 , B2 C2 , C2 A2 , . . O.
A1 B1 C1 ABC fO , A2 B2 C2 A1 B1 C1 yO .
4A2 B2 C2 = yO (fO (4ABC)). , yO fO
. , y = f, ,
O.
2.104. P w, ABC, A1 , B1 , C1
BC, CA, AB . A, B1 , C1 , P
a P A (. 143). A1 , B1 , C, P
g P C.
P , A1
g
C
P
g w, A1 B.
a
P , w a, w
B C1 . B1
g a A1 C1 .
g a = {P, B1 }, B1 (A1 C1 ) ( 3 21).
C1
B
2.105. (AD) (BC) = O, B1 OC1 = A
= A1 OD1 (). , OB1 OC1 , A1
. 143
D1 , , B1 A1 C1 D1 .
2.106. B C A, BAC k = AC : AB. B
m, C , m .
2.107. S1 S
a (. 20.1), S S1 SS1
a 180 a. , , S S1 .
2.108. O f .
. h h1 ,
O 303

, . 2.72
,
O f .
2.109. , a (M X) A CAB k = AC : AB.
2.110.
ABC A1 B1 C1 .
2.111. A1 B1 C1 A2 B2 C2 :
B1 A1 C1 = B2 A2 C2 A1 C1 : A1 B1 = A2 C2 : A2 B2 = k.
=

A1 A B1 B
=
=
AA2 BB 2

C1 C
A1 B 1 + lA2 B 2
A1 C 1 + lA2 C 2
= l, AB =
AC =
.
1+l
1+l
CC 2

A1 B 1 A2 B 2 f a k ( ): A1 B 1 A1 C 1 A2 B 2 A2 C 2 . ,
AB AC f . ABC
A1 B1 C1 A2 B2 C2 .
a
2.112. RPa (M ) = Q. , P M Q = 90 M Q : M P =
2
a
2 sin(a/2)
90 a/2
RM
= 2 sin . Q P HM
.
2

, w
, . . .
2.113. A0 , B0 , C0
BC, CA, AB. f , ,
A0 A2 , B0 B2 , C0 C2 , f
1
I,

. , 2
cos(f/2)
A2 B2 C2 A0 B0 C0 , 1
2

1
2

1
2

A0 = (B + C), B0 = (C + A), C0 = (A + B).


2.114.
.
2.115. , , ,
.
2.117. , . ,
.
2.119. . ,
.
304

2.120.
.
2.121. 2 24.
2.126. ,
.
2.127. 1/5.
2.128. ABC. AM = BM1 = x, AP = CP1 = y,
BN = CN1 = z, M N P M1 N1 P1 x, y, z.
2.131.
, .
2.132.
1
1
+
=
ABCD. :
AP
AR

2
. AQ A AP R.
AQ

2AP AR
, AQ =
.
AP + AR

2.133. ABC P A1 + P B1 + P C1
.
2.134. 4S.

III
3.01. OM OM 0 = R2
AOM BOM 0 .
3.02. (O, R) , M M 0
,
OM : R.
3.04.
.
3.05. .
3.07.
. ,
.
3.08. .
3.09. A.
3.10. .
3.11. 1 29.
305

3.12. A B a.
A a a0 B
B 0 , B 0 a0 . B 0 a0
.
3.13. 28 .
3.14. a b , w w1 , a b,
w, w1 a b .
, w w1 .
3.15. , ABD BDC . B . A0 C 0 D0 .
3.16. . 3 29.
3.17. c1 , c2 , c3 , c4 c1 c2 = {A1 , B1 },
c2 c3 = {A2 , B2 }, c3 c4 = {A3 , B3 }, c4 c1 = {A4 , B4 }.
A1 , A2 , A3 , A4 g ( ). A4 g, c1 , c4 ,
c2 c3 . , B10 B20 B30 B40 180 .
3.17 : AB, BC, CA,
ABC, B1 , C1 , A1 , AB1 C1 ,
BC1 A1 , CA1 B1 , .
3.18. O
.
3.19. ,
. , - ,
( 3.06).

, , .
.
3.20.
, ( , . 26.2).
3.21. a b, w,
a b. A w w ,
w ( 3.20). a b A .
306

[1] . . .: , 1957.
[2] . ., . .
. .: , 1955.
[3] . . . 79. .: ,
1990.
[4] . ., . .
// .
. 4. .: , 1966. . 307348.
[5] . . , . . . . .: , 1964.
[6] . . , ,
. .: , 1965.
[7] . ., . .
. .: , 1979.
[8] . ., . . . : , 1988.
[9] . ., . ., . .
. .: , 1965.
[10] . . . .: ,
1962.
[11] . ., . . //
. 1972. 2. . 3641.
[12] . . . .: , 1962.
[13] . . ., . . . .: , 1978.
[14] . . . . .: , 1966.

[15] . ., . . // . 1977. 6.
. 5863.
[16] . . . : . -, 1977.
[17] . .
// . 1978. 5. . 8284.
[18] . ., . . . .: - , 1961.
[19] . . . . 1. .
.: , 1948.
[20] . ., . . . : , 1981.
[21] . . // . 1979. 3. . 6267.
[22] . . // .
1991. 10. . 4852.
[23] / . . .
.: , 1982.
[24] . . , 2- . .: ,
1986.
[25] . . . .: , 1975.
[26] 910
/ . . . . .: , 1971.
[27] . ., . . . .: , 1962.
[28] . . . .: ,
1990.
308

[29] . . , . .
. : - ,
1974.
[30] . . //
. 1973. 6.
[31] . . // . 1975. 7.
[32] . .
// . 1974. 1.
[33] . . . . .: , 1982.
[34] . . // . 1976. 11.
. 2230.
[35] . .
// . 1990. 8. . 6669.
[36] . ., . ., . . . .: , 1970.
[37] . ., . ., . . . .: , 1974.
[38] . ., . ., . . . .: , 1967.
[39] . . //
. 1976. 3.
[40] . . , 2- . .:
. 1955. 1956.
[41] . . , . . // . . 4. .: , 1963.
[42] A. M. // . 1975. 5.

309

, 130
, 40
, 48
, 18
, 107
, 17
, 65
, 111
,
115
, 64
, 95
, 56
, 269
, 23
, 45
, 48
, 120
, 125
, 34
, 33
, 131
, 131
, 129
, 129
, 131

, 131
, 48
, 70
, 60
, 72
, 36
, 122
, 118, 122
, 121
, 81
, 83
, 96
, 96
, 15, 96
, 96
, 118
, 75, 132
, 20
, 70, 88
, 70
, 39
, 23, 80
, 69
, 53
, 75
, 72
, 57
, 56
, 21
, 13
, 45

, 66
, 28
, 27
, 145
, 107

, 24
, 38
, 36, 43, 78
, 22
, 30
, 77

, 86

311


. 1. ,
.
. .


01335 24.03.2000 .
07.09.2004 . 60 90 1/16. 1.
. . . 19,5. 2000 .

119002, , ., 11

. 143200, . , . , . 93.

, ., . 11. . 2417285. E-mail: biblio@mccme.ru

Вам также может понравиться